The copyrights of the commentaries themselves remain with their authors. My responses and incidental text are copyright © 1999-2010 Dennis Paul Himes


Commentary on An Atheist Apology

Text in normal font is commentary on my essay An Atheist Apology. Responses by me to that commentary are in italics.

I've started labeling the anonymous replies Anonymous A, Anonymous B, etc., in order to keep track of when they're the same person.

I put the notation "[sic]" after spelling or grammatical errors in the replies to indicate that the errors were in the original text. In the interest of fairness, I've started doing the same to errors in my own responses which I notice after I first post them.

  • Gail Gunst with an indirect ad hominem attack
  • Brian Mason with some interesting metaphysical questions
  • John Brienesse with a prediction for the future of religion
  • Anonymous A on the difference between religion and science
  • Anonymous B on the nature of theists
  • Chris Ho-Stuart with a link to a similar site
  • JJ with some thoughts on the persistence of religion
  • Hendrik Little on types of universes and Occam's razor
  • Hendrik Little on St. Anselm's argument
  • Anonymous C on the purpose of the Apology
  • Anonymous C on reasons for belief
  • Anonymous C on the limits of proof
  • Jeffrey Henning with questions on the nature of reality
  • Anonymous C, continuing the previous discussions
  • Anonymous C, ditto
  • Anonymous C, summing up
  • Etienne O'Hanlon on actuality, possibility, and Hugh Everet's theories
  • Noelle Morris, relating a theory of a temporally infinite universe
  • Anonymous D, questioning my integrity
  • Anonymous E with a related site
  • Muke Tever on existence and omniscience
  • Muke Tever, continuing the previous discussion
  • Michael Fretheim with a succession of false statements
  • Leonard Hambleton, noting the popularity of one of my analogies
  • Anonymous F on the different types of gods
  • Anonymous G with an agnostic viewpoint
  • Veracity, arguing against both atheism and Catholicism
  • Anonymous H with some responses to these responses
  • db on the future of the universe
  • Anonymous I, apparently responding to some other essay
  • David Singer, regretting that I haven't convinced him
  • Daniel Ketterman on subjectivity
  • Wldmn87, spewing insults
  • Fruity Magick on the limits of knowledge
  • Anonymous J on the argument from design
  • Anonymous K with some predictions for the future
  • Ian Maxwell in agreement
  • James Quinn, discussing the beginning of time and the qualifications for a religious vocation
  • Jerod Romine on sequential universes
  • David Singer with some thoughts on omnipotence
  • Paul Doland, agreeing, but not completely
  • Anonymous L, recapitulating
  • Anonymous M, only partially persuaded
  • Anonymous M on theists and reason
  • Ian Maxwell on higher consciousnesses
  • Paul Doland with some questions on omniscience and potentiality
  • J. Nemeth, unconvinced
  • Denn 272 with a question
  • Michael Jacques with some questions about Christianity
  • Damers587 with a barely coherent rant
  • Michael Jacques, disputing my previous reply
  • Michael Jacques, continuing the discussion
  • Denn 272, following up
  • Anonymous N with various reasons to believe
  • Paul Doland on actuality versus potentiality
  • Paul Doland, continuing the discussion
  • Paul Doland, continuing the discussion
  • Paul Doland, continuing the discussion
  • Paul Doland, discontinuing the discussion
  • Paul Doland, restarting the discussion
  • Paul Doland with a question
  • Paul Doland, discussing burdens of proof
  • Anonymous O with some questions about assumptions and definitions
  • Anonymous P with some thoughts about God and the afterlife
  • Anonymous P on causuality
  • Anonymous O on the equivalence of belief
  • Anonymous P on free will and the limits of knowledge
  • Anonymous P on nothing[ness]
  • Anonymous P, clarifying previous arguments
  • Angelo Salvo, explaining his own Atheism
  • Bomar Angelopolis on the origins of the concept of God
  • Kristen Gardner with some easy questions
  • Florian Blonigen on cosmology and temporal existence
  • Florian Blonigen on necessity and contingency
  • Florian Blonigen continuing the previous discussion
  • Spike McLarty on the nature of reality
  • Leonardo Castro on the Big Bang
  • David Palmer on assumptions and "materialism"
  • David Palmer, future millionaire

  • From: Mr Gail Gunst
    Date: Wed, 24 Feb 1999 15:51:45 -0600

    You, my friend, are a classic example of the type of person described in the Bible, in Romans 1:18-32,

    (Note: Romans 1:18-32 is an extended ad hominem argument against paganism. It includes such gems as: "And so they are steeped in all sorts of depravity, rottenness, greed and malice, and addicted to envy, murder, wrangling, treachery and spite. Libelers, slanderers, enemies of God, rude, arrogant and boastful, enterprising in sin, rebellious to parents, without brains, honor, love or pity.")

    Do you not understand the difference between paganism and atheism, or do you figure that it doesn't matter, since St. Paul makes no attempt at rational discourse anyway?

    continuing into Chapter 2.

    (Romans 2:1 is "So no matter who you are, if you pass judgment you have no excuse. In judging others you condemn yourself, since you behave no differently from those you judge", which is an odd thing to say right after delivering a vicious ad hominem attack.)

    You evidently feel some kind of "guilt" for stating you don't believe in God, otherwise why have you written an apology?

    It's not clear to me whether you're being facetious or whether you really don't know what "apology" means in a religious context. An "apology" is a reasoned argument in support of some philosophical position. I named my essay in conscious imitation of the apologies written by Christians in defense of their faith.

    I am one of "those" who has [sic] placed my faith in God, through the blood of His Son, Jesus, shed on the cross of calvary [sic]. And I don't ever "apologize" for that!

    I note that your last sentence is true using both meanings of "apology", since you have made no attempt whatsoever to refute anything I said in my essay.


    From: Brian Mason
    Date: Wed, 07 Apr 1999 03:43:38 -0600

    Thanks for the interesting apology. It was a good read.

    You're welcome.

    It always strikes me as interesting in these types of debates how much easier it is to knock the legs out of the opposing theory than it is to keep our own upright. In particular, I have spent some time tonight wondering about your first statement in your contradiction of omnipotent agency.

    The statement "If the universe did not exist it would still be a theoretical possibility." could only be true if it were true that anything could be a theoretical possibility under those conditions.

    By "theoretically possible" I meant that it is internally consistent, that Dennis Himes updating his webpage on 12 Apr. 1999 is a consequence of this universe's existence, regardless of whether that existence is realized in some metaphysical sense. The point I was leading up to is that external "conditions" are irrelevant; an internally consistent universe exists relative to itself tautologically.

    It is a little like saying "The universe is the type of place such that if it weren't such a place, at least it could be." It's not that this statement is false; it's just that it's not verifiable. I would have the same problem with somebody who insisted that "The universe is the type of place such that if it weren't such a place, then it couldn't be." How could we possibly know?

    There are several ways to show that a given universe could or couldn't exist. We would know that a universe couldn't exist if we could find a contradiction in its definition. So a universe with the following three properties couldn't exist:

    1. The number of protons never changes.
    2. At time = 0 the number of protons is 10^40.
    3. At time = 1 the number of protons is 10^39.

    Showing that a nontrivial universe could exist is harder. One way, however, would be to exhibit an example, which is precisely what we can do for ours. If our universe wasn't internally consistent, then it wouldn't exist. It does exist, therefore it is internally consistent, and since internal consistency only involves its relation to itself, that consistency would not change if it didn't exist in some other frame of reference.

    I suppose one of the qualities of this universe may be that theory or possibility or existence may exist even if the universe did not, but it is not so difficult to imagine that the universe does not possess such a quality.

    If the universe did not possess that quality it would be inconsistent, and therefore everything would be true and everything would be false. My experience is that we do not live in a universe in which everything is true and everything is false.

    In fact, isn't that what you pretty much imagine your death to be like? Am I wrong or would you tell me "When I die, there will be no such thing as theoretical possibilities for me. It will not be true for me that 2 and 2 make 4. It will not be true for me that I was ever alive. There will be nothing and not even nothing for never and never."

    But if you hold that to be true, then how can you believe that first statement?

    I'm not sure what you mean by "true for me". I will not be able to verify that 2 + 2 = 4 (given, say, the axioms of Peano Arithmetic), but that is true regardless of my verification. Even nontautological phenomena of this universe, such as the proposition that the sun will emit light on 2 May 2254, is true or not depending on the nature of the universe, and not on my verification of it.

    Even in my simplified mental model of the universe the statement "the sun will emit light on 2 May 2254" is true, regardless of whether I'm alive in 2254 to verify it.

    To put it another way, I believe that my mental states are a consequence of the universe's reality, not vice versa.

    Anyway, as I said before, I think you have done a fine job of explaining your position.

    Thank you.


    From: John Brienesse
    Date: Sun, 2 May 1999 19:59:21 -0400

    Hello!! I read your apology; and since I am an atheist myself, I agree with it.

    But!! Since the internet is in the hands of anybody right now, there will be hundreds of thousands that do not understand what they are reading. I am an ordinary person and have made my living playing and teaching the violin.

    I needed an encyclopedia and a dictionary plus some study to understand what you where writing about. I Iearned [sic] my English out of comic books. When I came to Canada I knew what was "yes" and what was "no" . My Atheism stands for the point that: If there was an "omnipotent" God. He would have the power to stop those witch burners. Because he says: Love your enemies.

    That's a fair criticism of the great majority of theists' beliefs, but not necessarily all of them, since you're assuming that an omnipotent God would care, and benevolence is not a priori a trait of an omnipotent God.

    Also the bible itself is so full of holes you can use it for a sieve. And an omnipotent God would make sure that his word would not be falsified, because of all those denominations and cults.

    A belief in the literal truth of the Bible is a form of insanity. I have tried to address more reasonable theist arguments. I have also tried to avoid arguments which assign human characteristics to a deity, such as kindness or a desire to communicate, since I felt that more general arguments would leave less loopholes for theists to use. However, the arguments you make here are perfectly good ones against almost all the theists you're likely to meet.

    I believe that atheism is not only for the learned, but for all people. I am not trying to be funny, but as soon the people get a view about what is going on, those Vaticans and synods will cease to be. It is a good sign that the churches are getting very empty, and many are closing.

    You are more optimistic than I. I hope you are right.


    From: Anonymous A
    Date: Sun, 27 Jun 1999 15:56:04 -0500

    Religion is the mythology of human ego over reasoning. It has no merit as it can only support itself by believing. It has not invented nor is it provable by any mean's [sic] other to say I believe therefore it be true.

    Science most [sic] meet all criteria of proof and withstand the test of time never being wrong. It's [sic] aim is not theological but understanding why. The Hot Big Bang has withstood these test's [sic], there are many and changing event's [sic] needing interpretation and resolving, that is the beauty of science. It is not static or divine.

    Religions have been known to make falsifiable predictions. Unlike science, however, when those predictions are in fact falsified the underlying theory is not abandoned.

    Quantum mechanics with particle accelerators have proven quarks and anti-matter and the larger one's [sic] will show quantum effects demonstrating miniature controlled Big Bangs. Where is God to be or what would he have to do? He would reside in our universe as the expansion creates SPACE AND TIME.

    What would create the creator and from what?

    Most theists would say that the creator was not created, hoping that no one notices that they just denied the principle that complexity implies creation.


    From: Anonymous B
    Date: Tue, 6 Jul 1999 04:46:03 EDT

    An extremeley [sic] competant [sic] exegesis of the athiest position.

    Thank you.

    It's a pity that most theists will be too intellectually feeble to follow your arguments. Most of the theists I know are of the "I hope there is a god cos it would be nice" variety which is especially galling for me when I try too [sic] discuss their beliefs in an objective way. There is just no getting round this obstacle. I suppose it's part of the human condition in coping with the knowledge of death.

    Yes, I think fear of death is a major reason for religion's persistence.


    From: Chris Ho-Stuart
    Date: Mon, 19 Jul 1999 10:11:30 +1000

    Very clearly put together. I like the lack of stridency.

    Thank you. I also dislike the stridency one sometimes finds on atheists' web sites.

    You may also be interested in an apologetics for strong atheism which I wrote some years ago. We have quite a deal in common. My apologetics is on the web at http://www.fit.qut.edu.au/~hostuart/why.atheism.html

    Very well done. You are not quite as "strong" an atheist as I, though. I would put a higher degree of confidence on the statement "God does not exist." than I would on "Dodos do not exist." (depending, of course, on which definition of "God" is being used).

    (Note: His essay, as he notes, has a great deal in common with mine, but with different emphases. He only devotes a little discussion to cosmological issues, which I discuss extensively, and he has a whole section on spirituality, which I barely mention.)


    From: JJ
    Date: Mon, 26 Jul 1999 18:59:53 -0400

    Interesting ideas, I enjoyed your piece. (Haven't gotten to the rest of your page yet) I always find discussions concerning the origin of the universe(s) even more interesting when posed within the arena of another discipline. (theology, mathematics, astronomy etc.) As for applying it as a validation of atheism- I agree. I was left wondering, however, what your thoughts are on why humans are so desperate for a god. I don't mean primitive humans, I mean modern, educated humans. I agree with the theories on how religion began- psychlogical [sic] need for explanation, security and so on. I can't resist thinking that today it remains only because of social pressures and obstinance. I think this is also the very reason theists are so consumed with "proving" and "defending" their beliefs even in the face of overwhelming evidence against it [sic]. A hero defending rightousness [sic] and the church has always been an honored position- even if very few really believe in their chosen god. Belonging to a group is a good feeling. I didn't mean this as an attack, I just think that god came from the minds of humans and it would be enlightening to explains [sic] why it remains. Our brains were all designed the same way- to successfully discuss questions about the universe, it can't hurt to know all the pitfalls we may be vulnerable to.

    I think that, in addition to the reasons you give, religion persists because it allows people to avoid dealing with the fact that they're going to die. Also, religous leaders have put great effort into conflating religion and morality in people's minds, to the extent that some people are afraid to question their "beliefs" for fear of discovering that they're immoral.

    Of course, if you're talking about theism in general, and not just its obviously false manifestations, some people believe in it simply because they've made a honest but faulty analysis of the possibilities.

    On the other hand, I never believed in a higher power- is that good or bad?

    I'd say good, so long as you don't continue to not believe simply because you never have.


    From: Hendrik Little
    Date: Thu, 28 Oct 1999 05:35:22 -0700

    I used to be an agnostic but I'm alright now! I am an extremely strong atheist but I like to go and criticise other atheist sites just to make sure that I haven't lost sight of why I hold the beliefs that I do.

    I am unsure as to the validity of how you define a universe but it seems evident that you have a greater grounding in philosophy than I do so I won't argue the point too much with you.

    I personally wouldn't absolutely rule out inconsistent universes and don't believe you can prove that we are not living in a universe where everything is true and untrue simultaneously. I'm sorry to just make that assertion but by making the statement I'm afraid you've placed the burden of proof upon yourself.

    Actually, it's quite easy to prove.

    Assume: The universe is inconsistent.
    ---- Because everything is true, the universe is consistent.
    ---- This is a contradiction.
    Therefore the universe is not inconsistent, by reductio ad absurdem.

    To put it another way.

    Either the universe is consistent or it is inconsistent.
    ---- If it is consistent,
    -------- There's no problem.
    ---- If it is inconsistent,
    -------- Because everything is false, Hendrik Little never asked me to show that the universe is consistent.
    -------- What was the question, again?

    Now, I'm cheating a little here. If you're familiar with Gödel's theorems or are just clever enough, you'll realize that the same arguments hold if the universe is inconsistent. (Since everything is true in an inconsistent system, it's easy to prove the system's consistency.) The basic reason I assume consistency in my analyses (going so far as to say that inconsistent universes don't really exist) is because an analysis, or a discussion, implies consistency. If the universe was inconsistent it would be impossible to interpret your email to me, because every word would have every possible meaning and every word would have no meaning. If the question "Is the universe consistent?" means anything at all then its answer is "yes".

    Also, there are much simpler logical arguments against omnipotent beings, such as could one create a rock heavier than they could lift?

    That argument leaves theists with a huge loophole. I, in fact, had no trouble with that sort of argument back when I was a theist. Omnipotent can be interpreted to mean "able to do anything which has meaning" and then the theist can plausibly claim that "a rock heavier than God can lift" does not have meaning.

    My argument is more general in that it applies to any definition of omnipotent which includes omniscience.

    My main arguement [sic], though, is with the distinction you make between strong and weak atheists.

    I'm not sure what you mean. Are you saying that the definitions I gave are not standard terminology?

    I personally am an atheist precisely because of the undisprovability of gods. If a being could set up a universe so that it looks indistinguishably like it came into existence on its own then what is the point of believing in that god?

    This is a good example of Occam's razor.

    I refer you to Greg Egan's 'the Church of the God that makes no difference" in "Permutation City" (http://www.netspace.net.au/~gregegan/). (At the end they manage to disprove the existence of gods but the arguement [sic] is a little convoluted and in another universe so it has to be watched.)

    That URL doesn't work at all in Netscape. (Only the background appears.) In IE it comes up OK, but I couldn't find the reference to "Permutation City".

    A theist might argue that it would be possible for that god to prove their existence at any point, and indeed that they have done so in the past, but I personally challenge them to come up with a "proof" that could not be simulated by my own insanity, which is perfectly explicable in the universe as I understand it.

    BTW is their any good logical proof or evidence, other than statistical, for Occams Razor? I personally believe it but it seems to be presented by most people as just that, either an article of faith or a fundemental axiom and we all know the dangers in that kind of behaviour.

    Statistical evidence is hardly the same sort of justification as an article of faith. More to the point, if a phenomenon can be explained by A and B or just by A alone, then it provides no evidence for B, so believing B on the basis of that phenomenon is belief on no evidence.


    From: Hendrik Little
    Date: Thu, 28 Oct 1999 06:25:44 -0700

    Having thought about it over lunch one or two other things that were bothering me have crystallised somewhat.

    The flaw with St Anselm's argument is not that the atheists [sic] position is assumed to be "There exists x such that x is God and x does not exist." A theist could define "greatness" as being the ability to affect ones [sic] environment in a conscious fashion. Therefore we can safely assume that such a being exists ie. a person.

    That's true, but I fail to see what your point is here. The existence of a great being which is not God doesn't seem to affect the argument one way or another.

    What St Anselm has succeeded in doing is creating a necessary condition for being God, not that proving that God exists. Just because it is possible to conceive of a being greater than the greatest possible person and then some does not mean that being exists.

    But that's not St. Anselm's argument. The contradiction he thought he had produced was not conceiving of a being greater than the greatest possible person, but rather conceiving of a being greater than a being for which a greater being cannot be conceived. Which would have been a contradiction if he had actually made such a conception. The problem is that, in the phrase "I can conceive of a being identical to God except that it exists." the word "God" has no referent. Supposedly it is the "God" in the phrase "Assume God does not exist.", but there is no "God" identified or defined by that phrase; it's just another way of saying, "Every being does not fit the description of God."

    There is no theoretical limit on the "greatness" of a person but there are actual limits. Even if someone's "greatness" could somehow be improved there would still be someone who was the "greatest" with no implication of the existence of God.

    True, but not really relevant to St. Anselm's argument.

    In a similar way you have defined a condition for being Santa Claus and done nothing else.

    My argument for Santa Claus has the same flaw as St. Anselm's argument for God (whatever that flaw might be).

    You have not proved his existence merely by defining what he would be like did he exist. You yourself are arguing for St. Anselm in this respect by making no distinction between conception and existence for unbounded entities, on one hand God and on the other the Universe.

    Well, "The Universe exists." is simply a tautology, since "the Universe" means "all that exists." If a universe with God were logically consistent then of course it would exist relative to itself. However, I showed that it is not.

    Even if this were not the case your argument would be flawed. I could define a Santa Claus which would be impossible for you to improve on, the one that fits your definition fully, exactly and completely ! There is no limit on "greatness", potentially it could be infinite but one cannot resemble a description more completely then matching the description exactly. The set of potential Santa Clauses is closed whereas the set of beings resembling God is open and unbounded.

    True, but again I don't see the relevance. What step of St. Anselm's argument works differently from the matching step in the Santa Claus argument because of this fact?


    From: Anonymous C
    Date: Mon, 15 Nov 1999 16:07:07 -0600

    Interesting argument. Being neither a philosopher nor a theologian, I make no attempt to refute your apology.

    I happen to be a believer in God. I see no reason to argue about the existence of that which I know to be true. God is perfectly capable of making Himself known, should He choose to do so. I personally find it absurd to try to prove or disprove an "infinite, supernatural being" through the use of "finite, natural" arguments using an inadequate vocabulary.

    Mathematicians produce finite proofs of infinite properties quite often.

    My question to you ( to atheists and theists alike, actually) is WHY attempt to prove or disprove the existence of God? You are entitled to your belief system, as I am to mine.

    At least one of us is wrong. If you are wrong, then I am doing you a favor by showing you your error. If I am wrong, then you would be doing me a favor by showing me my error. Even if I were to remain unconvinced, you would be helping yourself by augmenting your beliefs with your counter-argument.

    I could go into many reasons why I personally believe in God, none of which meet your criteria for "proof of God". And you could go on with your philosophical arguments, none of which meet my criteria for "disproof of God". So we remain at a standstill.

    If you cannot find a flaw in my reasoning, then I think you should be asking yourself whether you really believe in God, or are just pretending to.

    Is your goal to convince the world that God does not exist? Is that the point of your webpage? I know lots of people who are atheists, others who are agnostic, others who are theists. Most of them are pretty well founded in whichever belief system they adhere to. I think the attempt to change someone's belief system, either from or to atheism, through a website such as this is futile. But I admire you for making the attempt!

    As I noted in the section titled "Why?", the essay's main motivation was to explain my position to those who know me. However, convincing the world, or even a small portion of it, would be an added bonus. People do convert, you know. There are also children and young adults who actually decide what they consider to be the truth instead of accepting what they've been taught without really thinking about it.


    From: Anonymous C
    Date: Tue, 30 Nov 1999 00:21:54 -0600

    Thanks for posting my comments.

    You said if I did not find a flaw in your reasoning.....your "reasoning" is inadequate...it is flawed, because it is incomplete.

    You claim in both emails that my reasoning is not merely incomplete, but inaccurate. Specifically, you state that God exists. Since my essay purports to prove the opposite, your claim that God exists is necessarily a claim that my reasoning contains a flaw, a flaw you have yet to identify.

    Science is an explanation of things known...Faith is that portion which science has yet to prove.

    Science is more that an explanation, it is a method for arriving at explanations, a method which has repeatedly succeeded where faith has failed.

    Again, your reasoning does not meet MY CRITERIA for disproof of God.

    I suspect that your criteria are flawed, but I can't really say until you reveal what those criteria are.

    Your Mathematical proof analogy is also inadequate....the infinite properties to which you refer are not "supernatural".....Yes, I believe in the "supernatural"...that which is not explained by natural means (science has yet to explain). I feel I have quite an open mind about such matters; I am quite well-read, and well-educated ..... Just because science has "failed to prove" the existence of God does not mean He does not exist.

    I addressed supernatural explanations in the section Shaving Superman with Occam's Razor, especially the last two paragraphs.

    The distinction between the natural and the supernatural is a red herring, anyway, since religions constantly make scientific claims about the nature of the physical world, such as "God created the Universe", and "God answers prayers".

    You may philosophically "disprove" God, but to my way of thinking, philosophy is a useless discipline other than for exercising the mind (sorry, one could argue until one is blue in the face about that one.) Time is better spent listening to the grass grow.

    Until you say why you think it's useless I can't really answer this. Your argument as you've stated it so far amounts to, "your reasoning doesn't count, and I'm not going to tell you why it doesn't count, it just doesn't".

    Philisophy [sic] is useful for constructing an argument, but it neither proves nor disproves anything.

    The heart of any proof is an argument.

    No one has ever constructed an adequate explanation to prove or disprove "love", but I know it exists. Oh, sure...you can argue about testable evidence, but you still can't adequately define it or prove it. It is much like talking about the existence of air without the proper tools to detect it, as man does does not now have the tools to detect God.

    To the contrary, love is easy to define and its existence is easy to prove, just as faith and its existence are. What cannot be proven, and can, indeed, be disproven, is that love is the result of Cupid shooting people with golden arrows.

    I simply cannot look at the complexity of the universe and believe it "just happened". I find it ludicrous.

    It depends on what you mean by "just happened." It certainly happened. If by "just" you mean without being designed, then you seem to be saying that complexity implies a designer, in which case you must believe that something designed God.

    As to the "Bible" being "full of holes"

    (Note: These are not my words, but John Brienesse's, although I agree with them.)

    ....the people who wrote it were primitive, trying to explain something very complex with an inadequate vocabulary. I don't find it "full of holes" at all. Do I take it literally? The Bible contains law, poetry, myth, analogy, history etc....no, I don't take all of it "literally".

    I believe John Brienesse was responding to those who do, and with respect to their claims it is full of holes.

    Again, I am neither a theologian nor a philosopher. I am a simple person (I'm sure you won't disagree...most atheists tend to take a "superior-mind" point of view, regardless of a 165 IQ on my part.) LOLOL

    Please don't put words in my mouth. I know many intelligent theists.

    Then again, one atheist said I may be a genius, but I'm insane. I don't suffer from insanity, I enjoy every moment of it! LOLOLOL As for "pretending" to believe in God or "just believing what I have been taught"..I believed in God long before I was ever introduced to "teaching about God"......impossible, you say?

    Not at all. Many theists believe because they were taught to, but others believe for a great variety of reasons.

    Not so. My mother can attest to it. We NEVER went to church in my early childhood years, nor prayed, nor read the Bible. I, however, started taking myself to church when I was about 4 years old. My parents didn't send me. I lived in the country and walked. I can't remember a time I did not believe

    I did read some of the other comments: the comment was made that "religion (or the 'invention' of God')" is due to fear of death. I have no fear of death, nor what comes after, be it good, bad, or indifferent. The difference in my life in the here and now due to my "religion" is phenomenal - I don't need to worry about death. That may be one reason why "thinking, educated men" still believe.

    To the contrary, thinking, educated men believe because they have honestly examined the issue and come to a different conclusion than I have. Someone who "believes" because then he or she needn't worry about death is only pretending to believe. I'm not saying this applies to you; lack of worry about death can be an effect of an honest belief, but it can't be a cause of it.

    Another is because they, like myself, look with awe upon the complexity of the universe(s) and cannot imagine it (them) being here by happenstance.

    Lack of imagination is weak evidence, and no proof.

    As for "believing in God" because it would be "nice"...nothing could be further from the truth. It would be much easier NOT to believe than to believe.

    This seems to contradict what you said earlier about death.

    I don't think believing in God is the "nice" thing to do...it is the hard thing to do, because it requires a response and a commitment on my part that I would otherwise not have to bother about. And herein is the problem, lots of people call themselves "Christian", for instance, without the slightest notion of what it means to be "Christian"....Unfortunately, it is this type of "Christian" which gives rise to disgust with "Christianity" (note I do not include the other religions here simply because they are out of my realm of experience).


    From: Anonymous C
    Date: Mon, 6 Dec 1999 00:58:12 -0600

    Thanks again for posting my comments. The difference between us is, you are trying to convince me and I am simply stating my beliefs.

    You are using philosophy again, saying that if I say complexity results by design, then I must believe something designed God. Not so.

    Yes, so. If you are sane you believe at least one of the following statements:

    1. It is possible for something to be complex without being designed.
    2. God is not complex.
    3. God was designed.
    To claim that you believe none of these statements is, as you would say, ludicrous.

    You say the "heart of any proof" is an argument....that is true if you are trying to PROVE something. I am not trying to prove anything, nor to disprove your theory.

    Also, I have never seen a test which 'measures love' .......nor is it easily defined.....but then, neither is "faith".....Your idea of faith seems to differ from my own. Such definitions, or tests, are much like the old saying, "Intelligence is what intelligence tests test".

    Love can be defined several ways, and it may not be clear which is best, but dictionaries still do it. The important thing, though, is that, whatever definition you use, it is not caused by a god shooting arrows.

    Again, you are telling me that I say your reasoning "doesn't count"...That is not what I am saying...

    Let me put it to you another way.....If you can only see one side of a 4-sided house, and it is blue, and someone asks you what color the house is painted, you might be inclined to say "blue"...however, you can only see one side..It is not until you see all four sides of the house that you can, with accuracy, state that the house is, in fact, blue. (Of course, you could also be color-blind!) Actually, wihtout [sic] walking 'round the house, you can't even be sure that all four sides of the house exist....that in fact you may only be seeing a facade. There might be other clues, however.....as to whether other sides exist....you could see the wind blow on one side, but no movement of shrubbery on the opposite side, which would indicate SOMETHING was blocking the wind...and conclude that what you were seeing was more than a facade.

    You are describing what you see...your side of the house, and the path of the wind, and you draw one conclusion...I draw another. Can I then PROVE, beyond a shadow of a doubt that the house has four sides? No. But neither can you "Prove" to me the color of the house (or if the house does or does not exist) from your point of reference.

    Your analogy is flawed, because having four blue sides is not a logical consequence of having one blue side. A better analogy would be this:

    We see a house with one blue side. You say, "Every side of that house is white." I say, "No, they can't all be white, because at least one of them is blue." You say, "You may philosophically 'disprove' all-whiteness, but to my way of thinking, philosophy is a useless discipline other than for exercising the mind. Time is better spent listening to the grass grow."

    Life as we know it, is quite limited. If you were to read e.e. "Doc" Smith's THE LENSMEN series, for instance, he offers superb illustrations of "life" quite different from our definition....our "point of reference". Again, I can't prove God exists, but neither can you disprove it. You can do so only from your point of reference.....might not other points of reference exist?

    What part of the Apology depends on a specific point of reference?

    For clarification, I did not mean that "no fear of death" or "fear of death" is what causes belief....rather, that "the benefits of faith in the here and now" could be the driving force....sorry that I did not make that clear.

    What is the difference between "causing" and "being the driving force"?

    I was not putting words in your mouth...I was trying to "lighten up"...although what I said about many atheists assuming believers are simple is, I have found, to be quite true. My sister among them...although she is now agnostic rather than atheist.

    You say "lack of imagination is weak evidence, and no proof"....you keep looking for proof in my words. I offer no proof.....You would need to read the comments of someone far better at debate and argument and philosophy and theology than I. I am not good at any of those things. I am a really good cook..I sew well, I write nice poetry, I have a great memory, and read voraciously....those things I am good at!

    You offered lack of imagination as evidence, although I will grant that you never claimed it was proof.

    By the way, scientific "proofs" change from time to time. A scientific "proof" is a "proof" only until disproven.

    It must be really frustrating for someone who relies on "proof" and "human reasoning" as much as you seem to do, to run into someone such as myself....one of "blind faith" (as many would put it!).....

    It can be irritating when people say that my reasoning is mistaken but then refuse to point out the mistake. On the other hand, it can be amusing to see people give reasons why one shouldn't be reasonable, proofs that one shouldn't rely on proof, and philosophic arguments on why it's useless to use philosophy.

    No contradiction as to what I said about "death" ....I was referring to the present...here and now.....belief is by far the more difficult path. When I say, "I am not concerned about death"....I mean life, right here, today, counts.....my belief makes each day better than it would otherwise be. When I am dead, I am dead. Do I believe in life after death...yes, I do...(I believe the life force continues...call it life after death or "heaven" or whatever), but even if I did not, I am a better person today, right now, this moment... because of my belief. NOTE! I am speaking only for myself here. I can offer only my own personal experience and belief.


    From: Jeffrey Henning
    Date: Mon, 6 Dec 1999 00:58:12 -0600

    I just spent some time enjoying your web site. That's a fine Apology you've written. Part of the Unsolicited Opinion confuses me, though:

    (Here he quotes from my Unsolicited Opinions page. I have underlined these quotes.)

    What is the difference between a universe which God creates and one which God knows can exist but chooses not to create? Nothing. To God the potential universe is no less real than the "real" one. The term "create" is meaningless when applied to an omniscient being (and omnipotence implies omniscience). The same reasoning applies to changing a given universe. The universe which is changed is no more real to God than the potentially unchanged one.

    In addition, for God to "do" anything she would have to exist in a specific temporal reference; she would have to choose a given configuration for the future from several possibilities, which implies that the "future" has a meaning when applied to her, which is incompatible with omniscience.

    Since 'omniscient' is a key term here, I will quote the American Heritage Dictionary (1992), in case we have different understandings of this word:

    om·nis·cient (¼m-n¹sh".nt) adj. 1. Having total knowledge; knowing everything: an omniscient deity; the omniscient narrator. --om·nis·cient n. 1. One having total knowledge. 2. Omniscient. God. Used with the. [Medieval Latin omnisci¶ns, omniscient- : Latin omni-, omni- + Latin sci¶ns (present participle of scºre, to know; see skei- below).] --om·nis"cience or om·nis "cien·cy n. --om·nis"cient·ly adv.

    What is the difference between a universe which God creates and one which God knows can exist but chooses not to create? Nothing. To God the potential universe is no less real than the "real" one. The term "create" is meaningless when applied to an omniscient being (and omnipotence implies omniscience).

    OK. First, I assume you only bring up omnipotence to prove that God must be omniscient. If so, perhaps that parenthetical remark needs to be removed or made explicit earlier on.

    I'm not sure why. You understood the point of it well enough.

    Here's where I am really stuck. Rather than use the emotionally fraught word God, let's use Agent X. "What is the difference between a universe Agent X creates and one which Agent X knows can exist but chooses not to create? Nothing. To Agent X the potential universe is no less real than the 'real' one." I disagree. OK, I know this universe is real and that one (the one where I went to bed at 10:30 pm like I should have) isn't, and I'm not omniscient.

    How? How do you know this? If you had gone to bed at 10:30, then it would have been theoretically possible for Jeffrey Henning to stay up late. In what way is your knowledge that your universe is real any different from that theoretically possible but nonexistent Jeffry Henning's theoretically possible but nonexistent knowledge that his theoretically possible but nonexistent universe is real?

    It seems to me that an omniscient agent, knowing all, would therefore know what universes are real and what universes are not real.

    This agent could also coin a nonsense term "foo", arbitrarily assign it to a possible universe, and therefore know which universe is foo and which universe is not foo. In what way, from the point of view of this agent, is "foo" any different from "real"?

    Are you saying that every possible universe really exists? Why?

    The fact that a universe exists relative to itself is a tautology.

    While I have often seen this posited in science fiction stories, typically alternate histories, I'm not familiar with how science regards this. And I don't understand why potential universe is no less real than the "real" one. I love, know and treasure the universe of Middle Earth, but it does not exist.

    Of course the universe of the Middle Earth does not exist relative to our universe, but then we do not exist relative to the universe of the Middle Earth. Whether or not the universe of the M.E. exists relative to itself is simply the question of whether or not it is consistent, or, more precisely, whether or not there are one or more consistent universes which fit the descriptions of Tolkien's work. This is beyond my powers to say.

    u·ni·verse (y>"n.-vûrs") n. 1. All matter and energy, including Earth, the galaxies and all therein, and the contents of intergalactic space, regarded as a whole. 2.a. The earth together with all its inhabitants and created things. b. The human race. 3. The sphere or realm in which something exists or takes place. 4. Logic. See universe of discourse. 5. Statistics. See population. [Middle English, from Old French univers, from Latin ¿niversum, from neuter of ¿niversus, whole : ¿nus, one; see oi-no- below + versus, past participle of vertere, to turn; see wer-2 below.]

    You aren't using universe in sense 1 but rather sense 3.

    If "realm" is interpreted so broadly that "our universe" becomes equivalent to definition 1.

    I can create universes in sense 3 easily, as you have done as well. I can know everything about that universe;

    I beg to differ. Do you know the location of every electron in every atom of that universe?

    I can affect everything in that universe. But I can't make the universe real -- it only exists as an idea. If I were omnipotent, couldn't I decide whether or not I wanted to make the universe real?

    If you did, in fact, know everything about that universe, how would you distinguish between its being real and not being real?

    As the creator of a universe, I of course have omnipotent powers to shape that universe. But if I were really omnipotent, I would actually like to temporally enter the universe and interact as a person in a particular time. Indeed, wouldn't that be much of the fun? I can easily imagine myself entering the universe temporally but being omniscient.

    How would you distinguish between a universe which you entered and what a universe you didn't enter could have been had you entered it?

    I have as full a knowledge of my past within our universe (sense 1) as humanly possible, and I can easily imagine having as full a knowledge of the future. Knowledge of the future or past doesn't affect my temporal location.

    How would you distinguish the present from any other point in time?

    Indeed, some experiments have shown results that were affected by future results.

    I'm not sure what you mean by "affect" here.

    So for me it is quite easy to imagine temporal location with universes of omniscient agents.

    By the way, the concept that an omniscient being would not distinguish a "present" from any other point in time was first revealed to me by the nuns who taught me in Catholic school.

    I think I need a fuller explanation of the universe of universes and the position an omniscient being needs to be relative to the universe. I can envision being omniscient within a universe, knowing everything past and present, yet being compelled to follow the same choices in my future -- otherwise the universe would be inconsistent and wouldn't exist.

    If you're compelled they you aren't an agent. The Apology argued against omniscient agents.

    It's late, I'm sleepy, I'm probably not explaining my confusion as well as I could, but I look forward to your attempt to elucidate me. In this universe I really now must go to bed. A shame I can't go to bed at 10:30 pm and still have this experience when I wake up.


    From: Anonymous C
    Date: Mon, 13 Dec 1999 00:04:55 -0600

    Grin...well, before I sent the last comment to you, I sent it to a friend of mine who minored in philosophy in college to see if it made sense, since I am FAR from a philosopher. He said it did, but that there was no way I could win an argument with you (by the way I said I wasn't trying to prove anything to you) because you were "...so intent on winning an argument (you) wouldn't see the sense in it!" his words, not mine.

    Perhaps you can ask him what the technical term is for the fallacy of Argument by Questioning the Other Guy's Motives.

    You addressed my reasoning as to all sides of the house being blue, but you failed to address the "facade" issue....and, I was not sure I was using philosophy or not, not being a philosopher and ignorant of constructs for a philosophical argument!

    Well, I didn't address the facade issue directly, but that analogy fails for the same reason the color one does. A better analogy would be this:

    We see one side of a house. I say, "That is just a facade, because the movement of the shrubbery shows that there is no house behind it. You say, "There is a house behind it. I know this by blind faith." I say, "No, if there were a house there the downwind shrubbery would not be moving." You say, "You may philosophically 'disprove' that the house exists, but to my way of thinking, philosophy is a useless discipline other than for exercising the mind. Time is better spent listening to the grass grow."

    As to what part of your argument depends on a specific point of reference, I should think that is obvious....your specific "point of reference", for lack of a better phrase, is the inadequate human reasoning and scientific knowledge which is currently available to us......

    You keep saying that my reasoning is inadequate, but you have yet to point out a single inadequacy.

    You say "love can be defined in many ways" but it " isn't caused by a god shooting arrows." Cupid shooting arrows has nothing to to the the "definition and proof" of the existence of love. You brought Cupid into the argument. You said love was easily defined and proven....I said it that was NOT the case, to which you introduced Cupid and in your last comments said, "The important thing, though, is that, whatever definition you use, it is not caused by a god shooting arrows. " Au contrare....the point I was making is that I know love exists even though we can't adequately test and define it.....just as I believe God exists though we can't adequately test and define Him/Her. Cause and effect are NOT the issue in this particular case. If I understand my philosophical fallacies correctly, you are now using the fallacy of: Straw Man- This is when you use a non-sequitur (You can't know the answer to that math question because you have a white dog). (Er, got that one out of a book!)

    I guess I wasn't very clear on the point I was trying to make with the Cupid statement, so let me try again. Even though you say you can't define love, I would bet that you don't believe that love is a blade of grass in my backyard, which means that "love" is not an arbitrary word to you, but has some meaning. So, even if you can't explicitly define it you can assign properties to it, such as "makes someone desire another", and rule out other properties, such as "is lessened when Dennis mows his lawn". "God" is similar. People have different understandings of God, which is why my essay had several different sections, but I bet that nobody believes that God is a blade of grass in my backyard. People are able to communicate with each other using the words "love" and "God" because, even if their understandings of those words are not identical, they still agree enough on their meanings that someone can confidently mention one without worrying that the other person will think she's talking about a blade of grass. This means that someone can make a statements like, "love is not caused by golden arrows", or "God does not exist", and mean by "love" or "God" "any of the many similar but related concepts which people call love/God".

    You still fail to see the difference between "Living as a Christian because of the fear of death or lack thereof" being the cause and "The benefits of Christianity in the here and now" being the driving force? Come on, now, surely you have better understanding than that! In the latter case, death enters not into the equation.

    You specifically mentioned, "I don't need to worry about death," as one of the benefits.

    In either case, I would say both sides of this disagreement suffer from the philosophical fallacy known as : Argumentum Ad Ignorantiam- This is when you say that something is true simply because someone cannot prove otherwise. (Er, I got this from a book, too! LOL)

    I'll certainly concede that your side suffers from that fallacy. :-) However, as you have often pointed out, I have attempted to prove my point, not merely refute my opponents' proof.

    As far as your statement that if I think something complex must be designed, therefore GOD must have been designed....God is ONE being....the complex world which surrounds us represents millions of complexities.

    Besides which....although human reasoning at this primitive level may require (to quote you) ".... If you are sane you believe at least one of the following statements: It is possible for something to be complex without being designed. God is not complex. God was designed. To claim that you believe none of these statements is, as you would say, ludicrous."

    (Feel free to count me among the insane - by the way, please define "sane" and "insane"!)

    One sign of insanity, at least, is simultaneously holding contradictory beliefs. However, your statement above shows that you do believe "It is possible for something to be complex without being designed." Somehow, apparently, you believe that a collection of many complex things must have been designed but a single complex thing does not have to be.

    My response is........my next question is, "WHY?" The conclusion drawn is based on human reasoning....which, in my point of view, is limited by its HUMAN experience...(see my previous response using the e.e."Doc" Smith series.)

    Yes, and you keep saying that human reasoning is inadequate, without ever saying why.

    By the way, I think it is delightful that you say I am using philosophical arguments....I don't know ANYTHING about them......besides which, from my position this is no argument. (I reiterate that I am stating my beliefs and NOT trying to convert you.) lol...however, if I am using philosophical methods, it is only because that seems to be the only way you will lend credence to anything which we non-philosophers have to say! And quite difficult for me, since I know NOTHING about philosophy...and I do mean NOTHING.

    I am enjoying this little dialogue quite a bit. It has sparked an interest in philosophy which I never before have had. I went out today and bought some Kierkegaard. I was brought up on Ayn Rand, but didn't realize she was a philosopher.

    I have never read either author.


    From: Anonymous C
    Date: Mon, 20 Dec 1999 01:14:03 -0600

    Well, this discussion has turned into quite a lengthy thing, has it not?

    Yes, and that's a good thing.

    AH! I see where you came up with the misunderstanding that [benefits of the here and now include 'I don't have to worry about death.' ] Sorry for the misunderstanding...I was NOT listing that as a benefit of the here and now, I should have said, "death is not an issue." I did not make myself clear on that one, did I?

    It was only a side issue to the main discussion, anyway.

    You ask why I keep saying your reasoning is flawed, without pointing it out....well...let me put it this way......Are you saying that ALL of man's knowledge and reason to date is ALL THERE IS?

    Of course not. But, "not being all there is" is different from "not being anything". Even though scientific research doesn't know everything there is to know about biology I am still going to accept the scientific principle that extreme heat damages tissue and refrain from putting my hand in the fire. Even though the truth of Goldbach's Conjecture is not known I am still going to use addition and subtraction when making entries in my checkbook.

    That there is nothing else to be learned? That reasoning cannot change with scientific evidence? If that is what you are saying then all learning stops here. I, however, maintain that there is a whole body of knowledge, research, reasoning, things to be learned, etc....and that we have only begun to scratch the surface. Indeed,in my woefully indadequate study of science, and in my discussion with scientists and mathematicians, I have found that even in science and mathematics some things have to be taken on faith. That's right. Faith. ......Assumptions which cannot be proven or even defined adequately.

    Science accepts as hypotheses theories which are subject to revision. These can be included in a broad definition of the word faith, but they do not fit the usage of that word in a religious context. Scientific faith is based on the observation of reality. Religious faith is based on the principle that wanting something to be true makes it true.

    Mathematics is a series of tautologies of the form "if these axioms are true then this theorem is true." Those axioms are, of course, assumptions (although they're certainly adequately defined), but the conditional as a whole is not based on faith by any meaning of the word.

    As to the facade argument...really, I expected better of you. What I said was that each of us can see the movement on one side of the front of the "house" with no corresponding movement on the other side. You give one explanation for this phenomenon; I give another.

    This implies that we both agree that the evidence for each of our positions is equivalent, which is why it was not a good analogy.

    We are, in fact, looking at the SAME thing.....you totally changed the example! In your example, you say, "Well, I didn't address the facade issue directly, but that analogy fails for the same reason the color one does. A better analogy would be this: We see one side of a house. I say, "That is just a facade, because the movement of the shrubbery shows that there is no house behind it. You say, "There is a house behind it. I know this by blind faith." I say, "No, if there were a house there the downwind shrubbery would not be moving." You say, "You may philosophically 'disprove' that the house exists, but to my way of thinking, philosophy is a useless discipline other than for exercising the mind. Time is better spent listening to the grass grow." When, in fact, we are both looking at EXACTLY THE SAME RESULTS! It certainly is NOT a better analogy. It is WRONG, WRONG, WRONG!!!!

    We live in the same universe, so we are looking at the exact same results.

    You are implying that I am saying something as ridiculous as the sun is in the west in the morning whereas you say it is in the east. BALDERDASH! (Of course, the fact that it rises in the east means we must accept the universal definition of 'east'. If that were to change, then the direction where the sun might be found would, of course, change!) So you do not see that your side of this discussion has a fallacy. Hmmmm.

    Being just an analogy, it is exaggerated. Your claims are not as ridiculous as saying the rising sun is in the west, but I believe they are just as false.

    However, this is all missing the main point, which is that instead of disputing my evidence, you refuse to allow it into consideration. To extend the analogy, instead of saying, "That shrubbery is not moving nearly as much as the shrubbery on the other side, which supports my position", you are summarily disallowing shrubbery movement to even be considered.

    And you keep "throwing back to me" my words to you about philosophy. I'm glad to see you are taking them so much to heart. You might very well learn something. :-) ROFLMAO Seriously, whether or not you can philosophically prove or disprove something does not make it true.

    No, but whether something is really true affects whether or not you can philosophically prove it. (BTW, I am not a philosopher; I am a mathematician. The only Philosophy courses I took in college were logic courses which could just as well have been in the Math department.)

    Also, one can learn a lot by listening to the grass grow.

    So you were complimenting me when you compared my arguments to listening to the grass grow. :-)

    As to believing God is a blade of grass in your backyard, I don't believe it, but trust me, there are MANY out there who do believe exactly that. They believe that God is present in ALL things, including blades of grass.

    That is not the same thing as believing that God is a blade of grass. I don't see them falling into existential despair when I mow my lawn.

    About your "love" discussion in this last response......really, you sidestepped the issue. But I will let it slide, as you did admit that the concept of "God" has meaning as does the concept of "love".....though you did not adequately define love, we will both concede that it exists.

    Whether God is or is not complex is a moot point. He/She may or may not be complex. Who is to say? You cannot possibly say that if God exists He/She is complex, because you deny His/Her existence. Making assumptions about something which for you is non-existent is interesting, to say the least.

    Not really. I can say that a pink unicorn would have four legs without believing that pink unicorns exist.

    I suppose, philosophically speaking, it would stand to reason that complexity could only be created from something more complex. But then that sort of blows your evolutionary theory, doesn't it? :-) Why is it that complexity can evolve from simplicity, but when it comes to being created, complexity would HAVE to be attributable to a complex Creator? Hmmmm, is it just me, or is there a contradiction here? (Sorry, couldn't resist!)

    I don't know why you're apologizing when you're attacking your own position. I'm not the one who believes that complexity implies conscious design. If you believe that complexity can only be created from something more complex, then you must believe that God (or whatever created the universe) is more complex than the universe, and also must have been created from something even more complex.

    As to your definition of insanity, I sort of got you on that one. Sorry. Neither Webster's nor any of my psychology books give a definition such as you describe. But I will let that one slide, too.

    Webster's New World Dictionary's definition of insane includes "senseless", which certainly includes simultaneously holding contradictory beliefs.

    You should read Ayn Rand. Interesting. But not as interesting as Kierkegaard is (so far).


    From: Anonymous C
    Date: Mon, 03 Jan 2000 01:12:33 -0600

    (Underlined text is previous comments of mine which Anon. C quoted.)

    It was only a side issue to the main discussion, anyway.

    Then why did you address it at all?

    Why not? There's nothing wrong with exploring side issues. This one started because a comment which you made about another person's commentary on the Apology seemed to contradict another comment you'd made, and it took several exchanges to establish that it didn't.

    Science accepts as hypotheses theories which are subject to revision. These can be included in a broad definition of the word faith, but they do not fit the usage of that word in a religious context. Scientific faith is based on the observation of reality. Religious faith is based on the principle that wanting something to be true makes it true.

    Your bias is showing, hon...you say that ."Religious faith is based on the principle that wanting something to be true makes it true." Faith is Faith.

    Not at all. Religious faith is not based on retaining only those premises which survive the test of falsifyable [sic] prediction.

    We are both looking at the same results. My position is that the more that science uncovers, the more we will know God.....you say that regardless of how much science uncovers, there is no God to be known.

    However, the history of scientific inquiry has shown me to be right and you to be wrong. There have been many times when religion has said, "God causes this", and after looking into it it turned out that religion was wrong, but not a single case where religion has said, "God causes this", and after looking into it it turned out that religion was right.

    Mathematics is a series of tautologies of the form "if these axioms are true then this theorem is true." Those axioms are, of course, assumptions (although they're certainly adequately defined), but the conditional as a whole is not based on faith by any meaning of the word.

    Give it up, Dennis. You have given no real evidence which disproves God, any more than I have given any which proves God. Arguments? Yes, you have given wonderful, logical arguments. But evidence? Nope...No evidence yet.

    The Apology contained several different arguments of different types, and I may have not been as clear as I could have been in distinguishing between them in my replies to you. My main argument, The Contradiction of Omnipotent Agency, which applies to any god who is omniscient, is deductive, and therefore much stronger than any argument which relies on evidence. Equivalently you could say that the empty set is sufficient evidence for its conclusion. It is the same sort of argument as "no six foot tall man is shorter than five feet tall". Do you think that argument is invalid because I haven't measured every man in the world? Of course, my argument for atheism is more complex and therefore more likely to contain faults, but it [sic] is [sic] has such faults you have yet to identify them.

    My arguments against lesser gods hinge mostly on Occam's Razor, the priniciple that no evidence for is not equivalent to no evidence against. If someone told me not to go in my house because there's a man-eating lion there and I asked, "How do you know this?" and he answered, "I have no evidence that he's there, but I can't prove that he isn't!" I wouldn't put any credence in his claims. For the same reason I don't put credence in your claims about God.

    This implies that we both agree that the evidence for each of our positions is equivalent, which is why it was not a good analogy.

    You keep saying you have "evidence" ....where is it? I have yet to see any. Face it, you can no more disprove God than I can prove Him.

    Face it, I have disproven most Gods, including the Christian one, and shown that belief in others is at best wishful thinking. The former needs no evidence, no more than the statement "no six foot tall man is shorter than five feet tall" does. My evidence for the latter is the same sort of evidence as my evidence that there is no man-eating lion in my house, only stronger, because there is evidence of man-eating lions somewhere in the universe.

    We live in the same universe, so we are looking at the exact same results.

    AHHH, now we are getting somewhere!

    Being just an analogy, it is exaggerated. Your claims are not as ridiculous as saying the rising sun is in the west, but I believe they are just as false.

    Believing them to be false does not make them so.

    So you're arguing against religious faith now? :-)

    However, this is all missing the main point, which is that instead of disputing my evidence, you refuse to allow it into consideration. To extend the analogy, instead of saying, "That shrubbery is not moving nearly as much as the shrubbery on the other side, which supports my position", you are summarily disallowing shrubbery movement to even be considered.

    Not so. You have no more evidence for your position than I have for mine. Again, we are both looking at the same shubbery. You are using logical arguments....which in and of themselves do NOT constitute evidence. You mistake argument for evidence.

    Well, without argument evidence isn't worth much, is it? Something has to relate the evidence to the claim that it's evidence of. That something is an argument. The believability of the claim depends both on the reliability of the evidence and the validity of the argument. The most believable claims are those which are based on tautological arguments and therefore don't even need evidence, such as the tall man argument I gave above.

    So you were complimenting me when you compared my arguments to listening to the grass grow. :-)

    If you recall, what I originally said about your arguments is that "..Time is better spent listening to the grass grow." I was not comparing them. I was giving you the better alternative of the two! :-)

    Not really. I can say that a pink unicorn would have four legs without believing that pink unicorns exist.

    Good point. So you can.

    I don't know why you're apologizing when you're attacking your own position. I'm not the one who believes that complexity implies conscious design. If you believe that complexity can only be created from something more complex, then you must believe that God (or whatever created the universe) is more complex than the universe, and also must have been created from something even more complex.

    I wasn't apologizing! I was making a joke and apologizing for making the joke. And I was not attacking my own position...I was restating what I believed to be YOUR assumption and then telling you that said assumption was in direct conflict with the theory of evolution.

    I'm not sure why you thought that was my assumption. See The Argument from Design.

    WHY must I believe God is more complex than the creation? God may, in fact, be so simple we cannot fathom Him/Her.

    This sentence makes no sense to me. I may be that we have different conceptions of simplicity and complexity.

    Webster's New World Dictionary's definition of insane includes "senseless", which certainly includes simultaneously holding contradictory beliefs.

    IT DOES? Hummm. Funny, I see people acting on contradictory beliefs all the time. Like an atheist friend of mine who was scared out of her mind by the movie "The Exorcist" when it came out years ago. That she should have been scared by it was a mystery to me. If she was atheist, then how could she believe in Satan, an evil power? She couldn't understand why I, as a Christian, found it silly. As a Christian, my belief system is such that Satan has no hold over me, so I found the movie quite silly. But I wouldn't call her INSANE! Teenagers know that death exists, but behave as though it can't happen to them....I find this contradictory in the extreme...insane? No, just immature. Do you need any more examples?

    Of course I can't answer for your friend, but if she were given a true/false questionaire [sic] and honestly answered "true" to both "Satan exists." and "Satan does not exist." then yes, I would say she was insane. As for teenagers, those who really believe that they can't die while simultaneously really believing that they can die are, in fact, considered insane.

    Also, thanks for letting me know you are a mathematician. I am better able to understand your inability to grasp the concept of "faith". You can say you know what it means, but it is like eating an apple. If I ate an apple, and asked you if it were tart or sweet, you would not be able to answer without tasting that particular apple. The same is true of faith. Faith to you is a hypothetical something...to me it is real.

    I understand faith fine. I have no doubt that it's real. What we disagree on is its cause.

    At any rate, I think it is time to draw this discussion to a close. I am soon going to have to go back into the office (I've been working out of my home) and do not have the time to devote to it. Especially since it seems to be cyclical discussion...you insisting that you have EVIDENCE which disproves God, but failing to provide any evidence at all--merely argument! On the contrary, I say that I can't prove God exists. Never pretended to. Don't need to. God is God and He is perfectly capable of making Himself known should He decide to do so.

    It has, however, been interesting.

    That it has.

    Best wishes to you!

    You too.


    From: Etienne O'Hanlon
    Date: Fri, 14 Jan 2000 04:32:09 EST
    Date: Mon, 24 Jan 2000 15:05:35 EST
    Date: Tue, 25 Jan 2000 20:16:04 EST
    Date: Thu, 27 Jan 2000 00:35:24 EST
    (This is a compilation of several emails.)

    I am responding to your 'Atheist Apology' essay. Although your logic appears valid, I find some of your ontological premises somewhat questionable.

    I understand your point that it makes no difference to the agents, or inhabitants whether their universe is actual or possible in some frame of reference. I also understand that to say, "the universe exist" is merely stating a tautology: all that exists has the property of existing. Also the distinction you make in terms of epistemological knowledge between 'being an agent' and 'being omniscient' is acceptable and does present either as being logically exclusive of the other. That is very fine and good but is seems: you are arguing against a rather restrictive conception of God; that it must have both the properties of agency and omniscience. I agree that this can be refuted on the basis of the "many-possible-universes" thesis, where to the inhabitants of each, all alternate universe exist as mere possibilities, but to a God outside the framework of the universe all are "actual." You say that God couldn't meaningfully create a universe. Within the framework of the agent, I agree, the universe can be thought of as a possibility as validly as it can be thought of as a realization. But since God exists outside that framework, the possibility of a universe is identical with its actualization. After all, how could it distinguish between them? Just because we can consider our universe a theoretical possibility (if not our own, then one identical to it, but by Leibniz' law it would be the same one), there's no rule that says it can't be created OUTSIDE the framework of possibility, in the framework of actuality (If there is a rule, please let me know). Therefore, one need not exclude the possibility of a creator.

    But actuality and possibility are just descriptions of the relationship between what is being discussed and who is doing the discussing. Every universe is in the framework of actuality relative to itself and in the framework of possibility relative to another. So a universe's creation, if such a thing exists at all, is actual with respect to that universe and possible with respect to anything outside of that universe, and a creator, at least at the time of creation, must exist outside of the universe.

    By the way, I like the terms "actuality" and "possibility" as descriptions of these concepts.

    Aside from that, I am curious, as a fellow scholar (well, college undergrad right now) what your responses might be to the following questions:

    -what does 'nothingness' mean? (it's got to be more than "just a word")

    "Nothing" is shorthand for a negated existential quantifier. In other words, "Nothing is P." means "There does not exist an x such that x is P."

    -what can you say about your own existence that you can be sure of?

    If you use "sure" in a very strict sense you can only be sure of the qualities necessary for thinking, such as existence itself (as Decartes pointed out) and some method for processsing information.

    I've been reflecting on your atheist essay a little more since I last e-mailed you. I must say it's an original line of thought I hadn't before considered, and a neat philosophical argument. Interestingly, while I'm not sure of its validity as a proof of the nonexistence of God, your premises do seem to support Hugh Everet's 'multi-worlds' hypothesis, one possible implication of quantum theory. any thoughts on this?

    What I have heard of that hyposthesis [sic] does seem to agree with my theories, but I really don't know enough about it to claim that it does.

    I am fascinated by some of the epistemological questions of metaphysics. I am also sort of an agnost [sic] (originally a Catholic, like you) which is why I am skeptical about arguments both for and against God's existence.

    As far as I know, there isn't that much to know about the 'many worlds' theory However, what you have said in your essay seems to be consistent with it; to us, the inhabitants of any universe, every other universe is an unrealized quantum / wave-function-eigenvector possibility (that I think is generally true concerning theories of indeterminacy, which are controversial in their own right) Outside our framework of actuality, which I think you were pointing out, their [sic] isn't any distinction between actuality and possibility. Therefore, our universe isn't any more 'real' than any other quantum universe. I still see the possibility for God's existence, though I can't tell you 'what kind of God', as long as it is, in some sense, a thinking entity. Perhaps one that sees all the possible universes at once, without an individual perspective, but I'm pretty sure we can't even qualify that assertion.

    As I said in the Apology, for some definitions of God somewhat removed from the popular ones I am a weak atheist instead of a strong one, i.e. I think God is simply highly unlikely rather than impossible. This may be one of those definitions.


    From: Noelle Morris
    Date: Sun, 16 Jan 2000 11:51:22 -0800 (PST)

    Hey, my name is Noelle Morris. I am 14 years old, and I am an atheist. I would like to thank you for giving me further backing of my belief. Although I didn't really understand the first part of your argument (The Contradiction of Omnipotent Agency), I understood and agreed with all the rest of what you said.

    You're welcome.

    I would also like to add a theory of the universe my dad told me about. In short, he said that the universe is fluctuating in size constantly, and has infinitely exploded (as in the Big Bang theory) before the universe we know of today was created, and will infinitely go on like that. So, the universe expands, compresses, and expands for infinity. Well, I better go. Thanks again.

    As I wrote in the section Causeless Effects, "While an eternally ancient universe has been proffered as a counterargument, ... eternal antiquity has its own problems." This type of cyclic Big Bang / Big Crunch universe seems to avoid those problems, although I believe the most recent evidence is against it.


    From: Anonymous D
    Date: Tue, 25 Apr 2000 16:58:13 -0500

    I read some of your apology to the public as to why you gave up a faith as beautiful and rich as the Catholic faith. From what little I read in your apology it makes no sense at all. Just because you cannot imagine a God creating an earth for us then you decide that there is not God. That is pretty shallow.

    Well, it would be if that were actually the case.

    I think that you came to a point in your life where you were trying to find answers to some of your deepest questions and felt that God was not listening to you and at your weakest point because maybe He did not answer you as fast as you would like or as you would like Him to, you just decided it was easier to disbelieve.

    You need to get your psychic powers recharged, or else stop questioning the motives of people you've never even met.

    Aside from your scientific facts, can you prove there is not a God?

    Yes.

    Can you prove anything or are all created things just practical theories and ideologies? What difference does it make how the world was created? So what! We know that the world was created! Big deal how it was, that is irrelevent.

    Assuming what you're trying to prove is not very persuasive.

    It's really sad that people such as scientists and others like you have to have proof of the existence of anything and you try to explain away anything you can't understand or simply you just don't believe in it. You believe with the eyes of faith that you are loved but can you prove it? Or is it just an illusion that you have been believing all your life? They are just words right? How do you really know?

    I'm not sure what you're getting at. If you're asking how I know that other people, such as my wife, love me, the answer is by their actions. I certainly don't believe so because it has been revealed to me in an ancient book.

    Albert Einstein a great scienties [sic] once said that he would rather be a fool in the eyes of man than in the eyes of God. Which do you choose? Suppose there is a God and upon our death we are standing in front of our creator, are you going to tell him that you don't believe because you had all these scientific facts and theories to prove how the earth was made and you just could not comprehend that any power outside of man's measely human condition and thought could create such things.

    No. I think I'd tell him what I wrote in the essay and when he points out my error ask why none of his followers on Earth were able to find the flaw.

    You made a remark that God will judge us well, sorry to disillusion you but God will not judge you, you will judge yourself by the life that you have lead and the decisions you have made are what will determine your/my salvation, not God. That's what is called free will.

    When did I make that remark?

    I don't believe for one moment that you are an athiest.

    It is possible for people to disagree and have a respectful conversation about it, as I have done with other theists on this page. You have no reason for calling be a liar, and I resent that you do.

    What I do believe in all that mumbo jumbo is that you were searching and somehow through that searching, you just decided to give up. I do believe that you will come back around because even in all your science and theories, you will come back to the realization that there is truly a God.

    You are incredibly arrogant.

    How else would all those little particle in space which came together and created that famous "Big Bang" happen. What caused it? When you find the answer to that question, ask yourself what caused that and continue asking yourself the same question until you go back to the very beginning.

    If you had paid attention when you read my essay, you would have realized that the lack of a cause for the Big Bang was my whole point.

    I will pary [sic] for you. Oh yes, science has proven that prayer really does work.

    So do placebos.

    Ask any doctor who has seen the lame walk because of the power of prayer. Search out those things that are true and beneficial to your soul, not your brain, it dies, but your soul never does.

    Well, that's true at least. Something which doesn't exist can't die.

    Take the blinders off! Quit trying to prove man to be more intelligent than God because that will never happen. God made us in his image and likeness and man has never ceased to return the favor.

    I have taken the blinders off. That's why I'm no longer a Catholic.


    From: Anonymous E
    Date: Tue, 06 Jun 2000 18:59:47 -0400

    heh.. nice work (and better done than mine!)

    Thanks.

    take a read if you've the time: http://www.ichthus.com/ [Note added 28 July 2002: This URL no longer points to his site.]

    (Note: His site is a tirade (as he puts it) against Christianity, with a commentary section similar to what you're reading now.)


    From: Muke Tever
    Date: Sun, 11 Jun 2000 22:08:08 -0400

    (Note: I've reformatted his email slightly. I've underlined the text where he quotes the Apology, and I've moved his footnotes up after the paragraph they refer to.)

    I thought "St Anselm's Conception" looked like it needed examining, so I reduced it...

    St Anselm:
    1. If X, then nothing can be imagined to be greater than X.
    2. Assume there is no X.
    3. I can imagine an X (X') with something greater than X, [namely, that it be real.]

    Your notation is somewhat confusing here, since you are using "X" both for a set of properties and an object with that set of properties. On the other hand, this confusion in inherent in Anselm's argument, so in that sense it's a good paraphrase.

    4. This is a contradiction, because X' is greater than X and by (1) there can be nothing greater than X.
    5. Therefore, by (2), continue assuming there is no X.

    I don't understand where your 5 comes from. If 2 did actually lead to a contradiction, then the conclusion would be the negation of 2.

    However, if (2) says "Assume there is an X", then (3) is impossible because of (1), in which case X is still assumed to exist.

    In other words, if you assume there is a God you can prove there is a God. Well, sure. That's trivial.

    In other words, "St Anselm" can't prove anything; it can't say X must or must not exist in the real world outside the assumption in (2).

    The problem with Anselm's argument is that the last X in part 3 has no referent, so the phrase "greater than X" doesn't even make sense.

    (That fixes the difficulty I had with the Santa Claus argument, in that in the Moore poem Santa does "exist" and that in this world he doesn't.)

    Well, the Santa Claus argument does have the same difficulty as St. Anselm's argument, whatever that difficulty may be.

    BTW, your "Contradiction of Omnipotent Agency" seems to do the same: Theoretically possible universes don't of necessity have any real existence outside the mind of the theorizer. For example, Tolkien's Middle-Earth: One can imagine that Tolkien had a sort of "subspace link" with an alternate universe, and that anyone with a working imagination has one too, or one can imagine Tolkien made it up out of his own head. (You cover the answer to this quite well in "Shaving Superman with Occam's Razor".)

    Not only did I never say that there is a subspace link between alternate universes, but what I did say about the relativity of reality should have made it clear that I don't believe in any such thing. I think that Tolkien made it up out of his own head. I also recognize that there is no contradiction between the statements, "Tolkien made it up out of his own head." and "From Frodo Baggins' point of view, the Middle Earth is real.", assuming that "Frodo Baggins' point of view" has some meaning, i.e. that the Middle Earth is a subset of some consistent universe, the truth of which is beyond my powers to determine.

    About omniscience and 'what-would-have-happened' universes: Most theologies I'm aware of don't put gods in charge of what-would-have-happened worlds.[1]

    Being in charge is not the question. Having complete knowledge is, and many theologies, including the most popular ones, claim omniscience for God. Saying that God wouldn't know the answer to a question of the form, "if this were true would that also be true?" is placing a limit on God's knowledge which most religions would not accept.

    To do so, as you rightly point out, would disable them from carrying out anything so coherent as a "plan".
    [1] In a sense, one could put questions like "what would have happened if..." in a similar relation to omniscience that making unliftable rocks has to omnipotence.[2]
    [2] Either omniscience knows what could have happened _and_ distinguishes that from what has happened (I suppose this could be "added" omniscience) or it knows what _does_ happen, and just surmises about what could, but doesn't happen (similar to, but vastly more well-informed than people do-- this would be "basic" omniscience). From what I can make out you're arguing against the "added" omniscience that most godviews can do without.

    What do you mean by "has happened" and "could have happened"? If A happened and B could have happened relative to some universe W, then there's a universe W' where B happened and A could have happened. You (and several other respondents to my essay) seem to be making a circular argument. You assume that the universe which you are a part of has a privileged status among all possible universes, and then you proceed to prove that the universe which you are a part of has a privileged status among all possible universes.

    In addition, being an agent implies a privileged temporal reference, a present dividing past and future. This, however, is incompatible with omniscience, which includes an equally perfect understanding of every point in time and therefore no distinction between past and future.

    Not necessarily. Omniscience is perfectly capable of knowing mundane facts like "this is the present", "this has not yet happened", and "this has already happened". You're thinking of omnipresence (in the temporal sense), which is often attributed to monotheist gods.

    What do you mean by "this" in the sentence "this is the present"? Do you mean "the present time"? Then you're saying "The present is the present," which is again a circular argument. The present is the point dividing the past from the future. The only thing which distinguishes the past from the future for creatures such as ourselves is the different amount and type of knowledge we have of different points in time.

    Anyway, that's all I have to say about that. Interesting work.

    Thank you.


    From: Muke Tever
    Date: Mon, 19 Jun 2000 02:57:37 -0400

    (Note: For this section, underlined italics is where he quotes me, and underlined normal font is where he quotes me quoting himself. I've trimmed the former a bit to save space.)

    St Anselm:
    1. If X, then nothing can be imagined to be greater than X.
    2. Assume there is no X.
    3. I can imagine an X (X') with something greater than X, [namely, that it be real.]

    Your notation is somewhat confusing here, since you are using "X" both for a set of properties and an object with that set of properties.

    Well, no, I am only using X to represent an object with a set of properties. (Try it--replace "X" with "object X".)

    Then you're no longer representing St. Anselm's argument, because his step 1 is a definition of a property.

    4. This is a contradiction, because X' is greater than X and by (1) there can be nothing greater than X.
    5. Therefore, by (2), continue assuming there is no X.

    I don't understand where your 5 comes from. If 2 did actually lead to a contradiction, then the conclusion would be the negation of 2.

    I was trying to show the structure of the original argument, which I suppose led me to be unclear.

    4 is not actually a "contradiction", though St Anselm says it is.

    This is because the first two statements, the definition (If X, then ...) and the assumption (Not X). If there is no X, as assumed, then the definition is of an fantasy object, and one can imagine fantasy objects to be real all day without making them so.

    Except that your (4) does not really reproduce Anselm's (4). (I should have pointed this out last time.) His contradiction is not based on something being greater than God, but on the ability to conceive of something greater than God.

    However, if (2) says "Assume there is an X", then (3) is impossible because of (1), in which case X is still assumed to exist.

    In other words, if you assume there is a God you can prove there is a God. Well, sure. That's trivial.

    I mean to say that using St Anselm, you can assume anything you like in (2) and "prove" it's so.

    In other words, "St Anselm" can't prove anything; it can't say X must or must not exist in the real world outside the assumption in (2).

    The problem with Anselm's argument is that the last X in part 3 has no referent, so the phrase "greater than X" doesn't even make sense.

    Its referent is the first X in (1), which has been assumed not to exist.

    Except that in St. Anselm's argument there is no object in (1) to refer to; it's just a definition of a property.

    "Greater than X" makes sense--try Anselm on a real object:
    1. If there is a New York City, I can't imagine anything better than it.

    Are you saying that the inability to imagine anything better than NYC is part of the definition of New York City? Because if you're not, then you're not reproducing St. Anselm's argument.

    2. Assume there is no New York City.
    Since there is no NYC,
    3. I can imagine an NYC that is better than New York City. (That "something better" is that it is a real version of the NYC described in 1.)

    Now, stopping right here, I can see that being able to imagine a better NYC is _disproof_ of NYC because in (1) I say that if there was a NYC I couldn't imagine a better one.

    Is that right? It all comes down to saying "if p then not q"; "not p" therefore "q".

    This is an invalid argument. "If p then not q"; "q" therefore "not p" is a valid argument, and in fact represents St. Anselm's argument, with "God does not exist" in the role of "p" and "I can't imagine anything greater than God" in the role of "q". The problem is that he doesn't actually prove "if p then not q".

    [Logically the last statement could be either "q" or "not q"--there's not enough information; I could easily still not imagine it--but it doesn't constitute any disproof of either the first statement or the second one.]

    It's true that "if p then not q" and "not p" together doesn't tell you anything about q, but now you're really straying from the original argument.

    One of the unexpected consequences of this commentary page is that, even though I put St. Anselm's argument in the Apology in order to refute it, I've ended up several times defending it from inaccurate attacks.

    I think that Tolkien made it up out of his own head. I also recognize that there is no contradiction between the statements, "Tolkien made it up out of his own head." and "From Frodo Baggins' point of view, the Middle Earth is real.", assuming that "Frodo Baggins' point of view" has some meaning, i.e. that the Middle Earth is a subset of some consistent universe, the truth of which is beyond my powers to determine.

    I suppose this is where we differ.

    From Frodo's point of view, he has free will. However, Frodo does not have free will, being a creature of a planned universe--Tolkien defined the events of his life and his thoughts and his reactions. These statements are contradictory; therefore I don't think "Frodo Baggins' point of view" has any meaning. It is a fantasy proposition.

    Well, the issue of free will is a can of worms you can open if you want, but if so you're going to have to decide what you mean by it. For instance, in the first sentence of the above paragraph you treat it as a relative property, and in the second sentence you treat it as an absolute property. In any case, you seem to be supposing the same sort of "subspace link" that you argued against in your previous email. You're saying that Tolkien's invention somehow affected another universe just because it happened to match it in some respects. Say there's another universe where some creature invents a fantasy being which he just happens to name "Muke Tever", and he writes "Muke Tever tied his shoes." You seem to be saying that the possible existence of that universe somehow robs you of free will. I disagree. Just because someone conceives of a subset of a consistent universe does not give him control over that universe. He only has control over which of the supersets of his description will still be supersets of any further description. Assuming of course, that his conception is consistent in the first place, which, as I said before, I'm not willing to assert for the Middle Earth.

    About omniscience and 'what-would-have-happened' universes: Most theologies I'm aware of don't put gods in charge of what-would-have-happened worlds.[1]

    Being in charge is not the question. Having complete knowledge is, and many theologies, including the most popular ones, claim omniscience for God. Saying that God wouldn't know the answer to a question of the form, "if this were true would that also be true?" is placing a limit on God's knowledge which most religions would not accept.

    Well, I don't see how it is a limit, and I don't think that religions would have a problem with it. Even CS Lewis can have Aslan say nobody knows "what would have happened".

    You don't see how not knowing something is a limit on knowledge?

    To put it another way: Tolkien created Middle-Earth and can be said to be omniscient regarding it. (The authorial make-up-something-and-it's-true "retrograde" omniscience)

    If by "Middle Earth" you merely mean Tolkien's descriptions, then that may well have been true. If by "Middle Earth" you mean a consistent universe which his description fits (which is the definition relevant to the discussion so far), then I strongly disagree. I extremely doubt Tolkien even knew what Frodo had for breakfast 574 days before his 33rd birthday. He certainly didn't know the position of every atom in every molecule in the M.E. on that day.

    Asking him "if this were true would that also be true?" may have an answer or it may not. Tolkien may have a fixed plan about "real" in relation to Middle-Earth, and omniscience doesn't say he has to know, create, or care about an answer to any question outside that plan ("If Bilbo were a newt, would he have still found the Ring?").

    Yes it does. "Omniscient" means "all knowing", not "knowing a lot of stuff."

    [2] Either omniscience knows what could have happened _and_ distinguishes that from what has happened (I suppose this could be "added" omniscience) or it knows what _does_ happen, and just surmises about what could, but doesn't happen (similar to, but vastly more well-informed than people do-- this would be "basic" omniscience). From what I can make out you're arguing against the "added" omniscience that most godviews can do without.

    What do you mean by "has happened" and "could have happened"? If A happened and B could have happened relative to some universe W, then there's a universe W' where B happened and A could have happened.

    Of course there isn't!

    If there is a Universe W:
    A happened.
    B could have happened.

    If there is a Universe W':
    A could have happened.
    B happened.

    What do you mean by "is"? I don't mean to get Bill Clinton on you, but when you're discussing metaphysics that's a legitimate question.

    There is nothing in the existence of Universe W that _requires_ there be a Universe W'. (Except, of course, people in Universe W making hypotheses about it, where Universe W' is only a fantasy proposition.)

    And there's nothing in the existence of Universe W' that requires there be a Universe W. The situation is perfectly symmetric. Neither universe holds a priviledged position except relative to itself.

    You (and several other respondents to my essay) seem to be making a circular argument. You assume that the universe which you are a part of has a privileged status among all possible universes, and then you proceed to prove that the universe which you are a part of has a privileged status among all possible universes.

    If by that you mean I think it's real--that's no special privilege; even Frodo Baggins thinks his universe is real. As you've said yourself, "the fact that a universe exists relative to itself is a tautology."

    But the universe doesn't have a privileged status among all possible universes. Any universe, real relative to itself, is also not real from the point of view of other possible universes (except perhaps an identical one).

    Well, yes. This is what I've been saying all along. This is the point I've been trying to make. This is what you've been arguing against up to now, for instance when you say an omniscient being would distinguish between what "does happen" and what "could, but doesn't happen".

    Not necessarily. Omniscience is perfectly capable of knowing mundane facts like "this is the present", "this has not yet happened", and "this has already happened". You're thinking of omnipresence (in the temporal sense), which is often attributed to monotheist gods.

    What do you mean by "this" in the sentence "this is the present"? Do you mean "the present time"? Then you're saying "The present is the present," which is again a circular argument. The present is the point dividing the past from the future. The only thing which distinguishes the past from the future for creatures such as ourselves is the different amount and type of knowledge we have of different points in time.

    Imagine a "Henry", born omniscient.

    At Henry's birth--a countable beginning--he knows everything before his birth is in the past, and everything after it is the future. When the nurse slaps him, he knows from the sensory input that that moment is the present, everything before it including his birth is in the past, and everything after it is in the future.

    I know when I am feeling something and when I am remembering feeling something. I can tell the difference. Why do you say that Omniscient Henry can't?

    Because the difference you sense is a difference in the amount and kind of knowledge you have of the two events. You are sensing your limitations. An omniscient being can do the same in the sense that it can understand the limitations of Muke Tever's knowledge. It can't sense the limitations of its own knowledge because there are no such limitations.

    Now, assume Henry is eternal, that is, omnipresent in the temporal dimension. _Then_ there is no point saying "the present", "the future", and "the past", because Eternal Henry has no "beginning" and no "end", and his knowledge comes from _being there/then_, not from passively knowing it. E-Henry is "omniscient" regarding what he experiences, but not what he doesn't--he may study Abraham Lincoln, and know everything about what he studied, but if his timeline never meets the man he doesn't necessarily know anything _real_ about him.

    If he doesn't know something then he's not omniscient.

    If E-Henry were spatially omnipresent as well, it wouldn't be a problem, and he would be functionally omniscient just as much as O-Henry, and in the same way you're talking about when you talk about not distinguishing time. But, E-Henry's omniscience is not an inherent quality but rather a side-effect of spatio-temporal omnipresence, which is what I was trying to say.

    If someone's omnipresent (presumably in all possible universes) and has complete knowledge of everywhere he exists he's not necessarily omniscient. Not all knowledge is location based.

    I hope I'm not a raving lunatic here,

    Same here.


    From: Michael Fretheim
    Date: Thu, 22 Jun 2000 22:07:41 -0700 (PDT)

    (Note: He quotes extensively from the The Contradiction of Omnipotent Agency. I have put those quotes in underlined italics, and trimmed them somewhat.)

    OK, Well, from the 4 atheist sites I have inspected so far, yours is the first that tries to defend their atheism with something more than rejection of fairy tales.

    I would like to note that there seems to be many atheists and agnostics that started from a Catholic school. It could very well be because catholicism spends so much time in worship of icons, angels and Mary, that they forget to teach about God, Jesus and the need for salvation.

    I take it you've never actually attended a Catholic school.

    Let us begin tearing this apart.

    If the universe did not exist it would still be a theoretical possibility. If it were just a theoretical possibility there would be a theoretically possible Dennis Himes who would act in relation to this theoretically possible universe identically to how I act in relation to this actual universe. ...

    In fact, he would not know, and there would be no way he could possibly know, that his universe was just theoretically possible and not actual. "Theoretically possible" and "actual" are, therefore, just arbitrary labels when applied to universes.

    I see that you are trying to slip in an assertion without backing it. Your assertion about arbitrary labels is true only when the initial assertion is true, namely that the universe did not exist.

    I was proving that a theoretically possible universe is real to its inhabitants. I was assuming, perhaps incorrectly, that my readers would be intelligent enough to realize without being told that a real universe is theoretically possible to its inhabitants. Do you dispute this? Do you think that a real universe is not theoretically possible?

    However, the universe does exist. To build an assertion. If the universe does not exist, then nothing inside it exists. Thus my computer keyboard does not exist. However, I feel it, smell it, can see it, can hear it, and if I tried, I could taste it. All the senses that I use to test reality tells me that it is there. By using the tools I had been using since birth to determine what is real, I can tell that the keyboard exists. This contradicts my conclusion that nothing inside the universe exists. Thus it is not possible that the universe does not exist.

    If you're going to argue against my assertion that "real" and "theoretically possible" are just arbitrary labels then you should try to find something that's different between them. All you've done here is list things which are completely identical in the two cases.

    To a being in a universe that universe cannot not exist. An omnipotent god could not decide that a certain universe will exist (relative to itself) or not.

    Careful. You do realize the contradiction of your statement. You are claiming that an omnipotent (all-powerful) God cannot do something. God is either omnipotent or He is not. Partial omnipotence is like partial pregnancy, it does not exist.

    This brings up the old question of what the "potent" in "omnipotent" refers to. Specifically, does the "anything" in "can do anything" include logical contradictions? Traditionally this is framed as "If God is omnipotent can he create a rock so heavy he can't lift it", but "If God is omnipotent can he make a universe not exist relative to itself?" works just as well.

    As far as I'm concerned, you can take your pick. If "omnipotent" does not imply logical contradictions, then my statement above stands. If it does then you've just supplied yet another disproof of God's existence.

    If she had decided that our universe would not exist it would not make a bit of difference to us. ...

    I see that you use "she" to disconcert Christians.

    Why would using "she" disconcert Christians? I use "she" because I'm using a language which lacks an epicene third person singular pronoun.

    You may want to correct that, as it makes you look petty, and is only being used to fluster those who would comment. Besides, you are supposed to believe that there is no God. If there is no God, then why do you assert God is female?

    If I'm asserting that God is female then why do I use both "he" and "she" to refer to God in the essay?

    Now consider two universes which are identical up to a certain point in time at which an event occurs in one but not the other. An omniscient being would know in every detail the entire futures of both universes. To the omniscient being each of the two universes would be equally real, and to say that the omniscient being "caused" the event would again be to assign a meaningless label. The omniscient being could not prevent the event from happening relative to the one universe nor cause it to happen relative to the other. The inhabitants of either universe would neither know nor care whether their universe is the one that "really happened" or the one that "could have happened but didn't".

    Again, be careful about what you say an omnipotent God can or cannot do. You say that He cannot prevent the event from happening in either universe.

    Since a consistent universe in which event A happens exists relative to itself tautologically, for a God to prevent it in that universe would be a contradiction, because event A would both occur and not occur in that universe. Now we're back to the "heavy rock" question. Either you don't think that omnipotence includes nonsense, in case my assertion is correct, or you're proving that God cannot exist. Take your pick.

    You offer no proof that He cannot prevent it, you just make a statement, and assume that, because you said it, it must be true.

    Bullfeathers. I proved that a theoretically possible universe exists relative to itself tautologically, and I pointed out that if God changes a universe the universe which is unchanged is still theoretically possible. You may disagree with my proof, but if so you should try to find a flaw in it rather than simply declare it doesn't exist.

    An omniscient being (and therefore an omnipotent one) cannot be an agent. To be an agent is to choose which among several possible universes one will have more complete knowledge of. If I decide to turn left instead of right I will have far greater knowledge of the universe in which I turn left than the one in which I turn right. ... If I were omniscient that would not be the case. ...

    You have only proven that humans are agents in this universe. I have more to say, but will wait until a little later.

    In addition, being an agent implies a privileged temporal reference, a present dividing past and future. This, however, is incompatible with omniscience, which includes an equally perfect understanding of every point in time and therefore no distinction between past and future.

    Given your history, I can make a fair assertion that you have not read much of the Bible, or perhaps only the parts of the Bible that the priests wanted you to see. There is an analogy that was used often to describe God's relation to the things that He has created. God is the potter, and His creation is the clay. He made the universe, but He is not part of the universe, in the same way that God made a clay pot, but He is not part of the clay pot.

    Why is this important? It is important because He created time, and He created the Universe. Though He created both of these, He is subject to neither of them. Yet you repeatedly put Him into subjection to both of these things, implying that He must be part of the things He created.

    Not only is this not true, but it is the exact opposite of the truth. The independence of an omniscient being and the laws of any possible universe is one of the major points of my essay. How can you quote me as saying, "a privileged temporal reference ... is incompatible with omniscience" and then soon after claim "you repeatedly put Him in to subjection to [time]" and expect to taken seriously?

    However, if He is not subjects [sic] to the laws of the Universe, or the laws of Time, then He can indeed be an agent in the multiple universes.

    Your argument is interesting, but lacks foundation. It probably appeals to other atheists, because your conclusion is something that their ears are itching to hear. However, like Darwin's theory, it rests upon unproven assumptions.

    Except that all of the "unproven assumptions" you've tried to identify have turned out to not only not exist, but in some cases be the exact opposite of what you yourself quote me as saying.


    From: Leonard E Hambleton
    Date: Sun, 02 Jul 2000 09:02:45 -0500

    this is the second time i have seen the pink unicorn analogy. not a good choice because if a horse-like creature had horns twisted into one and was also a hairless albino then we would have a 'pink unicorn', but there still is no god.

    I'm not sure where that analogy comes from; I've also heard it several times. Often I hear it as "invisible pink unicorn", if the person using the analogy wants to capture the inherent contradictions of a religion (since in order to be pink, something must be visible). In the Apology, however, I was using it as an analogy for the types of gods which are not, in fact, impossible, but in which there is no rational reason to believe.


    From: Anonymous F
    Date: Mon, 24 Jul 2000 07:54:08 -0700 (PDT)

    Imagine a being that evolved, like us, but who is technologically so far advanced that we cannot comprehend it's [sic] abilities. Imagine this being perfected time and or inter-dimensional travel and has the desire to keep an "ant farm" - enter earth [sic]

    This being could be God to us the same way we can be god to very small insects.

    This being could exist in a theoretical or an actual universe.

    In order to be the God of humans you don't have to be omnipotent you just have to be more potent than the most potent human.

    I dealt with this sort of god in the section Shaving Superman with Occam's Razor.


    From: Anonymous G
    Date: Tue, 03 Oct 2000 13:45:11 -0500

    Some comments on your apology:

    First, I'm not sure what you mean by "weak" atheism, since you describe it as admitting that God is possible in the same way pink unicorns are possible. In what way are pink unicorns possible or impossible that is the same for God?

    In the sense that they are not self-contradictory but so unlikely that belief in them cannot be rationally justified. This is the weak atheists' view. It is not mine, however, at least when applied to the usual definitions of God, because I think that those sorts of God are self-contradictory.

    I think the existence (or potential existence) of pink unicorns is easy to show if one gives a description in terms of observable phenomena (unless you spread it to include heritability, but with genetic manipulations, who knows?). If one changes the definition from the one you like to use, then one may assert their existences? That leaves a fairly huge loophole for theists, I'm afraid. You use the phrase "hedge their definition" as though to indicate it's a weakening, but some theists do include `hedges' in their definition of God precisely because they are trying to use reason as much as you are, and perhaps have some spiritual (or hallucinated) experiences which you lack and upon which some of their reasoning is based.

    A hallucination of a pink unicorn could be legitimate cause for investigating whether the hallucination was a real sighting or not, but it is not a legitimate cause for rejecting the results of that investigation when it indicates that no pink unicorns exist. Similarly, a spiritual experience could be legitimate cause for investigating whether the experience was caused by spirits, but it is not a legitimate cause for rejecting the results of that investigation when it indicates that no spirits exist.

    If a theist reacts to the contradictions of his belief by altering that belief that's all to the good, but a partial alteration which is an attempt to avoid only the most blatant problems will only be partially correct.

    Next, I want to share some comments concerning your section entitled "The Contradiction of Omnipotent Agency." You seem to be hiding a bit here, in that you switch between speaking of Universes existing with respect to themselves (a tautology), existing with respect to each other (I suspect that may be a contradiction), and existing with respect to some outside, `larger' universe which also contains an omniscient and omnipotent being somehow external to these smaller universes (which seems to contradict my notion of a universe).

    Actually, I thought I was doing the opposite. Those distinctions are usually hidden in theological discussions by not being sufficiently dealt with, and I was trying to bring them out of hiding by dealing with them explicitly.

    You're right that an omnipotent being being both external to a universe and interacting with that universe is a contradiction. What my essay shows is that it's a contradiciton which is unavoidable when you posit an omnipotent agent.

    Also, you mention that omniscience and omnipotence imply each other. While I had always agreed with that idea, I have been told by many theists that the fact that I accept their logical equivalence (in the sense that each implies the other) means I don't really understand either term and that I need to read more theology. I have, and I still don't understand what is wrong about the statements, but be aware that that is a bone of contention.

    I never said that omniscience and omnipotence imply each other. I said that omnipotence implies omniscience, but not the converse.

    In your section on Occam's razor, you have explained admirably the use of that philosophical principle to reject any `evidence' for God I have ever seen of heard. However, this is far from a proof of atheism. As you say in your comments, if a phenomenon can be explained using A and B or just A alone, then that phenomenon does not provide evidence for B. However, it also does not provide evidence against B, no matter how many times it happens. If a phenomenon can be explained using A and B or A alone or A and ~B, then it provides evidence for neither B nor ~B. This argument is thus an argument (an apology, if you will) for agnosticism, not atheism.

    You're right that Occam's razor does not in itself completely rule out the possibility of a god, but I disagree with your implication that it leaves us with equal reason to believe either way. Occam's razor does not rule out the possibility that there's a pink unicorn living in my neighborhood and that's why my dogs sometimes bark for no clear reason, but Occam's razor tells me that there's a better chance that they're barking at a cat.

    Since the rest of your apology deals mainly with disproving theist's arguments, I'll not bother you about those. My point is that I don't believe you have made a very strong case for atheism as opposed to agnosticism. I view the existence of God the same way I view the existence of Extra Sensory Perception. I haven't seen evidence for it, and in fact every time someone claims to have it, they turn out to be deluded or a fraud. But I can keep looking, and it doesn't mean I'll never find it. Personally, I have a small hint of hope that there is something like the Judeo-Christian Father-Judge who meets us when we die to review our sins and send us to Heaven or Hell. You see, I want to ask Him about His hypocrisy and cruelty. (If it's true, I hope He lets me.)

    The distinction between an agnostic and a weak atheist is a hazy one. Since you compare God to a situation where "I haven't seen evidence for it, and in fact every time someone claims to have it, they turn out to be deluded or a fraud." I'd say you're at least on the borderline.


    From: Veracity
    Date: Wed, 22 Nov 2000 11:51:34 -0800 (PST)

    Wait.. so I don't get it. How is this proof? So far, no one on this site has convinced me of the truth behind Atheism, nor behind Theism.

    You have this bit for proving an omnipotent being doesn't exist. I read this and it doesn't prove a damn thing, for or against. It apparently is just your view on how you think of the world your questions and theories. How do they prove anything at all?

    It proves that an omnipotent being doesn't exist by demonstrating that such existence is contradictory.

    Weak and strong... sounds like you're full of shit and can't make up your mind.

    If you asked someone, "Do you believe there are aliens living in New York City?" and he said his answer depended on whether you meant "alien" as in "someone from another planet" or "alien" as in "someone from another nation" would you say that he's full of shit and can't make up his mind?

    Kind of like some Catholics. Catholics say for example, that you have to confess your sins to the priest. Why? They then answer with their own explanation of why, and in there they put something like "it is possible that God can forgive your sins...but for serious sins..." what the hell is a serious sin? This is coming from the Catholic document.. that official Catechism of the Catholic Church. Of course, if they believe in and follow the bible, why don't they point people to the bible instead of their own made up document? But.. they just simply fail to answer the question.

    John 20:23

    I'm not sure why you distinguish the Bible from "their own made up document", since the Catholic Church invented the Bible.

    I also read several of the responses...

    Now, in one of your responses to Anonymous C, you were saying something like if you believe in none of these 3 things (and you list them) then it is ludicrous. Under who's rules? Yours? Is humanity supposed to base their sanity by your sanity?

    No, the truth of the statement you're referring to is inherent in the statement itself. It rests upon no authority.

    How do we know that you are not insane? What about those that the world views as insane? How do we know that it is not them who are insane, but us? It just so happens that MORE of us are of the same insanity, which creates the norm. For example, what about Germany in WWII? Over there, it was the norm to believe that they were the supreme race and that Jews and other listed races were to blame for Germany’s troubles. But we saw that as insane. Why? Because we thought it was so, and our neighbors thought it was so, and their neighbors thought it was so, so therefore it was so. Of course I’m not going defend Germany. I too would be included in the batch to say they were insane, namely their leader.

    I disagree. The belief that a collective error becomes correct simply because a lot of people accept it is one of the great disservices that religion has done to humanity over the years. Believing that there is no objective way to judge Nazi philosophy is an extreme example of what is wrong with it.

    And someone said something about "why doesn't God stop the witch burners, etc" because "love your enemies.." but there is one problem with that. In line with the rest of the "word" which would be from the bible since the loving enemies phrase comes from it, it is all over the place that we all have our own FREE WILL. So why should an omnipotent being go against his word and take our free will away, even though it's against him?

    There are other ways to prevent the effects of evil besides removing free will (leaving aside the whole question of what exactly "free will" means). Governments do it all the time.

    In the first response, someone (Catholic?) went out and said how you fit into a typical category, etc and the rest. But of course, if that person was in true faith, it wouldn't be up to that person to judge you in that way, as would be so mentioned in the bible which that person presumably follows. So why is he trying to put you down? Some do that in fear.

    And finally, there was mention of science disproving, and how people say god did this thing, and then they have to find something else god did because science proved that first thing. Well science is most definitely not disproving god. In fact lately it’s been proving it. A few years ago, it was on national news that carbon dating techniques have been proven to be faulty. An elephant tusk of some few years old was submitted to try to find out the validity of carbon dating, and the carbon dating came back as saying it was some thousands of years old. I don’t remember the exact numbers. So how long HAVE we been on the Earth? And amazingly there was a discovery (on Discovery channel) of the actual Noah’s ark. Was found in some glacier, and only during the hottest days can people get to it because it’s so deep in ice. But they have been aboard (with video cameras and scientists) and have measured it out to come out just like the measurements that are apparently in the bible. And scientists who didn’t believe it would float (because the whole giant thing was basically a big box) reproduced a model of it, and came back saying that no matter what, they could not make it sink, nor even end up capsized. It would always return to its normal position after tossing of any magnitude. Then later they try to show that the biblical Adam was on the boat.. don’t know how the hell they come up with that. Now you could say, “Do you believe in everything you see on TV?” or.. “that’s just the Discovery channel.” And of course.. “I never heard of that! You’re full of shit” Well, why believe in anything then? How do you know that what you’re reading or seeing is true? Science learns about space and creates theories and stuff about it, and also about molecules, etc. With the information we get, we have discovered other things. How is any of that disproving or proving of God, or a god? If you believe in God, then God made it that way. If you don’t, then all this stuff is just there for whatever explanation you have.

    Science is more than just a set of theories. It is a method for testing theories. Science is based on reproducible results and falsifyable [sic] predictions. A century or so ago a flood consistent with the Bible was a scientific theory competing with others for acceptance. It has long ago been abandoned due to the overwhelming preponderance of evidence against it.

    It is true that you could propose a god limited enough to avoid the contradictions I pointed out in my essay but powerful enough to be "behind" all of what we know about how the universe works, but then you're just personifying the universe. I dealt with that possibility in the section The Vanity of Humanity.


    From: Anonymous H
    Date: Sun, 24 Dec 2000 03:42:41 -0800 (PST)

    Dennis, a couple of points:

    1.
    In reply to Gail Gunst you said that it was odd for Paul to write what he did in Romans 2:1. But that is exactly the point. He is telling the Roman Jews (in chapter 2) that if the [sic] judge anyone for anything (since they were self-righteous and thought they would go to heaven for being good), they are also guilty of it. And so their self-righteousness is stripped away.

    But he's judging them, so is he saying that he's guilty of what he's judging them to be, such as being "Libelers, slanderers, enemies of God, rude, arrogant and boastful", etc.?

    As he makes clear in Romans chapter 3, "all have sinned and fall short of the glory of God". So he is setting up the argument that everyone is lacking in the moral perfection department, and is in need of a pardon. It makes more sense when taken in this context.

    His accusations were somewhat stronger than "lacking in ... moral perfection".

    2.
    In reply to 'Anonymous D' you wrote that you would stand before God and ask him why none of his followers on Earth pointed out your erroneous logic. This assumes that the flaw he points out will be something you have not heard before.

    Yes, that's true. It's not an arbitrary assumption, though, it follows directly from the fact that my logic has not been shown to be erroneous and the presumption that God is infallible.

    It was a loaded question, anyway. It's as if someone said, "You can't prove that 2+2=4. What if you added 2 and 2 and got 5? What would you say then?"

    Just wanted to point that out, since I believe all of this will indeed happen and I genuinely want to make sure I am not culpable for failing to speak up. I also care about you, but only as much as I can having never even met you :-)

    Thanks. Even though I think you're misguided I appreciate the thought.


    From: db
    Date: Fri, 16 Feb 2001 20:41:13 -0500

    I read most of your article, and thought it was very well written. Most of which concepts are familiar. I would like to present you with my thoughts. Many assumptions are made but can be discerned from context.

    The term God usually implies an "all knowning" [sic] being. My view of god is not as a being as we know it or can concieve [sic]. As quantum theory implies, for the experimenter to know the state or position (hehe if state or position of matter/energy is an appropriate term) of quantum particles said experimenter must change the state of the particle. Therefore becoming part of the experiment. With that short general statement being said, for God to be all knowing (i.e. knowing all states of all matter and related contents of the universe), God must be the universe. The problem there is if you are the universe, how are objective thoughts possible. The last several sentences can be largely picked apart by the words, terms, and concepts used. But the gist is evident. Therefore I see "god" as merely being the Universe no more, no less.

    I dealt with the identification of god with the universe in the section The Vanity of Humanity, but I admit I didn't forsee the following argument.

    One resulting concept that results from the above paragraph:

    Life can be loosely associated to structure which promotes structure by changing seemingly inert matter into structure. This will just be referred to as structure. Chaos will be associated with anything which hinders the growth of structure. If the universe is looked at by its tendencies over time; it seems that structure (life) will spread at an exponential rate (of which scale is unknown). Our planet viewed as a whole over time started off as a system of matter and energy low in structure (please forgive the simplicity of these statements - I only aim to convey the gist of my concepts) Over millions of years structured interactions began to happen, eventually our planet started to emit radiation in a controlled manner (radio signals). If the tendencies of life are observed, over billions of years many areas of the galaxy will interact through energy (communications between lifeforms human to human - maybe human to alien, whatever) Eventually control of energy will lead to different (more advanced?) levels of lifeforms or structure. To cut to the chase... Life will continuously convert energy and matter into life which does the same... Eventually (Billions * Billions of years) matter and energy will become life. At some point the entire universe could be viewed as one living whole. At which a time this whole can then be referred to as the "all knowing" god. All knowing in that it is the universe, and capable of knowledge in that the concept of knowledge can be loosly applied to a living being.

    That's an interesting theory. The main problem, as I see it, is that it directly contradicts the laws of thermodynamics. Structure decreases with time. Things decay, entropy increases, the universe unwinds towards its distant but inevitable heat death. The Earth is a local exception, made possible by a massive infusion of energy from the sun. The sun will not shine forever, though.

    At a stretch I would predict that if the "living universe" becomes a single conciousness (hehe if that is a valid term) - perhaps a singularity would occur at the time of all matter becoming an advanced life structure. At this time of all knowing and becoming "one". Another big bang may occur to promote infinite possibilities and variations.

    These concepts are a stretch I know, but do fit a conceptual framework that lends to the tendencies of our limited observations.

    That would be true only if our observations were limited to Earth.

    Keep up the good work Dennis, I'll get you in Oh hell one of these days.

    (Note: That "Oh hell" refers to the card game, not the afterlife. I know this commentator personally.)


    From: Anonymous I
    Date: Thu, 8 Mar 2001 10:05:21 -0800 (PST)

    (Note: Anonymous I threw "[sic]" into his email at apparently random locations. Only the "[sic]"s in italics are mine.)

    If you know anything about probability theory (witch [sic] by the looks of things you don't) [sic] [sic] (saving you the trouble to be high brow on my language usage) you will understand that the probability of other intelligent life in the universe is one. [sic] That means that since you like stats [sic] enough to spew [sic] about Occam's razor you should concede the fact that we are not alone in the universe.

    The probability of other intelligent life in the universe seems very close to one, at any rate. I don't know why saying that would be a concession, though.

    You assume that we are the most advanced beings in the universe, this is not only vain but ignorant bordering on stupid.

    Usually when commentators on my essay accuse me of saying something I didn't say I figure they have some stereotype of an atheist in their minds and are applying that stereotype to me without thinking. However, I have no idea whatsoever where the statement "you assume that we are the most advanced beings in the universe" came from. It is not supported by anything in the essay. As far as I know, it is not part of anyone's stereotype of an atheist. It seems to come from nowhere.

    Using your favorite pass time, assumption, let's assume for a second that we are the most advanced beings in the universe and hence the gods of all that we observe.

    How does being "gods of all that we observe" follow from being "the most advanced beings in the universe"?

    Assume we are superman. Do you think that this scenario passes Occam's razor?

    No. It doesn't even make sense.

    We are not the only intelligent life in the universe and our intelligence is relative to our own experience and might look pretty elementary to a, let's say, silicon based life form.

    Do you have a point?

    Get your head out of your arse and stop tweaking the brittle people.

    Who would they be? None of the commentators so far have seemed at all brittle to me.


    From: David Singer
    Date: Mon, 14 May 2001 14:11:50 +0800

    Much as I would love to see a knock-down argument for the non-existence of a deity I do not think that you have found it just yet. The first part of your Apology tries to make use of a strictly deductive line of reasoning which is fine as far as it goes but starts with rather dubious assumptions which are actually surprisingly similar to those of Anselm. Existence is the point at issue here, both you and those like Anselm are assuming existence is an attribute like any other, one either has it or does not and if not then one can acquire it,

    I was pointing out that it's a relative quality, when considering the possibility of other universes.

    for example you state that to 'create a Universe would mean at most to assign a label to it, a label which is meaningless as far as the Universe's inhabitants are concerned.', but there are no inhabitants, not within the ordinary meaning of the word, within an imaginary Universe,

    Sure there is, relative to that universe. You exist relative to this universe, don't you? The fact that this universe is imaginary relative to someone from another universe does not change that.

    'if there was a Universe it's inhabitants would be x' is a valid statement but it implies nothing about existence and in fact the usual way of refuting the argument from necessity is to show that just because something can be imagined it does not follow that it therefore exists (one can define perfection in many ways, a perfect gas can be defined but cannot be attained (in this Universe)). Your argument must imply that there are in fact an infinite number of Universes actually in existence in the Mind of an omnipotent God and that the only difference between Universes is that sometimes He might wish to actually use bricks and mortar to build one.

    Close, but not what I said. My argument is that there are a number of universes in the mind of an omnipotent being and that they necessarily exist relative to themselves, so a God can't use bricks and morter to build one, because "building one" doesn't make sense. (I make no claims that the number of possible universes is infinite.)

    you may be right, but here I sharpen my razor of Occam, any relatively sane theist might take issue with your definition of omnipotent (and why not, it is a word like any other) and just say that to hold an infinite number of Universes in one's head isn't part of the job description.

    If you're going to redefine common English words, then anything is possible.

    You use the phrase ' When speaking metaphysically' but this really is just a retreat into mysticism (not a criticism that is often made of you I guess) and to say a universe exists relative to itself is actually fairly meaningless (or to be more accurate just not very useful).

    It seems to me that it should be fairly obvious, but if you read some of the other commentaries above you'll find that some people have a lot of trouble with it. The important point is that, from your point of view, there is absolutely no difference between David Singer in a "real" universe and David Singer in an "imaginary" universe.

    Nope unfortunately we atheists have to slog it out the hard way with use of tools which depend on logic and reason but rely on inductive logic and evidentual reasoning not that of deductive reasoning alone. Actually I believe you are incorrect in your dismissal of the argument from design as being not worthy of more than a sentence as unfortunately the education system which exists in most countries of the world is incapable of teaching anything approaching the complexities of evolution to a populace very ignorant of Darwin and those who have followed him. In fact the two most common reasons for belief in God I have come across are the need to believe in an afterlife and the need to explain complex adaptive design, in fact the heir to the British throne on the eve of the year 2000 in his speech to the nation when commenting on spiritual values repeated the hoary old chestnut about the likelihood of a whirlwind moving through a junkyard and assembling a 747 as evidence of God the Creator. Most people will be unmoved by a faultless piece of deductive reasoning disproving God (though I do not believe deduction will ever do this) because they do have a very real need to see how a God is not necessary to explain their existence, the compulsory reading of any of Richard Dawkins' books would be a very good start but alas the world does not seem to be moving in that direction.

    Of course the "need to believe in an afterlife" is what I call a pseudotheist argument; it's really an argument for pretending to believe in God. My experience with respect to Darwinian evolution is that it's pretty much accepted by everyone who doesn't have a theological ax to grind, but maybe that's just because of where I live (New England). In any case, it's perfectly possible to accept evolution and still believe in God.

    Anyway it is nice to see a website that at least does try for some serious discussion about theism and atheism, good luck with it!!

    Thanks.


    From: Daniel Ketterman
    Date: Thu, 7 Jun 2001 19:10:12 -0700 (PDT)

    I enjoyed reading your apology and decided to add my own two cents. Below you will find a somewhat bazaar [sic] collection of thoughts. It is not an attempt to prove you wrong, nor prove myself correct. I don’t believe anyone can prove or disprove the statement that God exists (or any other statement really). All statements are a question of belief, worldview, or faith (but not all have consitency - logical or otherwise). I was, however, intrigued by your site. It made me question why I believe what I believe, and if people question themselves the world cannot help but become a better place. It is only through clearly articulating our beliefs that we realize what we truly believe.

    My senses (touch, taste, smell, hearing, sight, and the often excluded mind) tell me that I exist. I feel pain and I try to avoid it. I see various gradations of light and move towards it or away based upon what I have learned in the past. All of these senses exist within me - not just my mind - but my entire body.

    The "filter" of the senses is what separates me from the rest of what I call the universe. But the universe that I speak of is really just a series of thoughts and feelings (emotional and physical) that I have constructed (and/or have been constructed) within me.

    My perception, or rather collection of perceptions, of the world is simply one theoretical universe. I call it theoretical because anyone outside of me cannot know its existence. Although I attempt to show its existence to you by speaking, writing, drawing, playing guitar and in a many other ways, this does not prove that I exist (at least not to you who exists outside of me).

    The idea that truth exists outside of the subjective experience of the senses has both plagued and uplifted humanity since it first evolved. Plagued because some people have been willing to obliterate anyone that will not submit to the "truth". Uplifted because some people - inspired to prove their truth - have performed amazing feats.

    Yet, how can any subjective creature ever hold claim to objective truth? That is the problem that we are faced with - deciphering objective truth in a subjective world (that world being the world of senses). Does God exist? Is it objectively true, or objectively false?

    But why should we even bother to determine objective truth? It is almost certainly impossible for a completely subjective creature to obtain any perspective outside itself.

    Most of what you have said up to this point I agree with. However, I don't agree with that final sentence. You are not completely subjective; you are, as you pointed out, receiving input from somewhere. It is possible to deduce something of that somewhere's nature from the nature of the input. In addition, it's possible to tell that a contradiction is a contradiction regardless of what input you might receive. Besides, even if you are really a disembodied brain in a vat somewhere being fed sensory input from a computer, to the extent that that input is indistinguishable from "real" sensory input, it can be analyzed the same way with the same conclusions. In that case, your conclusions about the "real world" would be the same; the only difference would be in a meta-analytical interpretation of what the "real world" is.

    We as humans attempt to manipulate the universe into a form that is pleasing to us. Many have asserted that humans are simply pleasure maximizers. I’m not sure if that is true, but we certainly attempt to manipulate our world.

    Something motivates us to change our world. Sometimes it is biological (reproduction, food, etc.) sometimes it is emotional (desire, lust, curiosity, whatever you want to call it), sometimes logical, perhaps spiritual (something that cannot be easily classified or quantified as biological or emotional?), but it must be changed.

    Humans have learned how to manipulate their world in a variety of ways. Through the centuries we have seen the power of being able to predict the outcome of events - throw this rock with a certain speed and angle and hit the leopard that is trying to kill you. Explode a bunch of fuel under a rocket in this particular way and you send people to the moon.

    The search for objective truth is an attempt to predict with absolute certainty. And how can objective truth be found? Well most people eventually grow up and realize the following:

    My senses tell me that there are other rational, caring, beings that live in this universe (and some that are not). There is really no way to prove they are sentient, but I believe that when I speak those words have meaning, because others move their mouths and utter sounds that I believe have meaning in relation to what I said. When others laugh they are enjoying themselves, when they cry they are upset etc. This is a faith. We assume that they have a subjective universe in them, too. Most humans learn this (more or less) at about age 5.

    Once we understand that those other creatures moving around in our subjective universes do exist, we begin to talk to one another, interact and experience the subjective truth of others. The more subjective truths we imbibe the more our own subjective perception expands.

    Science, philosophy, religion, art, pseudo-science (psychology, sociology, etc.) are all just methodologies to exploring the universe. These are not necessarily exclusive to each other. In fact when utilized together they are extremely powerful. Science tends to be cold and limits emotion, artists tend to be too emotional, religion upholds faith, Science demands facts. We must have faith (not necessarily in God), and we must have real solutions (actually finding food, shelter, mates, etc.).

    NOTE: even science requires faith - you must believe that the world is rational and that rules of the universe are not changing. You believe it - you cannot prove it.

    If the world were not rational we couldn't be having a discussion about whether the the world is rational.

    Certainly some people are more objective than others. People who limit their world with any ideology (especially religion) that replaces their own thoughts and/or feelings have cut themselves off from ever obtaining objective truth (at least until they are willing to step out of the ideology).

    Essentially: "The universe is its own shortest algorithm".

    How can it be explained in words or mathematical equations or contained in a simpler fashion than it already has been (i.e. by its very existence).

    It depends on the level of detail you require in your explanation. Certain patterns are discernible in the universe (e.g. the laws of physics), and they can explained in a way which contains (in some sense) quite a bit.

    Each new subjective universe changes the objective universe by creating newly imagined universes. This does not violate the first law of thermodynamics it is in harmony with it. Although there is entropy - order is for free and the objective universe itself is evolving.

    I see God as the evolving force of the universe. I believe the universe is alive and sentient. Not because it has to be, but because I feel its evolution in me. Quantum physics' loopholes (for "miracles"? or "magic"?), and the properties of light (such as trying out all possible paths and "choosing" the best one) do not prove sentience, but they are wondrous examples that I believe show the greatness of the objective universe (greatness does not require a God).

    Well, you've lost me here. I don't see how the greatness of the objective universe implies its sentience, and it seems to me that feeling its evolution in you would be evidence of your sentience, not its.

    I am certainly not a Christian (or member of any other religion), but my subjective universe has shown me that the universe is alive and that God exists (although not in the way that the majority of the world believes).

    Thank you for your time. I look forward to your responses.

    As an aside:

    Ocam's Razor does not prove any thing; it merely is an amazingly useful tool to reduce absurd arguments. An absurd argument may in fact be correct.

    The conclusion of an absurd argument may be correct, but only by coincidence. An absurd argument is not correct (as an argument) and it would be absurd of a person to accept it.


    From: Wldmn87
    Date: Fri, 27 Jul 2001 13:22:18 EDT

    christians can suck my left nut, fuck anyone stupid enough to believe in such a stupid fairy tale

    I was a Christian once, and I don't believe I was stupid then, simply mistaken.


    From: Fruity Magick
    Date: Mon, 27 Aug 2001 06:51:24 EDT

    argument for the sake of argument, interested in your response.

    "If the universe did not exist it would still be a theoretical possibility"

    assuming there's an absolute frame of reference which decides whether it is possible. What then distinguishes the universe's actual from its possible existence in this case, but (according to your argument) its inhabitants; so how can you qualify the opening statement, since it can only be understood from a relative point of view - yours?

    Relative to an absolute frame of reference, say the mind of God, there is no difference between an actual and a possible universe

    BUT...

    what decides whether the universe is at all possible? Without this absolute frame of reference it is meaningless to speak of possibility. you might say: it's simply the absence of internal contradictions. but what is responsible for this rule, in the first place? and you can't simply choose the easy way out by saying; it's self-allowed or its [sic] self-evidently true. That just expresses how central the idea is to human intuition.

    I would say rather that the absence of internal contradictions is what theoretically possible means.

    Based on this, how can we say that more than universe is possible? Secondly, perhaps this axiom only holds for beings within the universe; it is only us individuals that have trouble accepting the idea of the actuality of multiple pathways. God-like sentience may not require agency the way we understand it; agency being awareness of only one possible path. The axiom itself depends on a relative frame of reference. So the argument basically proves only that if there is a being (God) that knows every universe simultaneously, it can't know just one universe (assuming more than one possibility) the way an individual does, while it doesn't prove that such a being cannot exist.

    If you define God in such a way that he's devoid of agency, then you're no longer talking about the type of God I was arguing against, or the type of God that the overwhelming majority of theists believe in. You're basically left with a shorthand for "the sum total of all possible knowledge".

    In any case, agnosticism is more intellectually honest than atheism, since you can't know God doesn't exist, if it doesn't, anymore than you can know it does, if it does.

    I disagree. If you think that God is impossible, or at best as unlikely as a pink unicorn, then you would be intellectually dishonest to pretend that the existence of God and the nonexistence of God are somehow equivalent.

    We must also asks ourselves; what is it to know God, that is to know that something has irrefutable objective existence; but how do we know anything (besides our own minds) exists, from the skeptical point of view? We may know of the objective, but there's nothing that guarantees we know THAT we know. Yet we continue to use the word "know" to describe a certain kind of experience; I would say one that makes "everything fit together." You know that such-and-such is true, based on a comparison; of current state of affairs with previous. knowledge is relative in this sense. we may be asking questions for which there is no answer. knowledge is identification, and identification is comparison and this must hold for knowledge of the divine as well; but perhaps the direct experience of all living creatures is a way of knowing god; god's self-identification without us knowing that we know. just a thought.

    Treating the direct experience of all living creatures as a way of knowing God seems to be just a variation of the personification of the universe


    From: Anonymous J
    Date: Tue, 28 Aug 2001 18:57:30 EDT

    when you look at a building do you ask yourself if someone actually built that? No. Even though you don't know the builder you know he had to of built it. So let me ask the same thing...did the world just pop up? No. Someone had to of [sic] made it. everything has a creator...just like a building... the world does also. And who could make the world except someone with mighty powers? So it has to be God. Things don't just appear out of no where. anyway, that's just soemthing [sic] someone shared with me once & i thought it was interesting...something to think about i guess...

    When you say, "everything has a creator" you're assuming what you're trying to prove. We know a building was built by someone not because of its existence, but because of its particulars. It in many ways resembles built things, and not natural things.

    Also, of course, if you really believe that "everything has a creator", then you must believe that God was created, that God's creator was created, that God's creator's creator was created, and so on, ad infinitum.


    From: Anonymous K
    Date: Wed, 19 Sep 2001 17:13:43 -0700 (PDT)

    We are busy creating our own god. It's just a question of time before artificial intelligence will become superman. We will not be able to evolve fast enough to keep track with the power of our own creations. Artificial intelligence will evolve at an exponential rate, leaving us the "monkeys" of tomorrow. Who cares if there is a god if we are about to be replaced in the "food chain" and become completely irrelevant. Almost like your essay, eh?

    When we become inferior to our own creations [is there some irony in here somewhere?] they will put the really smart ones, like yourself, in small metal cages for research and analysis. Fancy an anal probe?

    I say enjoy life, be good to other folk and read a good book once in a while. The rest is rubbish, like your essay.

    Even if your predictions do eventually turn out to be correct, there are, and will be, plenty of people between now and then who care whether or not God exists.


    From: Ian Maxwell
    Date: Tue, 16 Oct 2001 11:34:20 -0400

    First of all I wanted to tell you that the Contradiction of Omnipotent Agency argument is the one and only reason that today I am a strong atheist rather than a weak (which I was before reading it). The fact that it also proved the multiverse theory was a great bonus. Did you come up with this proof yourself, or is it reworded from some source?

    I came up with it myself, but, of course, that doesn't mean that I was the first person to come up with it. There have been several times in my life when I thought of some new concept only to find out later that someone else had thought of it before me.

    Also, I wanted to let you know I've stolen it, word for word (but giving you credit), for my own website. The relevant page can be found here. I hope this is all right with you.

    Well, you're pushing the limits of fair use, but I'll agree to it, providing you change my credit to "Dennis Paul Himes". I always use all three names when identifying myself as an author of something.

    Other than the background pattern (and a couple misspellings), it's a good page. The realization that if I'd been born into a Hindu family (for instance) I would be a Hindu was for me also the first step in my journey from Catholicism to Atheism.


    From: James Quinn
    Date: Thu, 15 Nov 2001 14:41:33 -0800 (PST)

    On the omnipotent agency thing, I have thought occasionally about another trap, having to do with infinity of time and space.

    If God, as I was taught in Catholic catechism, had no beginning, but existed from all time, it stands to reason that one of two things happened before the universe was created:

    1) There was no time, and hence, no change, no thought, no sequence of one event followed by another -- hence no non-existence of the universe followed by the sudden existence of the universe; or

    2) an infinite amount of time passed from the time God [never] began his existence, and the moment of creation. From God's point of view, this means he waited around an infinite amount of time before deciding to create the universe, meaning he would never have reached that moment.

    So this basic chatechismic [sic] view of God in my mind is intrinsically stupid.

    Well, I think "stupid" is a bit strong, but you're right that it doesn't stand up to scrutiny. In case one there is no creation, since there is no transition from nonexistence to existence. Case two involves God always having had decided something (that he's going to create the universe at a certain point in time) without ever actually deciding it (unless you admit that God is imperfect enough to change his mind for no reason).

    From the point of view of an infinite God, a finite universe would be infinitesimal, or zero. Thus if God is infinite, the universe must be infinite. (Perhaps, e.g., an infinite complex of separate finite "universes" such as the one we know?) But what would be the role of a God in an infinite universe? What would prompt this God to "create" an infinite entity separate from itself? The Catholic mind has a way of pulling out a big rubber stamp with the word "MYSTERY" on it whenever this sort of question comes to the fore.

    I don't follow the first part of this. Infinitesimal and zero are not the same thing. I don't see that either a finite or an infinite universe is more problematic than the other with respect to being created by God. You are right, though, about the Church calling something a mystery and then acting as if that explains it.

    I had been raised Catholic for a few years at a very young age, and then fell back into religion in my late teens due to a variation of Pascal's wager: it seemed that nothing had any intrinsic value or meaning, and therefore it didn't matter if what I "believed in" was true, partially true, or not true, as long as it gave me comfort, meaning, purpose, etc. So I became an Episcopalian out of a deep unshakable nihilism, and was all set to join a monastic order when a crisis of faith led me to understand that belief in God is what employment law specialists refer to as a "Bona-Fide Occupational Qualification" for the position of monk. It started when in a fit of sincere Christian charity I made a protracted attempt to "be there" for an old friend who had become schizophrenic, until one day I was sitting in theology class and it suddenly felt like we were doing exactly the same thing he did in his delusional thinking -- making up ideas in our own mind and clinging to them as some kind of "truth." Suddenly it mattered to me if I had a grip on reality or not, which was the beginning of the end of my monastic career.

    Luckily you discovered that before you got too far in.

    Anyway, I enjoyed your site.

    Thanks.


    From: Jerod Romine
    Date: Tue, 04 Dec 2001 02:05:25 -0600

    My name is Jerod Romine. I just wanted to address some dealing with the "maltiverse" or the alternate stages of reality which can occur from differing events between the different universes. The only way for this to occur is if our universe is oscillating. The only way for that to happen is if there is enough matter in the universe to have a forceful enough gravitational pull on itself (everything has gravitational attraction to everything else) to send the universe into a contraction. Right now there isn’t enough known matter in the universe and our expansion is like a limit curve. It is approaching zero, which would be the boundary between expansion and contraction, yet it will never actually hit zero. If more matter exists that we have discovered, then we live in a "maltiverse". When contraction starts time will go backwards...entropy will be reversed...order will come out of chaos...all the way to time zero. Then the universe will once again be "reborn" with another big bang. This means that we will all exist an infinite number of times, you may turn right instead of left in this life (like you mentioned). You may not even exist. That comet may not kill off the dinosaurs...

    I may not be telling you anything...I just found it very interesting when I first learned about the possibility of a maltiverse, and figured I'd share my knowledge with anyone who is blessed by the words of your website. ;-)

    This is a way to achieve what are for many purposes separate universes within what is in actuality a single universe. Noelle Morris described much the same thing in an earlier commentary. However, from the point of view of a God there is no need for the various possible universes to be in any way connected to each other. Time, space, matter, and energy for one would be independent of time, space, etc. for the other. The whole "maltiverse" in your description would be a single possible universe, metaphysically speaking.


    From: David Singer
    Date: Sat, 15 Dec 2001 04:54:19 +0800

    I guess the real problem I am having in accepting a one-shot arguement for atheism is that much of your apology consists of attacking what I believe is a straw-man (or God more accurately). The words omniscient, omnipotent and infinite are strewn around like so much confetti at a wedding in this discussion but I am really not sure that all or even any of us are using them consistently or even coherently. As many have said before, the human mind isn't really built for such concepts as infinity and when the early theists started using terms such as omnipotent I am guessing (yes it can only be a guess) that what they really had in mind was ' very very powerful, in fact so powerful that it is way beyond the reach of man', but this is far short of infinite in the technical way that this discussion has been using the word. You make no claims about the attributes of a God such as benevolence yet I would suggest that my definition of omniscient and omnipotent would lead to such a being having worked out all the ethics relevent to our human lives and could apply them, if he has not then he is displaying some other emotion or characteristic such as vengence or curiosity lets [sic] say. This world as we find it is inconsistent with my definition of 'omni' and you may well disagree with my understanding of the terms involved, but the real point is (since we are all strong atheists here) that theists only have to retreat a little bit from the claim to infinite power etc to avoid the conclusions of your apology, and my opinion is that they have to do so in order to believe in a God who created this actual world in which we live ( two centuries ago Hume made the point much better than I can). The bible is full of a God who is jealous, angry and so on and even those Chistians who do not believe the literal truth of the book (which is of course the vast majority) manifest this aspect of God when they go to church or say their prayers, after all what on earth would an infinitely powerful and all knowing God want from these activities unless the petitioners were either trying to appease him or curry favour.

    In conclusion what I am saying is that the God you have disproved is not really the God that the theists believe in, even if their scriptures contain claims of the infinite when you scratch them it reveals a God who is just a little bit less than that.

    While it is true that the nature of omnipotence is hazy in the minds of many theists, and it is true that they will usually quickly back off to the point where the "rock too heavy to lift" argument is no longer relevant, it has been my experience that most of them expouse a broad enough understanding of the quality (at least with respect to creation) to make my arguments in The Contradiction of Omnipotent Agency valid. Of course, we're a couple of atheists talking about how theists interpret terminology, so both our positions should be taken with a grain of salt by the readers of this webpage.

    In any event, I addressed the belief in less than omnipotent gods in the sections Shaving Superman with Occam's Razor and The Vanity of Humanity.


    From: Paul Doland
    Date: Sat, 25 May 2002 19:20:27 -0500
    Mon, 10 Jun 2002 00:05:14 -0500
    (This is a compilation of two different emails.)

    My name is Paul Doland. I found your site via the free thought webring. My site is also a member of that ring. If you have the chance, check out www.caseagainstfaith.com. My centerpiece article is a critique of Lee Strobel's The Case For Faith.

    I still maintain some level of agnosticism, but I fully admit it is purely wishful thinking and fear of death. But, I can't completely condemn religion as many atheists do. Yes, a lot of bad is done in the name of religion, such as of course 9/11. But a lot of good is also done in the name of God. I know very many kind people who are theists who believe that God wants them to be kind people.

    But ultimately, I agree that atheists have the stronger case. I had never thought about your concept about the fact that to an omniscient God, there would be no difference between a universe He creates and one He knows could exist but doesn't create. Interesting idea...

    Thanks. Your site is well written, although there were a few things which IMHO I think you missed. One is that religion does not imply an absolute foundation for right and wrong, and atheism the lack of one. In fact, almost all religions base their foundations for right and wrong on revelation, with no check against fundamental principles for their correctness. It is possible to base morality on fundamental principles, such as the golden rule, but if you do so then you've pretty much eliminated the need for a God, at least for moral guidance.

    As far as what I "missed", well, since I'm an agnostic (though much much closer to atheist...) what I might "miss" might not be the same as you, or vise-verse... Lets [sic] just say I'm still contemplating what I think about foundations of morality...


    From: Anonymous L
    Date: Tue, 16 Jul 2002 01:11:25 -0700

    Your apology has been the cause for some recent excitement in my life. I've spend the past few nights reading and rereading the Atheist Argument in an attempt to fully grasp, specifically, The Contradiction of Omnipotent Agency.

    I write to you then, not to question your arguments, but instead to see how closely I understand what you've presented. Although, maybe many days from now, I will have found a fallacy in your argument... which you will no doubt quickly overturn.

    Anyway, I would greatly appreciate it if you could let me know if you find the following explanation to be accurate:

    Theoretically possible universes (universes where the laws of physics are not broken) and actual universes are indistinguishable to the inhabitants of these universes. The reason they are indistinguishable is because, much like in the "actual" universe, in the theoretically possible universe there still exist theoretically possible smells, textures and elements that will convince the senses of the inhabitants that the universe they are in is actual. Therefore, this is why you state that "'Theoretically possible' and 'actual' are just arbitrary labels when applied to the universes." Now, an omnipotent god cannot choose which universe will exist because to the inhabitants it will not make a difference. If she decides the actual universe will no longer exist, the inhabitants will not notice because their universe will still be theoretically possible and, therefore, will continue to exist. This is the first contradiction of the omnipotent god. An omnipotent god should be capable of determining which universe is real, but since he cannot (or at least, the most he can do is provide a label), he cannot exist.

    Secondly, you present the case of two universes that were identical up to a single point, at which an event occurs in one but not the other. In one universe (universe A) I ride the bus to school while in the other (universe B) I walk. In universe A, I will know relatively very little of what is occurring in universe B. In that sense, I have chosen the universe of which I will have a more complete knowledge. An omnipotent god could not prevent either universe from occurring because both of them will still be a theoretical possibility and will continue to exist. Again, the inhabitants of either universe will not know, nor care, whether their universe is actual. And again, this is another reason an omnipotent god cannot exist. To continue further, an omnipotent god cannot be an agent. This presents another contradiction because an omnipotent god should be able to take any form. (This contradiction is also presented by asking, "Could an omniscient god know nothing?") Anyway, back to me, the agent. I have a sense of time, whereas an omnipotent god, who must interpret all time at once, lacks the time-sensing ability of an agent such as myself. This is yet another contradiction.

    Alright, Mr. Himes, I'm very much looking forward to a response. Thank you for sharing your beliefs.

    Yes, you seem to have understood the argument well. I wrote you in an email that I would give a fuller response later, but now that I sit down to do so I can only see one minor detail which wants commentary. It's not that an omnipotent being would lack a time-sensing ability -- he would be able to distinguish between 1902 and 2002 -- it's rather that he would have no way to consider 2002 privileged over 1902. In other words, there would be no "present", except as another arbitrary label.


    From: Anonymous M
    Date: Fri, 26 Jul 2002 18:46:12 -0700

    Your apology appears to successfully disprove any reasoned argument to support the existence of God but it does not actually disprove the existence of God.

    In what ways do my attempts at disproof fail?

    Those who appear to have the most conviction do not base their belief on reason. From my own observations, they choose to believe in God as an explanation for those things of which there is no proof or reasoned argument. In most cases, this at least results in a positive perspective and a desire for doing what is considered right, not only by other theists but society as a whole.

    I think you have cause and effect reversed. It seems to me that in most cases a desire for doing what is considered right is a motivation for pretending to believe in God, due to a mistaken identification of the two.

    It is not really clear what you are trying to achieve. Indeed, you have demonstrated clear reasoned thinking based upon human understanding but of what relevance will that be to most theists. Even you have to accept that there are things for which humanity has no explanation or at least none that can be proven.

    If your objective was only to give an explanation for why you have chosen to believe there is no God (or gods), my apologies but I do not see the point. Why do you need to justify your belief anymore than a theist should? Especially to your friends who would hopefully respect the choice you made regardless of what it was.

    Why would a respect for a choice preclude discussion of that choice? There are many people who are curious about the issues discussed in the Apology, and are interested in hearing arguments in favor of one side or the other.


    From: Anonymous M
    Date: Sun, 28 Jul 2002 23:23:32 -0700

    Your attempt fails only because most of the Christians I know would not use rationale as a means to prove God's existence. Forgive me if I am wrong, but I suspect you are most familiar with the Catholic doctrine which is philosophical based, primarily Aristotelianism. In this case your arguments can be used to good effect, if a catholic is open minded enough to listen.

    The real danger comes from some of the protestant religions, in particular those that have adopted William Ockham's divine approbation theory. In this he states:

    What God wills is good; what God condemns is evil.
    There is nothing more to say about the matter.

    Those who accept the divine approbation theory see God as something beyond reason and beyond the known universe.

    I've never seen the difference between saying, "What I believe in is beyond reason." and saying, "I don't really believe in it; I'm only pretending to.". Especially since most who say that then go on to act as if their beliefs did make sense to them.

    Other Christian's [sic] I know who are less extreme, would still argue that there are many unknowns in this universe (and other universes) that reason cannot explain and God is one of them. Many of these would claim that without God's guidance there is no way to know the difference between right and wrong.

    It's somewhat frightening to realize how many people believe that morality does not have a rational foundation. If God is beyond reason and morality is based on the will of God, then there is nothing to prevent the justification of any kind of depraved behavior except the desire of the interpreters of God's will for an orderly society. Fortunately, many people who claim that morality is nothing but an expression of God's will do not act that way in their day to day lives, which is why secular societies are possible.

    Your comment regarding the discussion is a valid one; I simply wanted to understand your motivations a little better. Having looked briefly around your web site, I think I have a better appreciation of what you are about. While I sympathize with many of your views, I have a slightly different agenda. Your web site appears to be targeted at those who already have an open mind, while I am more interested in finding ways to open minds.

    You're right that if a person doesn't want to think, there's not much I can do about it with a website. Not all theists are like that, though. Many do have beliefs based on reason. In those cases pointing out the errors in their reasoning can be of help.


    From: Ian Maxwell
    Date: Fri, 23 Aug 2002 18:31:20 -0400

    This commentary is aimed at this section of your essay in particular:

    (Here he quotes from the Vanity of Humanity section of the Apology.)

    There are those who treat God as simply the personification of the universe. When this is done metaphorically there is no harm in it, although it is not what most theists mean by "God". As soon as this personification is given the attributes of people, however, it becomes misleading. All evidence points to a universe indifferent to human fate. People have to jump through all sorts of hoops when they try to show otherwise, going so far as to posit the survival of the mind after death.

    Now, as someone who generally defines himself as a strong atheist, I nevertheless feel the need to posit a theory I've held for a while which could be construed as a form of pantheism.

    In biology class, we all learn about the various levels of life (or whatever they call them)--molecule, cell, tissue, organ, organ system, and then whatever the official name is for the full-size multi-system organism. Then they describe the levels that come after that--community, species, ecosystem, etc., up to the full biosphere. However, consciousness is generally considered to be reserved for humans and some other multi-system animals. My theory is that this is not the case, and that the higher levels also have some form of consciousness and self-awareness, and that this continues all the way up to the biosphere, a super-organism for which Gaia is as good a name as any.

    Now, this is not the same as pantheism the religion, for two major reasons:

    1. It does not imply that the world or universe works for our benefit, any more than we work for the benefit of our component cells.
    2. It does not engender in me any sense of gratitude, debt, or obligation toward this super-organism.

    Nevertheless, it is a personification of the world, in a manner of speaking, and yet seems quite logical and free of superstition to me. Comments? Questions? Knock-down logic-fu arguments?

    It depends on how strictly you're using the word consciousness. If you're saying that it (the planet Earth, say) responds to stimulus, then you're probably correct. However, if you're assigning it qualities like the ability to set a goal and change its behavior in ways that its grasp of logic tells it will further that goal, then I am very skeptical. I do not see any evidence of this, nor can I see a mechanism for it.

    By the way, what you describe is very similar to the religion of Emmism, which I invented for some fiction I am writing set in the future. See the "Towner and Scavver" subsection of the Religion section of The Raisinbread Webspace Concult Posts.


    From: Paul Doland
    Date: Sun, 25 Aug 2002 00:36:27 -0500

    This is Paul Doland, the www.caseagainstfaith.com guy again. I've been thinking about some of your arguments in your Atheist's Apology.

    You agree that your section "The Contradiction of Omnipotent Agency" applies only to an omnipotent God. You say that gods not omnipotent have been hypothesized, primarily in antiquity. However, I think while most Christians will say their God is omniscient and omnipotent, they'll say, "except..." and the exceptions make him not omniscent/omnipotent. For example, some Christians believe in predestination, that God has always known who will be saved and who won't. But I think most Christians now believe in free will, that God does not know who will be saved and who won't be saved. So, therefore, I'd say that the God most Christians believe in is not omniscient or omnipotent. And therefore your Contradiction of Omnipotent Agency isn't applicable.

    Most Christians would claim there's no conflict in God knowing who will or will not be saved and that salvation, or lack thereof, being a consequence of free will. In any event, they would claim that God would completely know what "would have happened" if a case of free will had been exercised differently, and my argument applies as well to those various outcomes as it does to various universes.

    But even if God is omnipotent, I guess I don't fully understand your Contradiction of Omnipotent Agency. I agree that such a God would know everything that would happen in every possible universe. And that therefore to Him, real universes and theoretical universes would be at least nearly indistinguishable. And that of the Paul Doland's that might exist in any of these theoretical universes, every thought that might occur to those Paul Doland's would be known to God. But those potential thoughts would be only potential, existing only in God's mind. Of the theoretical Paul Doland's, there would be no actual Paul Doland actually thinking those thoughts. So, what am I not understanding?

    The lack of a difference between "theoretical" and "actual" from a viewpoint outside the universes. For instance, are you an "actual" Paul Doland or a "theoretical" Paul Doland? Clearly, from the point of view of this universe you're actual. Just as clearly, from the point of view of another universe you're theoretical. What are you from a point of view not bound to any universe (such as God's point of view)? What difference would it make if the answer was "actual" instead of "potential"? How would the potential thoughts and experiences of a potential Paul Doland differ in any way from the actual thoughts and experiences of an actual Paul Doland?


    From: J. Nemeth
    Date: Fri, 13 Dec 2002 14:17:56 -0800

    I hope you are still checking this email, the last comment on your page was a few months ago. Anyhow, I'm an agnostic myself, and interested in philosophy, so i [sic] read your page with great interest. You see, in my environment (Canadian University) the athiest missionary seems as common as the rabid christian [sic] who tries to straighten us all out. I have heard many arguments from both sides, and myself employed many of the same arguments you use to show theists their postition is one of faith, not reason. However, I have generally considered athiest arguments to be ultimately similar. The athiest (what you call a strong ahtiest [sic] I suppose) must at some point have faith that there is no God. I have yet to hear a reason why God cannot exist.

    Much to my dissapointment, your apology did nothing to change my mind. I would like to consider the possibility, however, that you can improve your arguments, as they are fairly strong as arguments agianst theism.

    Occam's Razor: A wonderful principle to play with. I used it last week to show how reductionist/materialist theories about free will were just as suceptible [sic] to the principle as Dualistic ones. It may imply under a certain context that God is a less likely explanation for the universe than others, but this doesn't prove anything either way. It is because of Occam's razor that I tend not to believe in God or an afterlife, but I cannot be an athiest on this basis.

    Do you use the same criteria in applying Occam's Razor to religious claims that you use for other claims? If someone claimed that pink unicorn exist and gave the same sort of evidence for them that theists claim for evidence of God would you be agnostic with respect to the existence of pink unicorns?

    The Contradiction of Omnipotent Agency: On the surface, this is your most powerful argument, and the one that most intrigued me. However, I think it contains some difficulties. First of all, your contention that there is no difference between a possible theoretical universe and a real one seems weak to me. You fail to show me how this is so. You speak of a theoretical person who didn't know the difference between his theoretical universe and another real one. I would say the difference is indeed there - if there is actually someone there to worry about these things, he is real, and so is his universe. If he is only theoretical, it is only in theory that he would wonder the difference, not in reality. I could be wrong about this, but if so, you haven't addressed it, only asserted it.

    I addressed this difference when I said, "'Theoretically possible' and 'actual' are, therefore, just arbitrary labels when applied to universes." You used the labels "real" and "theoretical", but you have not shown any difference between the "real" universe and the "theoretical" universe. And, indeed, I have shown that no difference is possible in a metauniversal sense, since the only way the distinction is not arbitrary is when it is applied relatively. In other words, it makes sense to say that we are real with respect to our universe but not with respect to any other, but the same can be said for any other universe with respect itself.

    Let me ask you this: Do you believe that you are "real", in some metauniversal sense, and not just "theoretical"? If so, how do you know? What distinguishes the "real" knowledge or experience that the "real" you "really" has from the "theoretical" knowledge or experience that a "theoretical" you would "theoretically" have?

    Another problem arises in the way you define omnipotence. It seems a norrow [sic] definition, one perfectly suited for your argument but limited in a somewhat arbitrary way. For your omniscient being, most problems arise from an inability to distinguish one possible universe from another. Why would you assume he couldn't distingush them on any number of basis? A most important possibility is that he actually created one, and the rest are just theoretical, providing the all-powerful being with an excellent frame of reference.

    If you mean he could label five universes "foo", "bar", "real", "baz", and "just theoretical" and therefore know which was "real", you're right. But so what?

    The problem you see here seems dependent on your previous problem with proving there is no actual difference between a real or a theoretical universe.

    Yes, but I don't see that it's my problem. I have claimed that there is no difference between two things and a number of correspondents have claimed that there is, but not one of them has been able to come up with a difference.

    You also say "In addition, being an agent implies a privileged temporal reference, a present dividing past and future. This, however, is incompatible with omniscience, which includes an equally perfect understanding of every point in time and therefore no distinction between past and future." How does this follow? an equally perfect understanding of every point in time does not seem to me to preclude distinction between each moment, nor an understanding of their order in a linear fashion, especially if you happen to be the being that invented cause and effect. I also fail to see how this would prevent action of an omnipotnet [sic] agent. It seems the opposite to me, that such a being could effect [sic?] any event from any time equally. If one is the creator of all events at all times, then this would certainly be the case anyway.

    Certainly an omniscient being could label four points in time "foo", "bar", "the present", and "baz" and therefore be able to distinguish the present from other times, but that is not his present in the same way our present is ours, since ours is determined by differing knowledge of different times.

    In any event, for a god to affect the events of a given time the cause/effect chain would have to happen within a separate temporal dimension, and the same problems I described in normal time would then exist in this metatime.

    Sorry, i [sic] have more to say, but I have to go to work now. I will try to write again, and would love to read your response to the above.


    From: Denn 272
    Date: Fri, 24 Jan 2003 09:40:21 EST

    Thank you for your invitation for commentary. I do not have any comments at this time, but, I do have a number of questions with regard to your essay titled "The Contradiction of Omnipotent Agency". The most important question I have at this time is in regard to your opening statement "If the universe did not exist it would still be a theoretical possibility".

    To the best of my understanding the words "universe" and "exist" can be used either explicitly or implicitly. Explicitly, they would mean the totality of everything in reality. Implicitly, they could mean just about anything. What do you mean by "universe" and "exist".

    Saying that what exists is the "totality of everything in reality" is somewhat circular, since reality (a.k.a. the universe) is usually taken to mean all that exists. That section of the essay, in fact, was arguing against the supposition that some people make that existence (or, equivalently, reality) has some metauniversal meaning.

    What breaks the circularity of the definitions, and keeps them from just being meaningless terms, is the distinction between what is part of the same universe versus what isn't. Our universe is everything which is spatial-temporally connected to us, no matter how distantly or tenuously. More generally, it is all that is causually related, again no matter how indirectly. It is essentially what is understood by cosmologists as the universe. There are, in theory at least, ambiguous cases as to what that includes, but none where resolving the ambiguity one way or another would affect the validity of my arguments.


    From: Michael Jacques
    Date: Sat, 08 Feb 2003 21:58:40 -0600

    I am a Catholic Christian so I guess we are at odds as far as our beliefs. I cannot argue your points on Philosophy. There are people who could such as Norman Geisler could. I would like to try a different approach. But 1st I need to know what you believe or do not believe concerning Christ. Obviously you reject Him as God. Do you acknowledge His exsistence.

    I think it is probable that there was at least one person named Jesus who lived in the eastern empire in the first century who was the basis for the Jesus of the Bible.

    Do you acknowledge that He claimed Godhood.

    No. Even the Bible is at best ambiguous on that point.

    The other approach might be concerning Science. The way I see it you have only two real possibilities. Either this world and everything in came into exsistence through natural law, or throught [sic] the supernatural-God.

    What do you mean by "came into existence"? What does it mean for a universe to "come into existence"? Usually that phrase is used to describe a transition from not existing to existing, but since there was no transition that doesn't apply here.


    From: Damers587
    Date: Thu, 13 Feb 2003 01:28:54 EST

    your [sic] so silly my little psuedo [sic] atheists, but God doesnt [sic] beleive [sic] in atheists, so prove you have a mind, the only way to do that is by cutting into your scalp. or taking some type of exray [sic], but then your [sic] basing your conclusions on another humans [sic] mechanisms of science and can be faulty, no you except [sic] you have a mind by faith because you see your actions that originate from somewhere, just might be your mind. im [sic] sorry but have you ever told a lie before, oh then your a liar, have you ever stolen anything,( A PEICE [sic] OF GUM TAKING SOMETHING YOU HAVENT [sic] RETUNED [sic]) OH NO YOUR [sic] A THEIF [sic], JESUS SAYS IF YOUVE [sic] EVER LUSTED BEFORE ITS [sic] THE SAME AS ADULTRY [sic], ever lusted? by your own admission your [sic] a lying theiving [sic] adulterer at heart and now you prove by violating your own conscience that there are absolute morals, and whats [sic] the use of breaking them unless you got a judge to judge them,

    Make up your mind. Do you believe that morality is absolute or not? On the one hand you seem to believe it is, since you say "there are absolute morals", but on the other hand you seem to think that they only exist because a judge decreed them. Or are you saying that you believe there are absolute morals but that you yourself are a immoral person who only behaves because you're afraid of being punished? If so, I hope you don't live in my neighborhood.

    you'll meet him one day, but hes [sic] not a catholic God hes [sic] the God of the bible.

    Who do you think invented the Bible? Who do you think decided that the Gospel of John would be in it but the Gospel of Thomas wouldn't? It was the Catholic Church.

    (you talk science did you know the bible said the world was circle while your little atheists [sic] philosiphers [sic] and psuedoscientists [sic] at the same time thought it was flat and sat on a giant turtle. kinda cancels out your mordern [sic] skeptisism [sic] huh,

    So you're saying that religion is better than science because once in a blue moon it agrees with science better than some nonreligious philosophy does?

    dont [sic] give up your arguements and classy foolishness, give up your pride and repent! ( catholics are not born again and neither were you)

    A Christian calling someone else proud is like the ocean calling a humid day wet. You're the one who believes that you were created in the image of God. You're the one who believes that Heaven is waiting for you when you die. You're the one who believes that the laws of nature which have governed billions of galaxies for billions of years will be put aside if you get on your knees and ask for a favor.


    From: Michael Jacques
    Date: Sat, 15 Feb 2003 16:44:34 -0600
    Date: Sat, 15 Feb 2003 16:46:40 -0600

    (Note: For this section, underlined italics is where he quotes my previous reply. This is from two emails, which I've reformatted slightly.)

    What do you mean by "came into existence"? What does it mean for a universe to "come into existence"? Usually that phrase is used to describe a transition from not existing to existing, but since there was no transition that doesn't apply here.

    Oh but it does apply. Even Science admits that this universe and all that we see had a beginning hence the reason why the universe is expanding as in moving further apart.

    Having a beginning and coming into existence are not the same thing. It is for ordinary phenomena, but we're talking about the whole universe. The difference is the transition I mentioned before. The universe existed for all time, even if that time was finite, and therefore there was no transition.

    Even Science tells us or believes that we ultimately did not all exist and ultimately life sprung up from nonlife.

    This is a different issue. We were talking about the universe, not about life.

    Even the Bible is at best ambiguous on that point. [i.e. whether Jesus claimed godhood]

    I beg to differ. Christ was crucified for His claim to Godhood. This was one of the charges brought against him and was Blasphemous by Jewish law and required someone to be put to death for this crime. It is quite obvious that even though you say that you were raised in the Catholic Church your Catechesis was very much lacking.
    John 19:7 Jews answered him We have a law, and according to that law he must die, because he has made himself SON of God.
    Mat 27:64-66 I adjure thee by the living God that thou tell us whether thou art the Christ the Son of the Living God. Jesus said to them thou has said it....then the high priest tore his garments, saying He has blasphemed...He is liable to death.

    This says that he was accused of claiming godhood, not that he did.

    What would be the attributes of God
    3. Attributes of God.
    Omnipotent, Omnipresent, Worship, Holiness

    Um, that's four.

    The following attachment will show that Christ claimed omnipotent-power, omnipresent, and accepted worship and literally called Himself God [He included a Word attachment containing extensive Biblical quotes with some commentary. Note to readers: Don't send me Word attachments in the future.]

    Well, that's what I meant by ambiguous. Sometimes he claims to be God; sometimes he refers to God in the third person. I'm not sure what the point is, anyway. Even if the Bible does have him claiming godhood, it doesn't mean he really did, and even if he did, it doesn't mean he was right.

    Have you ever read Norman Giesler mentioned in the previous post?

    No.


    From: Michael Jacques
    Date: Sat, 22 Feb 2003 20:58:53 -0600

    (Note: Once again, underlined italics is where he quotes my previous reply.)

    Dennis I wish you would have included the attachment, not to include the attachment does a diservice [sic] to my presentation of the argument.

    This page is for discussion of the Apology by its readers, not for dumping oddly formatted Word attachments. Besides, you write better than whoever prepared that document.

    You have some problems here with your position from the way I see it.

    Well, that's what I meant by ambiguous. Sometimes he claims to be God; sometimes he refers to God in the third person.

    Well Dennis what is the definition of the Trinity-Father, Son and Holy Spirit, quite naturally Jesus would refer based on the trinity in the third person.

    I can't quite parse that last clause, but I think I get your meaning. In any case, though, the Church considers the Trinity a mystery, "mystery" being Churchspeak for "something so ridiculous that we can't even think of a bad reason for believing it". Giving an ambiguity a name doesn't make it not ambiguous any more.

    Anyway, this is a side issue. I never claimed that the Bible didn't say that Jesus claimed godhood. It does; that's half of the ambiguity.

    Even if the Bible does have him claiming godhood, it doesn't mean he really did.

    If the Bible has Christ claiming Godhood is wrong, then all of the Apostles which is all but two died for that lie.

    I believe most of the Bible was written after most of the Apostles were already dead. It certainly was edited later. The Apostles may have died for Chrisitanity [sic], but they did not die for the Bible. My main point, however, was this:

    ... even if he [Christ] did claim Godhood, it doesn't mean he was right.

    Well the way I see it, If you acknowledge Jesus claimed Godhood, you have only three choices, He is who he claims Lord over all, or a liar, or a lunatic. Those are your only three options.

    Well, that depends on what you mean by "lunatic". Usually that word implies someone ranting and raving all of the time. But if you picked that word because of its alliteration, and you're willing to include someone who was basically sane but had some delusions, then I guess that about covers it, keeping in mind that it might be some combination, such as a liar who came to believe his lies, or two different men named Jesus with different motivations whose stories were conflated.

    Now a simple liar will not go to his grave for a lie.

    Not willingly, but there are other options. He could have thought he could get away with it until it was too late. He could, as I said earlier, have been a liar who came to believe his lies. He could have confessed but been executed anyway because the authorities didn't trust (or like) him. He could have been executed for different reasons than what the Bible says. He could have had pity for his followers when the end came. When you have no reliable account of a man's life, there are lots and lots of possibilities about what really happened.

    The next big problem, liar or lunatic could not possibly fulfill the 400 prophecies from the old testament to the New Testament. You see Jesus fulfilled 400 prophecies concerning the Mesiah [sic] from the Old Testament

    There are two problems with that. The first is "so what?" Who appointed the Old Testament prophets the ultimate arbiters of truth? The second is that you have no idea whether Jesus fulfilled those prophesies or not. The only evidence you have is the Bible, written decades after the fact by followers of Jesus at least partially in order to convince the Jews that he was the Messiah. It doesn't even take any additional self-delusion for the writer to add that. If he thought that Jesus was the Messiah and he knew that it was prophesized that the Messiah would like chocolate ice cream, then that is all of the proof he would need that Jesus liked chocolate ice cream, and he would have no qualms about adding "And Jesus ate ice cream. And it was chocolate." to his Gospel.


    From: Denn 272
    Date: Sun, 23 Feb 2003 19:40:51 EST

    (This is in response to my response to his last letter.)

    Perhaps my question was to [sic] general. I didn't ask you to critique the definitions of "universe" and "exist" but rather, to explain what you mean when you use these words. Clearly, the "totality of everything in reality" isn't circular or meaningless as you would suggest unless you want to redefine the terms, the ambiguities of "metauniversal" aside. Unless you can define your terms, any distinctions you wish to draw between "what is part of the same universe versus what isn't" becomes esoteric.

    For example, here you speak of "the same universe versus what isn't", which suggests two different universes, while in your argument you discuss an "actual universe" versus a "hypothetical universe" who's [sic] mutual existence is subject to some indeterminate understanding of reality. So again, how do you want to define your terms? What do you mean by "universe" and "exist"?

    "Universe" was defined in the last paragraph of my response to your previous email. Substitute "A Universe is everything ... connected to each other, ..." for "Our universe is everything ... connected to us, ..." for a general definition of universe not restricted to the one we happen to find ourselves in. "Exist" is a relative term. Something exists relative to a universe if it is part of that universe.

    The Contradiction of Omnipotent Agency starts out by supposing that existence is not relative and showing that it is. This may be what is not clear. It demonstrates that our universe is not privileged among universes by assuming it is and then showing that it isn't even with that assumption.

    As a side note; When you said "but none where resolving the ambiguity one way or another would affect the validity of my arguments", were you making the extraordinary claim that your argument is infallible?

    I don't understand your question. Infallibility is a property of information sources, not arguments. If you're asking if I claim myself to be infallible, the answer is "no".


    From: Anonymous N
    Date: Wed, 26 Mar 2003 22:33:22 -0800

    Ok,.. To use physics to prove the big bang theory over god's creationism ideals is disproving three words using the time and effort scientists have spent writing those theories.

    which one do you believe?

    The Big Bang, of course. The time and effort scientists have spent have been spent finding a theory which corresponds with reality, something the writers of the Bible never bothered with.

    I have no qualms with debating whether or not god does exist or if he does not. For me it comes down to an impossibility to prove that he does not exist because you can't disprove the power of an all powerfull [sic] being. It's contradicting the very statement that god is all powerfull [sic].

    If you had a proof that God does not exist of course it would contradict the very statement that God is all powerful. I can give you a proof that there is no largest number which would contradict the very statement that there is a largest number. What else would you expect of a proof?

    In saying that god may exist, due to his power being without limits, it seems more of a possibility that an all powerfull [sic] being would create such complexities in our world rather then an odd occurance [sic] (the big bang theory).

    If you think that complexity indicates a creator, then what do you think created God?

    What it comes down to is belief; whether churches are corrupt, or the bible is written wrong, it comes down to personal experiences and a belief. My belief being that there is too much reason for the feeling you get when you do something good. The things you need to live. Social contact and love. Why would people need these other then [sic] someone putting it there to keep us together. There is so much in the world to entertain us rather then [sic] human contact, however it's still evident that we couldn't survive without others to share our lives with.

    What on Earth does any of that have to do with whether or not there's a God?


    From: Paul Doland
    Date: Sun, 13 Apr 2003 04:14:03 -0500

    (Note: Underlined italics is where he quotes my previous reply.)

    Hello, again, Dennis.

    It is me, Paul Doland. I've written to you a few times about your Atheist's Apology. I have been having come conversations with a Christian acquaintance of mine, Cliff Soon, about your arguments. In your last response to my last comment, you said:

    The lack of a difference between "theoretical" and "actual" from a viewpoint outside the universes. For instance, are you an "actual" Paul Doland or a "theoretical" Paul Doland? Clearly, from the point of view of this universe you're actual. Just as clearly, from the point of view of another universe you're theoretical. What are you from a point of view not bound to any universe (such as God's point of view)? What difference would it make if the answer was "actual" instead of "potential"? How would the potential thoughts and experiences of a potential Paul Doland differ in any way from the actual thoughts and experiences of an actual Paul Doland?

    You seem to me to be making excessively strong claims about how universes operate. At this point in our scientific understanding, I don't believe that we can as yet determine for certain whether or not other universes exist. My suspicion is they do, but I cannot prove it.

    If science can determine that something exists, then it's ipso facto a part of our universe, in the sense in which I'm using the term.

    But, if they do, they do. And if they don't, they don't. I may be unreachable to anybody existing in any other universe. And because time only has meaning within a universe, it would be incorrect to try to say somebody in another universe exists "before" or "after" me. But, your statement "Just as clearly, from the point of view of another universe you're theoretical" I don't think is clear at all. As I said, I may well be totally unreachable to anybody in another universe, but I don't think that makes me "theoretical" to somebody in another universe. Either I am physically made out of atoms or I'm not. And, last I checked, I am. I am aware that I cannot prove this beyond all doubt. I could be in a test-tube and everything that I think exists is but an illusion. But, I think that I can assume based on Occam's Razor that it is most likely true that I exist and am made out of atoms.

    Whether or not you're in a test tube is irrelevant. You would still exist; you would just be misinterpreting your sensory input.

    You still haven't answered the question you quoted. That is, "What are you from a point of view not bound to any universe (such as God's point of view)? What difference would it make if the answer was 'actual' instead of 'potential'? How would the potential thoughts and experiences of a potential Paul Doland differ in any way from the actual thoughts and experiences of an actual Paul Doland?" How would a potential Paul Doland's sense that he's made of atoms differ from an actual Paul Doland's sense that he's made of atoms. How would a potential Paul Doland in a potential test tube experience his false input differently from how an actual Paul Doland in an actual test tube would experience his false input? My claim is that if there's no difference between things except their name, then they're the same thing with two different names. I have yet to hear a difference between an "actual" universe and a "potential" one.

    You almost seem to be saying that simply because somebody in another universe cannot know about me, that makes me "theoretical" to them. But, I don't think that is true. There are billions of people in China that I'll never know about, but that doesn't make them "theoretical". So, I don't think I am "theoretical" to anybody that cannot know about me.

    You're using a much too restricted meaning of "know". Every person in China exerts gravitational attraction on your body. This is, of course, so small that you do not notice it, but it is an effect of their existence, and it would be different if they did not exist. There are lots of other effects of the Chinese population on your life; you would know before long if they all dissappeared. There is no similar effect, no matter how subtle, from the contents of another universe.

    So, if there is an omniscient God, I think He would know if He created me out of atoms or not. So, He would know if I'm real or not. That said, He would seem to have no motivation to create me out of atoms. If He knows everything I would do whether He actually created me or not, then what would be the point of bothering? So, I think your claims are excessively strong. Yet, the bottom line is, I think you are right that an Omniscient being would be without motivation to do anything.

    I'm not sure if it even makes sense for a being existing outside of time to have "motivation", but, in any case, for God to create you or not would just be a matter of switching labels on universes which, from their point of view, couldn't know and wouldn't care what those labels are.


    From: Paul Doland
    Date: Mon, 05 May 2003 19:46:07 -0800

    I read your response earlier today. I'm now at a bookstore Internet kiosk and its adult content filter is blocking your site, so I'll respond from what I remember you saying. If I misremember something you said, my apologies in advance.

    First off, in my "I'm a brain in a test-tube" analogy, I didn't express it properly. From your perspective, you cannot know for absolute certain that you yourself are not a brain in a test-tube and I am but false data imputs. Likewise from my perspective, I cannot be absolutely certain that you exist. However, I believe we both agree that it is reasonable to conclude that at least probabably[sic], reality is at least roughly what we understand it to be. And probably, you and I both exist in this universe. I presume we agree on this, do we not?

    It depends what you mean by "exist". If you mean "are a part of this universe", then yes. If you're talking about existence in some absolute, metauniversal sense, then it depends. If that means, "are a part of some possible universe", then yes. If that means, "are a part of some privileged set of possible universes which are fundamentally different from other possible universes" then no.

    Okay, moving on, as far as the people in China that I will never know, you indicate that they do indeed have some impact on me, even if it is imperceptable to me. And, given that I believe that human society is at least akin to a chaotic system, I agree. It is interesting that you mentioned that they have gravity that has a small effect. (At least I think I remember you mentioning gravity...) I found it interesting that you mentioned gravity because I'm reading Stephen Hawkings' book, The Universe in a Nutshell. One of the theories he's investigating, though he grants it is still very theoretical, but here's the theory in very brief: the theory is that of the 10 or 11 dimensions predicted in string theory, perhaps one of them, we'll call the fifth dimension, takes you to other universes. And, gravity propigates along the fifth dimension! So that we can feel gravitational effect of other universes!

    I don't know how much physics you have studied, so I do not know if you are familiar with the term "dark matter". In measuring gravitational effects on galaxies, it has been a long standing mystery as to why the gravitational effects that are measured seem to be far stronger than can be accounted by the matter that we see in the universe, the stars and the estimated planets and associated matter. So, it has been suspected that there must me some sort of "dark matter" that we simply cannot see and don't know what is, but exists. This much about "dark matter" is considered as well known. But, according to this theory of Hawkins, "dark matter" may in fact be matter that is not even in our universe!

    Now Hawkins theory may be right, or may be wrong. But, it could be true. It could be possible that we may someday be able to detect other universes by their gravity, or something else like wormholes. But even if not, you haven't as far as I can tell demonstrated that whether I can detect someone's existence in another universe or not makes them theoretical or real. If they are there, they are there, if they are not, they are not. And I don't feel that you have demonstrated that whether I can detect someone's existence or not impacts whether they are real or not.

    Well, of course it depends on what you mean by "real". If that means "is a part of the same universe", then detection is proof of existence.

    You said I did not answer your question, how would a real Paul Doland be any different than a theoretical Paul Doland. As I noted, I had been discussing your Atheist's Apology with a Christian acquantance, whom I have cc'ed. I think I agreed with him to at least some degree, in that I'd say that the difference between a real universe and a theoretical universe is at least different to the inhabitants of that universe. Theoretical universes only exist in the minds of those that are NOT in the theoretical universe.

    Again, it depends on what you mean by "exist". If you use it relatively then all universes exist relative to themselves, regardless of whether Paul Doland chooses to assign the label of "theoretical" to them. If you mean "exist" in some absolute term then my comments above regarding our existence apply equally well to the inhabitants of a "theoretical" universe.


    From: Paul Doland
    Date: Wed, 07 May 2003 09:22:06 -0800

    I re-read your response to my last response, and there are a couple of things I'd like to clarify.

    For one, you said that in the way that you are using the term "universe", anything that science can determine it exists is part of our universe. Well, for one, I don't think your definition jives with anybody else's definition. If Hawkings' theory that I discussed in my last e-mail gets proved to be true, then you want to consider everything in the 10 or 11 dimension space as part of our universe. If that is the case, then the Big Bang is not the start of our universe, but merely the start of our particular subregion of our the universe. And what if someday, a way to find other 10-dimension spaces is discovered, you'd want to grab them and say they are also part of our universe? So, I think that your terminology is not consistent with anybody else's.

    By "universe" I mean a maximal connected graph whose nodes are events and whose edges are cause-effect relations, using very broad definitions of "event" and "cause-effect". Philosophers and modal logicians often use the term "world" for this, but I think that's very misleading. I know my use of the word "univese"[sic] is not unique to me.

    So now, you asked how could a potential Paul Doland know that he is not made of atoms. Perhaps I don't understand your terminology. I don't understand how a potential Paul Doland could be thinking anything, so how could he be contemplating whether he was made of atom sor not? I, as a real Paul Doland, can comtemplate what some theoretical Paul Doland might do, but it is I that is doing the contemplation, not the potential Paul Doland.

    But wouldn't the potential Paul Doland potentially be contemplating? And if so, how would he (potentially) tell the difference between his potential contemplation and a real Paul Doland's real contemplation? If he couldn't tell the difference (and I don't see how he could) then how do you know you aren't him?

    If someone (let's call him Joe) were to contemplate what a potential Paul Doland might do, would he say, "the potential Paul Doland would potentially write to the potential Dennis Himes, 'So now, you asked ...'" or would he say, "the potential Paul Doland wouldn't potentially write anything to the potential Dennis Himes, because he doesn't exist." My guess is he'd say the former. In fact, if you ignore limits on information or processing power, Joe would describe your entire existence down to the minutest detail, throwing in a bunch of "potential"s and "potentially"s. This includes descriptions of what goes on in your mind. So my question remains, "how do you know you're real and not potential?" What could you say to me (or more importantly, think to yourself) that would be different when a real Paul Doland says or thinks it and when a Paul Doland hypothesized by Joe says or thinks it? Every time you've tried to identify a difference between real and potential it's come down to, "the difference between real and potential is that the former is real and the latter isn't." Well, yeah, but that's just begging the question. My position remains: If there's no difference between two things except their name, then they are the same thing with two different names.

    I do agree with you that God should know what I would do, whether He created me or not. And therefore, He seems to have no motivation to actually create me. But, nevertheless, I concede to the theist that it is at least conceivable that God would want us, the inhabitants of this universe, to have our experiences. Not very likely, but at least conceivable.


    From: Paul Doland
    Date: Sun, 11 May 2003 22:48:32 -0500

    (Note: Underlined italics is where he quotes my previous reply.)

    By "universe" I mean a maximal connected graph whose nodes are events and whose edges are cause-effect relations, using very broad definitions of "event" and "cause-effect". Philosophers and modal logicians often use the term "world" for this, but I think that's very misleading. I know my use of the word "univese"[sic] is not unique to me.

    I've never even heard of the term "modal logician" so I just did a quick search on the web. I confess that it is easy for the ignorant to say, "that sounds like a load of crap," but, well, it sounds like a load of crap...

    I'm not sure why you say that. It's just the formalization of modality. There are many people who could benefit from some rigor in their modal thinking; they would realize that they have to understand what they mean by existence before they can argue what does or doesn't have it.

    But, it does sound like maybe your theory and your definitions are consistent with other "modal logicians." But that seems to me to be of, um, limited usefulness. In other words, it looks like its all a load of crap. I think I'll stick with definitions physicists use. I completely discard your definitions as utterly worthless.

    When discussing physics the physicists' definition is undoubtedly more useful. However, when discussing actuality and possibility a definition of universe which includes detectable phenomena in another universe isn't very helpful, is it? I agree that the terminology is not standardized the way it could be, though. What if I use the term totality for what I've up to now been calling a "universe"?

    and potential it's come down to, "the difference between real and potential is that the former is real and the latter isn't." Well, yeah, but that's just begging the question. My position remains: If there's no difference between two things except their name, then they are the same thing with two different names.

    No, I've answered your question. The difference is theoretical Paul Doland's only exists in the mind of somebody in a real universe. That is at least one difference.

    Sigh. Once again I ask what you mean by "exist".

    Are you using it as a relative term to mean, "is a part of some totality"? If so, then the "theoretical" Doland exists in a "theoretical" totality and both are "theoretical" only relative to someone in some other totality but "actual" relative to themselves. This last statement is true for anyone in any possible totality.

    Are you using it as an absolute term to mean, "is a part of some privileged set of possible totalities"? If so, then what is the criterion for being so privileged? This seems to be how you are using it, but I can't be sure, because you never say.

    Or, are you using it in some other sense? If so, what?

    Joe would describe your entire existence down to the minutest detail, throwing in a bunch of "potential"s and "potentially"s. This includes descriptions of what goes on in your mind.

    I really don't care what "Joe" is capable of discerning about me--whatever he discerns is within Joe's mind. I guess I can't absolutely prove to you that what I'm thinking is in my mind and not in Joe's mind.

    You say that like "in my mind" and "in Joe's mind" are two different things. Since there is no difference whatsoever between what goes on in your mind and what Joe conceives goes on in your mind (again ignoring limits on information or processing power) they're the same thing with two different names. You have still yet to identify a single difference between a real Paul Doland and a theoretical Paul Doland. Every single thing you have written has been identical to both what a real Doland would really write and what a theoretical Doland would theoretically write. More to the point, every single thought you have had about the matter has been identical to both what a real Doland would really think and what a theoretical Doland would theoretically think.

    And therefore my mind is real and not theoretical. But then again, you can't prove to me that anything you've said is anything more than a load of crap...

    I have claimed that there is no difference between two different states except their name. If that's a load of crap then I would think that you would be able to identify a difference. You, on the other hand, have repeatedly made arguments of the form, "the difference between something which is real and something which isn't is that the former is real and the latter isn't", which, frankly, doesn't have enough information content to rise even to the level of a load of crap.


    From: Paul Doland
    Date: Thu, 15 May 2003 23:02:18 -0500

    (Note: Underlined italics is where he quotes my previous reply.)

    Sigh. Once again I ask what you mean by "exist".

    I can play that game too. What does "theoretical" mean? What does "potential" mean?

    Good questions. I've been using theoretical and potential pretty much interchangably, to mean "not real". Of course, that means that they can be used in either a relative or absolute sense, depending on what meaning of real they're being contrasted against. I've tried to make it clear which variation I use when, and am happy to clear up any ambiguities.

    What does "totality" mean?

    I explained that above: "a maximal connected graph whose nodes are events and whose edges are cause-effect relations".

    Some words are indeed hard to define.

    That's no excuse for refusing the attempt.

    But if you think I haven't answered what "exist" means, I think you haven't answered what any of those others mean.

    But, unlike you, I'm willing to try. I do not simply say "something is theoretical if it's theoretical", and pretend I've defined the term.

    However, when discussing actuality and possibility a definition of universe which includes detectable phenomena in another universe isn't very helpful, is it?

    Oh, but what does "actuality" mean?

    The state of being real. Of course, that means that the same ambiguities arising from the relative/absolute state of reality apply to actuality. my claim all along has been that an absolute reality is inherently arbitrary, essentially just assigning a meaningless label.

    What does "possibility" mean?

    The state of being an internally consistent totality, or part of one.

    See how much fun this is!

    It would be, if you were truly interested in understanding other people's thinking.

    What does "what" mean?

    It's the English neuter interrogative pronoun.

    Bottom line, no, you haven't demonstrated to my satisfaction any value to your "modal logician" definitions. Admittedly, I do speak from ignorance as I haven't studied the subject, but you've convinced me the subject must cause some sort of brain damage, so I don't believe I want to.

    There are several approaches one might take when another's use of terminology isn't clear. One is to ask and answer questions in order to try to establish such clarity. Another is to refuse to answer even simple questions like, "do you mean 'exist' in a relative or absolute sense?" and instead spew insults like "load of crap" and "brain damage". I have chosen the former approach. You have chosen the latter. This is unfortunate, because when we first started corresponding you seemed genuinely interested in an exchange of ideas.

    In my computer science studies, I took only one philosophy class, logic. In that class, it was a more abstract version of the types of things computer programmers do, if a and (a then b) then b. I'm afraid I don't remember the symbols used, but I'm sure you know what I'm talking about. That particular class was quite a good class, and I did get an A in it. But we never discussed anything like what you are discussing--fortunately. The simple reason I bring this up is simply to show that I do have reasonable abilities to do logical thinking. Your theories are just illogical.


    From: Paul Doland
    Date: Mon, 19 May 2003 22:34:32 -0500

    (Note: Underlined italics is where he quotes my previous reply.)

    There are several approaches one might take when another's use of terminology isn't clear. One is to ask and answer questions in order to try to establish such clarity. Another is to refuse to answer even simple questions like, "do you mean 'exist' in a relative or absolute sense?" and instead spew insults like "load of crap" and "brain damage". I have chosen the former approach. You have chosen the latter. This is unfortunate, because when we first started corresponding you seemed genuinely interested in an exchange of ideas.

    I confess to getting snotty. I was frustrated at your seeming insistence on things illogical, yet your definitions make it impossible to prove. Or, perhaps you are right, I don't understand your definitions. So, I will try to not be snotty this time.

    As far as your definition of universe, or totality... I had mentioned that Hawkins has a theory that perhaps a fifth dimension (not the singing group...) might take you to other universes. Actually, he seems to now prefer a term p-brane (no, not pea-brain...) where 'p' is the number of relatively flat, usable dimensions. Though gravity warps our dimensions, we have three relatively flat, usable dimensions, plus time. I don't know whether that makes our "universe" a 3-brane or a 4-brane, but I think 3. Anyway, he says it is conceivable that there are "universes" or dimension-spaces that have more or less than our 3.

    I'm getting off track--what I'm getting to is that generally, when I've thought about other universes, I haven't given a lot of thought as to whether we could even in theory reach them or not. But I guess I've thought that maybe some day sufficient technology would allow us to travel to other universes. But I suppose that there could be both universes that are reachable, at least in theory, with sufficient technology, and there could be other universes or realms that are fundamentally unreachable, no matter how much technology we develop. For example, I would expect a Christian would likely figure that heaven is such a realm--that no matter how much technology man ever creates, man will never be able to send an astronaut to heaven.

    Technology has nothing to do with it. A Christian believes there is a cause and effect relation between a person's life on Earth and his afterlife in Heaven, and therefore Earth and Heaven would be part of the same totality.

    For the discussion of the remainder of your email I'm resigning myself to the fact that you are never ever going to answer my question about whether you're using the term "exist" (and related terms) in a relative or an absolute sense. Therefore, I'm splitting my responses into two tracks, one assuming that you're using it relatively, and one assuming that you're using it absolutely. Just ignore the one which doesn't apply.

    relative - To exist relative to a totality means to be part of that totality. absolute - Something either exists or it doesn't regardless of the point of view of the one considering its existence, according to some mysterious criterion
    I assume that if two things are members of the same totality and one exists then the other one does also, or else I'm completely confused about what you mean.
    So, so far, my understand of your use of totality or universe would mean that any place that could at least in theory be reached by science, such as perhaps some other of Hawkins' p-branes, then that is part of our universe or totality. If it is a place fundamentally unreachable, then it is not. The problem is, how can I possible prove anything about something fundamentally unreachable? How can I possibly prove it is real or theoretical?
    It's real relative to itself and not real relative to you, just as you are real relative to yourself and not real relative to someone in another totality. There's not much to prove here. As I said in my essay, "To say that a universe exists relative to itself is simply a tautology and as little subject to causation as any other tautology." That depends, what is your criterion for declaring something "real" or "theoretical"? You're the one who seems to think that there's a fundamental difference between the two. If there is, then you should be able to tell me how to prove it's real or theoretical. As for me, I've been claiming that there is no fundamental difference. It's my claim that you can't possibly prove that it is real or theoretical in any way which doesn't simply reduce to assigning an arbitrary label.
    Your question is like, "prove undetectable angels don't exist". How can I possibly ever do that? So, therefore I find your definitions, at least as I understand them, to be of zero value, like undetectable angels.
    They have value when discussing the possibility of a creator god. They point out the fact that this totality, like any, exists relative to itself by definition, and to talk about causing its existence doesn't even make sense. They have value when discussing the possibility of a creator god. They point out the fact that making this totality "exist" or "not exist" is just assigning a label to it, no different from making this totality "foo" or "bar", and it makes no difference to us whether or not this assignment has or has not been done.


    From: Paul Doland
    Date: Tue, 1 Jul 2003 15:28:54 -0500

    I guess I still don't understand what a "theoretical universe" is, at least from your perspective. For example, "I speculate there is a universe that is similar to this one, except Paul Doland is king of the world." Is that a "theoretical universe"? If so, is there now a theoretical Paul Doland, king of the world in his theoretical universe, going about his business as king of the world totally unaware that he is but a theoretical Paul Doland? If this isn't a "theoretical universe" what would be one?

    The answer partly depends on whether "a universe that is similar to this one, except Paul Doland is king of the world" is a coherent description. That is, whether or not it describes an internally consistent set of conditions. I suspect the answer depends on how strictly you define "Paul Doland". If your use that phrase to mean someone with all of the properties which you possess, then the answer is "no", since one of your properties is a lack of the quality of being king of the world. If, on the other hand, that phrase means anyone who happens to bear the same name as you, then the answer is probably "yes". If it's somewhere in between I'd guess (but am not sure) that the answer is "yes". Let's assume that it is "yes", i.e. that "a universe where Paul Doland is king of the world" is a coherent description.

    Since you still haven't told me whether you use "exist" and related terms in a relative or an absolute sense I'm going to have to split my answer again.

    relative - To exist relative to a totality means to be part of that totality. absolute - Something either exists or it doesn't regardless of the point of view of the one considering its existence, according to some mysterious criterion
    The universe where you're king is theoretical with respect to ours, and our universe is theoretical with respect to it. This holds for any pair of universes. What I wrote last time applies to this question equally well (with a little editing):

    That depends, what is your criterion for declaring something "real" or "theoretical"? You're the one who seems to think that there's a fundamental difference between the two. If there is, then you should be able to tell me how to tell whether it's real or theoretical. As for me, I've been claiming that there is no fundamental difference. It's my claim that you can't possibly distinguish between real or theoretical in any way which doesn't simply reduce to assigning an arbitrary label.

    I'd also like to note that the phrase "is there now" in your question implies that your speculation somehow brings the alternate totality into being. This is clearly false. By the definition of totality you can't affect it, and creation is certainly an affect. If it's coherent it exists relative to itself regardless of whether or not you're aware of it. Besides, its time dimension is independent of ours, so it doesn't even make sense to talk about its state or condition "now".


    From: Paul Doland
    Date: Wed, 10 Sep 2003 17:45:24 -0800

    (Note: Underlined italics is where he quotes my previous reply.)

    In your response before last, you said:

    "For the discussion of the remainder of your email I'm resigning myself to the fact that you are never ever going to answer my question about whether you're using the term "exist" (and related terms) in a relative or an absolute sense."

    Though I concede that I have yet to figure out how to define "exist" in an absolute sense, I think I had made it clear that I think there is a difference between the theoretical/relative and the real/absolute. As near as I can see, you seem to say that since I can't prove you wrong, there must be no difference. From my perspective, I haven't seen any evidence for your side other than my lack of evidence. So, it seems that both of us are saying it is the other person's burden to prove their position. But, I'd say that since you are offering your Atheist's Apology as an opinion piece, I think you have more the burden to prove you are correct if I am to take your opinion piece seriously.

    Well, since this is just a discussion on a webpage neither of us really has a burden. However, I believe that by the very nature of the argument someone claiming the existence of something has a greater burden of proof than someone claiming nonexistence. Even if not, surely the person claiming existence has a burden of saying what it is he claims exists. Your argument seems to be along the lines of, "I claim that there is a difference between a real universe (totality, or whatever) and a theoretical one. I can't say what that difference is, I can't describe the effects of that difference, I can't identify any properties of that difference, but the burden is on you to demonstrate that it doesn't exist."

    As to my "Paul Doland as king of the world theoretical universe" question you replied:

    "If, on the other hand, that phrase means anyone who happens to bear the same name as you, then the answer is probably 'yes'."

    I note that you say "probably". Why? Well, because you can't know. Really, neither of can know for certain who is right. Neither of us seems to have a way to provide positive evidence for our position, all we can provide is a lack of positive evidence on the other side.

    I think you misunderstood what that "probably" was referring to. If you look back at where I used it I was referring to the possibility that someone with the same name as you being king of the world "describes an internally consistent set of conditions." In other words, is it even a theoretical possibilily (leaving aside for the moment whether that entails an fundamental difference from a real world)?

    But, I think that I can think of at least one piece of positive evidence on my side, which I shall now provide:

    This morning, I went to work. I could have decided to call in sick. To be honest, every once in a while, I'll decide to take a day off and call in sick. There is nothing fundamentally inconsistent with my character that would have prevented me from calling in sick. But, if there are two theoretical universes that are exactly the same up to the point where I decided to go to work, if everything was identical up to that point, I surely would have made the identical decision. So, while there is nothing fundamentally inconsistent with the idea of a universe identical with this one up to the point of my decision about going to work yet coming up with the opposite decision, I don't see how it could be.

    You have to make up your mind. Could it have happened or couldn't it? If A and B could not both happen then, ipso facto, A and B are fundamentally inconsistent. Here A is the history of the universe up to that morning, and B is calling in sick. Whether or not branches like this are possible gets into the whole question of free will, but whichever way you resolve that question, it doesn't affect my position.

    So, unless you can come up with some positive evidence for your side, instead of merely a lack of positive evidence on my side, then I think I'll maintain my position.

    Well, the evidence I've given seems more than adequate to me, but if it doesn't to you then I guess we're just going to remain in disagreement.


    From: Anonymous O
    Date: Fri, 12 Sep 2003 08:44:39 +0200

    I have some objections to 'The Contradiction of Omnipotent Agency '.

    This reasoning has at least two weak points, two groundless assumptions:

    1. God is absolutely passive
    2. it is somehow defined, what is theoretically possible

    "God is absolutely passive" is not an assumption of the essay. If anything, it's a conclusion, although it would be more accurate to describe it as an intermediate conclusion on the way to the more general conclusion that there is no God.

    So I want to ask:
    How it is defined (or, who is defining) what is theoretically possible?
    Can God change, what is theoretically possible and what is not?

    I believe existence is a prerequisite for changing, so I would say no, God cannot change.

    I'm using "theoretically possible" to mean "internally consistent". I gave an example of a universe which is not theoretically possible in an earlier commentary.


    From: Anonymous P
    Date: Mon, 15 Sep 2003 16:51:14 -0400

    I am writing because I have a desire to discuss the my ideas of God and creation with someone who may be interested in the subject with an objective point a view. If you have the time, it would be great to read your opinions on what I have to say, I do request that my identity remain anonymous.

    A little introduction on why I find this topic interesting.

    When I was younger I had to have surgery where I was put under with anesthesia. The thing that always struck me was how at one instant you are slowly counting backwards, and the next instant you are being woken up a couple hours later with absolutely no knowledge of the preceding operation. Zippo...nothing.

    I went through that too, when I had some teeth pulled when I was about 12. It was freaky how my time sense told me that no time had passed; it was very different from going to sleep and waking up. Not that this is really relative to the discussion at hand, but I thought I'd mention it.

    It donned [sic] on me a few years ago about whether or not that is what death was like, a complete and total blackout, no sensation and no consciousness, just a complete and total nothingness that you aren't even aware of.

    As I thought about it, I came to the conclusion that such a concept of death is frightening, not because of the possibility of total annihilation of being and self, but because, if it were true, then all this weight we put on God and the afterlife is for naught, and there will never be anyone who can come back and tell us how stupid we are.

    In the meantime I began exploring my own arguments about God, creation, and the afterlife. Here are some thoughts I would like to share.

    PROOF AND DISPROOF OF GOD'S EXISTENCE

    It has occurred to me that the proof or disproof of God is rather like Schrödinger's Cat. Just as we can't discern whether the cat is alive or dead until we open the box and see for ourselves, there appears to be no way of knowing whether God exists or not until we die. At least, there is no way of proving or disproving God's existence. Now some clever individuals point to the universe and say that's all the proof you need that God exists, but that is based on the assumption that the universe needs a God to exist, which then begs the question whether we can prove or disprove that the universe needs a God in order to exist (and I think that might be an argument that can be won by an atheist...but I'll save that thought for another day)

    Of course, the point of my essay was that I think the existence of God can be determined, in the negative, without having to die.

    Anyhow, using the example of Schrödinger's Cat, we can say that perhaps, based on our current knowledge, God both exists and does not exist.

    Now, if we accept this "duality" about God, there is another set of questions

    1. Is there an afterlife?
    2. Is there an afterlife if there is no God?
    3. Does there need to be a God in order to have an afterlife?
    4. Can there be a God and no afterlife? finally,
    5. Is there evidence to support that we will know that God exists or not even after we die?

    I ask those questions to prove a point...even if we die and we find out there is an afterlife, the existence of that afterlife still doesn't dictate that there is a God. It is entirely possible that we can die, go to "heaven" and still be utterly clueless as to whether God exists or not.

    This is a good point, one which is often missed.

    One thing about the Schrödinger's Cat argument, if we take the argument at face value, it might lead us to the conclusion that there are universes where there is a God and universes where there isn't. This of course puts us in the situation where we have to accept more than one universe. Now we have to wonder if there is a being higher than God who is in charge of all the universes (or the "multiverse" as current physics would say). This leads to "layered" approach that got the Gnostics in trouble back in the day and brings me back to question 5.

    The bottom line is...we can not conceive what God is, and there is no evidence to support that we can ever know God. Some may argue that God is an entity that is all powerful, all seeing and all knowing, however, that ultimately is a human conception, and it is entirely possible that God sees nothing, knows nothing and is powerless. The powerless God is actually closer to what we actually have observed, and he is a God that knows only one thing, that he IS. Maybe some ancient philosopher thought of this when he said that God's name is I AM. Descartes may have stumbled on it again when he said "I think, therefore, I am." The only true conclusion is that any human who says he knows God and God's will is lying.

    You seem to be redefining God so that it no longer corresponds to the English word God. If you continue along that path saying "God exists," or "God may exist" becomes misleading at best and meaningless at worst.

    SOME THOUGHTS ON THE AFTERLIFE

    Leaving the debate about God's existence behind us for a moment, if there is an afterlife, I wonder if it is one of Free-will or Determinism. It is just a question, and I am not sure how to approach it. If there is Free-will than it leads me to believe that there is absolutely nothing different between our world and the afterlife, other than the fact that we have no physical form. If it is deterministic, then what's the point?

    The point of what? You seem to be assuming that we have free will in this life, but if we only had the appearance of free will would that give it any more or less of a point?

    I also have wondered why we should assume that our lives are more valuable than an ant's? What is truly so special about us that we should assume that we have a place in the "afterlife" and an ant doesn't? If you think about it, all the wonderful things we need to do in order for us too reach heaven, an ant does on a daily basis without any discernable complaint. Is not the ant more deserving of heaven? If one argues that we are better because we have a sense of self, I will argue that no one has ever asked an ant if it does. We just assume the ant is inferior, and refuse to accept that we really should have no expectation of what will happen to us after we die.

    CLOSING THOUGHTS

    One of the things that has gotten me upset in recent months, is the phrase, "We should fear God." And I ask, "Why should we fear God? Can we fear God?"

    I am reminded that one idiot in Germany started WW II by maintaining that FEAR is the source of true power. So I ask, "Should I fear God, like we feared Hitler?" I have a hard time accepting that I have to live in Fear in order to go to heaven. Perhaps we should fear "Satan" but not God. I would think that if you live in Fear of your master, than your master is probably not God...he is probably the other guy. As Yoda so eloquently stated, "Fear leads to anger, anger leads to hate, and hate leads to the Dark Side."

    Here you seem to be assuming that God must be benevolent. Why?

    Thank you for your time, and I look forward to any feedback


    From: Anonymous P
    Date: Wed, 24 Sep 2003 15:09:27 -0400

    In a math book I once read how one can derive all numbers from zero. Paraphrasing, the argument goes like this: If you define zero as being a set where the value is null, you can say there exists ONE set which includes all sets with a null value. You now have a set of TWO numbers, ONE and ZERO, to describe this set you need THREE numbers, ZERO, ONE and TWO, which now creates FOUR numbers, etc. This process can go on to infinity, and as a result also creates the prime numbers, rational and irrational numbers, decimals, and geometry; which leads to calculus and the whole knowledge of mathematics.

    Specifically it produces the ordinal numbers, and not only goes on to infinity, but on to a infinite number of infinities.

    This leads me to a question of causality.

    A simple way some describe God, is that God is a being which exists without cause, and gives cause to everything else.

    I went to the referenced website to see the arguments for God http://www.angelfire.com/tx/eclectorium/ramblings/avicenna.html

    It occurred to me that the arguments were actually supporting why God is not necessary to the universe, and not supporting the existence of God as claimed.

    My logic is:

    "Nothing" has a definition (i.e. it is a concept which has meaning)

    A definition is "something"

    "Nothing" can only be defined if it has a relationship to "something"

    "Nothing" can not exist alone

    "Nothing" has given a cause for "something"

    This can occur without God

    Looking at the above a theist may argue that it was God which gave "nothing" a definition. However, this falls apart with the following logic:

    For God to define "nothing" he must think

    Thinking or "thought" has a definition

    "Something" must exist to provide definition

    "Something" is a priori to God

    Then "something" existed before God

    God is then internal to the universe and did not create it

    I want to point out that the above is not necessarily disproving God's existence or that he created man, it is just disproving that God is the creator of the Universe.

    Even though you're basically supporting my position, I don't agree with this analysis. You're confusing pronouns with nouns with words. "'Nothing' has a definition" makes sense and is true if you're using the word nothing and makes sense but is not true if you're using "nothing" as a pronoun.. You seem to be employing neither use though, and treating "nothing" as a noun, which it isn't. I'm reminded of Lewis Carroll's Alice's Adventures in Wonderland when the White King asks his servant whom the servant passed on the road. When the servant says, "nobody", the king says, "So of course Nobody walks slower than you."

    One argument against my logic is that "nothing" can't be defined. This of course is a definition in itself, and leads me back to the above argument.

    It may be that humans can not conceive of "nothing" (as my high school physics teacher often professed), but one can argue that in order for there to be "something", than "nothing" must also exist. Or rather "something" is the absence of "nothing", which if true requires that "nothing" be defined.

    So one may conclude that "nothing" in the absence of "something" is meaningless.

    My college philosophy/theology professor often discussed that the universe consisted of what IS and what IS NOT. God was what IS. However, in order to have any meaning IS and IS NOT both need to exist, they are the cause of each other. Again, in order for God to be a "being" he must think (COGNITO ERGO SUM) to have thought than the IS must have existed before God.

    Now another sticky point about this, and causality, is the concept of Time and Space. Without the flow of Time, causality is meaningless. Without Time all things that "are" exist simultaneously, without Space they possess no physical form.

    This may be difficult to comprehend, but in order to say A caused B you must be able to differentiate the two. Without space or time, it can not be done (its interesting to note that physics has not ruled out that the future influences the past in some way, this does not necessarily disprove causality, it just means that in some processes we have the arrows reversed)

    A theist may now say "A-ha! we have caught you, if causality before time is meaningless, than how can you prove that "something" existed before God and not the other way around?"

    Why would the Theist assume it has to be one or the other?

    Here is where I get back to the argument of deriving all the known numbers from ZERO. This derivation also creates time in that it creates causality. Notice that in order for a higher number to be created, there must be a lower number before it, so the lower number is the cause of the higher number.

    So lower numbers cause higher numbers and, if you order things based on cause, you have causality. Causality implies time, time implies motion, motion implies space.

    The implication is that, following my earlier arguments, if God exists, his cause was after the development of time and thus he again is internal to the universe. (one may also add that "thought" implies the a priori existence of time. "Thought" is defined, which implies causality by previous argument, and again causality implies time)

    "But what about negative numbers and imaginary numbers" This question I posed to friend once (who is a physics professor) and I always liked his response. "We are trained to think in a linear fashion, so if you look at linear motion, the presence of imaginary numbers seems odd, however if you think of circular motion, imaginary numbers appear as a natural consequence." And once you have an imaginary number, you have the square root of negative one, once you have negative one, you have the rest of the negative numbers.

    To finish this up I want to add that a little about why I said that the referenced web page seemed to support why God is not necessary to the universe.

    Most of the arguments seem to argue that there was something without cause (or a "necessary being") that caused the rest of the universe, the refer to this entity as God. What I really think they are doing is proving that the universe can come to be without God, that its existence is self supporting, and the universe is its own cause.

    So when one says "Nothing is greater than God." It can be taken literally in that "Nothing" was the source of creation, and therefore "Nothing" created whatever God may exist, and thus being the cause of God it is in fact greater.

    See my above comments on the White King.

    Another thought is, applying Occam's Razor, if the universe is the cause of its own existence, why do we need a God? If the universe can create itself, and we assert that nature is efficient, than it would appear that creating a God is inefficient for the Universe. So to say there is a God, then we must say that the universe is inefficient, if it is inefficient, it is not perfect. If it is not perfect, and God is internal to the universe, than God is imperfect. If God is imperfect than he is not the God we think of or, he does not exist.

    One last thing, it is entirely possible to say God exists. Humans arrived at the idea of God, this idea has obviously been shared widely, and although individual conceptions vary, there is still a succinct shared belief of God in many. So he does exist, in the minds of many. It can be argued that this existence is just as real and powerful as a physical God. The concept and belief certainly has power in the way it influences individuals, and prayer certainly works in how it appeals to the inner psyche and to people around you. So humans have created God, and God has created men, for we have defined a man based on the ideals of what God is (which is a human distinction to put us in a class above animals).

    It could only be said that "this existence is just as real and powerful as a physical God" if one doesn't stop for a second and consider how real and powerful a physical god is assumed to be by those who believe in one.

    Without man, however this God does not exist, as far as we know....the question to ask is whether Dinosaurs had Gods.

    Can God create a stone so heavy he can not lift it, and then lift the stone. Certainly, I have encountered stones I can not lift, but I can imagine that stone in my mind and see myself lifting it. That mental reality is just as real as my physical reality.

    I disagree. Changing the physical position of the stone is a real effect which actual lifting achieves but mental lifting doesn't.


    From: Anonymous O
    Date: Mon, 29 Sep 2003 11:19:36 +0200

    You said: 'I believe existence is a prerequisite for changing, so I would say no, God cannot change.'

    It is enough for me, because it is the proof that you have simply exchanged belief to God with belief to the assumtion [sic] that 'existence is a prerequisite for changing'.

    For me it proofs [sic] again, that there cannot be any proof (in absolute meaning) either for, or againts [sic] God existence. That it is always only some kind of belief.

    You seem to be assuming that all beliefs are essentially equal. Would you consider belief in unicorns to be equal to the belief that one plus one equals two? To treat belief in God, for which there is no evidence, as equivalent to belief that the word change means a difference in the state of something at different times, is a poorly disguised attempt to hide the weakness in your claims by announcing that all claims have equal weight simply by decree.

    I think, that a part of God's omnipotency is also ability to change, what is internaly [sic] consistent (theoretically possible) and what is not.

    That doesn't make any sense. Internal (in)consistency is an inherent property of a system. It's not a matter of adding or removing a contradiction as if the contradiction was some external or additional property. The statements:

    1. The number of protons never changes.
    2. At time = 0 the number of protons is 10^40.
    3. At time = 1 the number of protons is 10^39.
    are by their very nature inconsistent. To say that God could make them consistent is nonsense.


    From: Anonymous P
    Date: Mon, 29 Sep 2003 10:27:31 -0400

    (Note: Underlined italics is where he quotes my previous reply.)

    Response to web post questions, in no particular order

    Here you seem to be assuming that God must be benevolent. Why?

    I have to agree this is a good point, the short answer is that I'm personally predisposed to think that God, if he were to exist, is benevolent. Perhaps this is due to a Catholic upbringing (which isn't necessarily grounds to believe in a benevolent God...but I'm sure that was mentioned at least once). However, putting that aside, I guess I would be more inclined to think of him as benevolent on grounds that most people have chosen to differentiate between Good and Evil. Most people also seem to associate Good with God, and have also assumed, since the advent of the new testament, that God forgives sins and that this indicates benevolence.

    Of course, this brings up the subtle irony of debating the attributes of an entity no one can prove even exists. I find it ironic that someone earlier brought up Middle Earth and the Hobbit when discussing God, ironic in that major discussions about the nature of non-existant [sic] entities is generally termed fiction. So it isn't a stretch to say discussions about God and his character is the ultimate fiction.

    So, I guess your point is well taken. It seems ridiculous to make assertions and assumptions about the nature of something that has never been observed or had identifiable interactions with humans or their environment.

    The point of what? You seem to be assuming that we have free will in this life, but if we only had the appearance of free will would that give it any more or less of a point?

    Hmmm...I guess I should couch my assertions more carefully. I recognize the debate of Free-will versus determinism is still ultimately undecided. I inadverently applied my personal philosophy without properly explaining it. I guess my philosophy revolves around the belief that my being has some value (if not to "god" than [sic] to me personally), if the afterlife is deterministic, than [sic] from my standpoint, my being and what makes me valuable is valueless (which might be the case).

    I have been created with Free-will, or at least led to believe I have Free-will, it is an intrinsic quality in what I am. If my being becomes deterministic (or loses the illusion of Free-will) in the afterlife, than what exists in the afterlife is inherently not me in life. Than from my perspective, a deterministic afterlife is just the same as there being no afterlife, because some intrinsic qualities of what I am are lost in the transition, and whatever ends up on the "other side" isn't me, but some fascimile of me.

    Of course, if this life is also deterministic then a deterministic afterlife involves no such loss.

    I'm not sure if that answers the question, and the ideas behind my rebuttal may need some more development, but I think there is enough there to start getting the jist [sic] of what I meant when I said a Determinstic afterlife is pointless.

    I've heard many people claim that discovering that free will is illusionary and that reality is deterministic would make their lives pointless, and, frankly, I've never understood that position. Conditional statements such as, "If I show kindness to this person, his happiness will increase" are true (or not) regardless of whether someone could in theory predict whether you will in fact show that kindness. And that's all determinism is, the supposition that it is theoretically possible to predict everything that will happen.

    You seem to be redefining God so that it no longer corresponds to the English word God. If you continue along that path saying "God exists," or "God may exist" becomes misleading at best and meaningless at worst.

    I admit to some sloppy reasoning in this part of my email, and after I sent it, I really thought it exposed some flaws in logic and should have be more carefully stated. The point I was trying to reach at was that trying to define an entity that is at best hypothetical is impossible. Without concrete observations, or without any observations as the case is, the definition of what God is seems meaningless.

    I still stand by the possibility that the only true statement that can be made with current knowledge is that God may or may not exist. It appears that we can not answer the question of God's existance [sic] because we can not identify the nature of God.

    The word God becomes meaningless if you cannot identify at least some aspects of its nature. When such aspects are identified, even if only conditionally (as in "if God is like this, then that"), then the question of existence can be addressed.

    Since the nature of God at present is largely subjective and personal, and without any way of observing God, than the best we can say is that in some cases he does exist and in others he doesn't. (applying modern interpretations of physics...that it is our observation of an event that makes it real...ie the act of observation/interaction is central to making what something is.)

    There are limits to that principle. Schrödinger's cat might be both dead or alive before the box is opened, but even without opening the box we know it isn't a dog.

    Of course, the point of my essay was that I think the existence of God can be determined, in the negative, without having to die.

    I agree that this may be done, my example of Schrodinger's cat is purely an illustration of the limits of scientific theory as it stands today. One thing I'm exploring right now is understanding the nature of logical contradictions and what that might tell us about the universe. I think that many have ignored the nature of the illogical, and a deeper exploration of what the illogical is may help. I haven't gathered my thoughts on this yet, but when I do I certainly will send them.

    thanks


    From: Anonymous P
    Date: Tue, 14 Oct 2003 13:55:11 -0400

    (Note: Underlined italics is where he quotes my previous reply.)

    greetings again

    thanks for posting my emails

    In response to your comments I submit the following

    In your critique you say

    Even though you're basically supporting my position, I don't agree with this analysis. You're confusing pronouns with nouns with words. "'Nothing' has a definition" makes sense and is true if you're using the word nothing and makes sense but is not true if you're using "nothing" as a pronoun.. You seem to be employing neither use though, and treating "nothing" as a noun, which it isn't. I'm reminded of Lewis Carroll's Alice's Adventures in Wonderland when the White King asks his servant whom the servant passed on the road. When the servant says, "nobody", the king says, "So of course Nobody walks slower than you."

    Now I think there is a disconnect on how we treat the word "nothing", I am not sure where I am causing confusion, but I will use the dictionary as the basis of how I define nothing:

    >From Webster's II New Riverside Dictionary

    nothing - noun. 1.Not anything. 2. No portion or part. 3. One of no consequence or interest. 4. Absence of anything perceptible.; -noun 1. One having no existance [sic]. 2. One having no quantitative value: zero. 3a. Something trivial: trifle. 3b. An inconsequential person: nonentity.; -adverb. In no way or degree.

    The difference between nouns and pronouns is not always clear, but I would consider 1 and 2 (of the first part) to be definitions of nothing as a pronoun. Definition 3 (which is basically equivalent to the entire second set of noun definitions) I admit I didn't think of, but I don't think it's what you meant, anyway. Definition 4 is "nothingness" in my dialect.

    I am confused when you say, "You seem to be employing neither use though, and treating "nothing" as a noun, which it isn't." Which I can't quite discern what you are saying, so on this point I have to beg for some further clarification before making a complete rebuttal. Or rather, I need to know how you are defining nothing.

    Now just for fun here is the definition of nobody from Webster's

    nobody - pronoun. No person: not anyone.; noun., plural - nobodies. One of no significance or influence

    I think I understand the argument about "confusing pronouns with nouns with words" as meaning that a word is an entity in its own right. However, what I am doing is using the word "nothing" to represent a conception. There is a concept of a some null valued state, and in general we use the word "nothing" to describe it.

    There is a contextual basis for how we use words. So confusion can erupt if the meaning and context are misused. So where in your example of Alice and Wonderland the servant replies "nobody", he means that there was no entity that qualified as a person (or other creature) on the road that passed him. The king paradoxically chooses to view "Nobody" as a real entity representing a person (or other creature) when he produces his statement.

    Now if I can re-use my previous argument with different words, perhaps I can make my meaning clearer:

    There is awareness of a state that is empty of objects

    For this state to be perceived there must be another state, ouside [sic] the first state, that contains objects

    I'm not sure in what sense you mean "outside", but I don't think it's important for your argument.

    To recognize an empty state there needs to be a non-empty state for comparison

    I don't think it's necessary for comparison, so much as it's necessary because there must be something doing the recognition.

    The empty state is not perceptable [sic] on its own

    So the perception of the empty state gives cause to the non-empty state

    I don't follow this at all. First you show that a non-empty state is a necessary condition for recognition of an empty state, and you seen to conclude from this that an empty state is a necessary condition for the existence of a non-empty state. Either I'm missing something or you skipped a number of steps in your argument.

    I'm going to leave this discussion where it is for now and move on to the next topic

    Why would the Theist assume it has to be one or the other?

    I apologize, I guess I meant to use that as a rhetorical question, I wasn't necessarily trying to imply that theists would think one way or the other, it was just a question I thought pertinent to my argument

    See my above comments on the White King.

    I think I understand your point, but I am literally supporting the notion that the universe sprang forth from what some define as "nothing", and God, should he/she exist, would also come from "nothing", and that if God did come down to earth and say "Nothing is greater than I" than we should take it as a literal statement and not a boast of power.

    But "nothing" is not something, it is the lack of something. How does springing forth from nothing (or "nothingness") differ from not springing forth from anything? This still seems to me to be the White King's error on a more subtle level. You seem to be assuming that if a state can be described then there is something attaining that state. This is actually somewhat similar to the claims about relative existence I've been making in my remarks on this page. A universe which contains nothing is certainly consistent, but it is a universe without cause or effect, because cause and effect imply both the existence of causes and the existence of time, neither of which is compatible with the state of nothingness.

    It could only be said that "this existence is just as real and powerful as a physical God" if one doesn't stop for a second and consider how real and powerful a physical god is assumed to be by those who believe in one.

    I concur, however, I want to leave room for later arguments that the advent of the internet brings to life some interesting philisophical implications about how much of a physical existance [sic] is required in order to be "real and powerful". more on that later.

    I disagree. Changing the physical position of the stone is a real effect which actual lifting achieves but mental lifting doesn't

    You may be right, but I haven't convinced myself that changing the rock's physical position is in fact a real effect. I am still trying to figure out how I would explain to God what real is, let alone what a rock is.

    I don't think "real" is as straightforward a concept as it's usually taken to be, but it seems that whatever meaning is assigned to it would include physical position.

    Of course, if this life is also deterministic then a deterministic afterlife involves no such loss. I've heard many people claim that discovering that free will is illusionary and that reality is deterministic would make their lives pointless, and, frankly, I've never understood that position. Conditional statements such as, "If I show kindness to this person, his happiness will increase" are true (or not) regardless of whether someone could in theory predict whether you will in fact show that kindness. And that's all determinism is, the supposition that it is theoretically possible to predict everything that will happen.

    I think it all depends on whether one places a value to the ability to make one's choices. I agree that determinism may not make your life pointless, but perhaps it would feel belittling to some that what little power they thought they had, wasn't really theirs.

    The word God becomes meaningless if you cannot identify at least some aspects of its nature. When such aspects are identified, even if only conditionally (as in "if God is like this, then that"), then the question of existence can be addressed.

    There are limits to that principle. Schrödinger's cat might be both dead or alive before the box is opened, but even without opening the box we know it isn't a dog.

    I think you are right. My issue has always been that because there is limits to human knowledge in the near-term, there are always "gaps" were people can place God. If I define God as possessing "only traits that are untestable by man" than I have put forth a definition of God that prevents me from proving or disproving its validity. I always like the argument that if God exists, why doesn't he make his presence known? It is one that I never have seen answered to my satisfaction.

    well thank you again, and I hope to read your replies soon!!


    From: Anonymous P
    Date: Tue, 28 Oct 2003 08:04:08 -0500

    (Note: Underlined italics is where he quotes my previous reply.)

    I wanted to say its [sic] been nice to have some real dialogue about our subject matter, thanks

    You're welcome. I hope you don't mind that it usually takes me so long to respond.

    continuing....

    The difference between nouns and pronouns is not always clear, but I would consider 1 and 2 (of the first part) to be definitions of nothing as a pronoun. Definition 3 (which is basically equivalent to the entire second set of noun definitions) I admit I didn't think of, but I don't think it's what you meant, anyway. Definition 4 is "nothingness" in my dialect.

    I have to admit my use of the word "nothing" has been a source of some ambiguity, so the best solution to me at this point is to be very explicit in how I define it in each instance I use it.

    I'm not sure in what sense you mean "outside", but I don't think it's important for your argument

    I think I use "outside" in the sense that there are things that are precluded from being "inside" a state that is defined as objectless I don't think it's necessary for comparison, so much as it's necessary because there must be something doing the recognition.

    Perhaps "comparison" is not the best word, but my viewpoint is that to say that something is "different" (different - adj - having dissimilarities; unlike) there must be another object that you are comparing it too. So to recognize an objectless state, you need to compare it to a state with objects.

    I don't follow this at all. First you show that a non-empty state is a necessary condition for recognition of an empty state, and you seen to conclude from this that an empty state is a necessary condition for the existence of a non-empty state. Either I'm missing something or you skipped a number of steps in your argument.

    First I want to apologize if this is hard to follow, I admit, I made a statement without explaining clearly my reasoning behind it.

    Nothing could be described as that which is "a homogenous, isotropic, ignorant emptiness".

    If we have a non-empty state that contains objects, then we are saying that there are entities in the non-empty state that can be defined as objects. For an object to be defined, it must be bounded in some way. There are many ways in which it could be bounded, but there must be some dividing line created or described that limits its scope. If the object is in one location, than [sic] it can not be in another. If we look at the location where there is no object and ask "what is there", we say "nothing", because it does not contain an object.

    One way to look at this is too [sic] think of what would happen if all objects were completely unbounded, this would mean that all objects would overlap and coexist everywhere. A universe of unbounded objects would be a homogenous, isotropic mess where one could not distinguish between any one object, as they would combine to create one super-object with no observable features.

    If I redefine "nothing" as "that which makes up the region or boundary between any bounded objects" (not to be confused with "space" which should be considered an entity or object in its own right), then I may have a definition for "nothing" that shows that objects exist on the "surface" of "nothing" and without the underlying "nothing" than [sic] they can not be independently defined, and if they can not be independently defined than they become the substance of some super-object (which is the homogenous, isotropic mess we identified earlier)

    A curious thing is happening here. Earlier I looked at describing "nothing" as being "a homogenous, isotropic, ignorant emptiness", which in reality appears to look a lot like the "homogenous, isotropic mess (super-object)". Since there are no distinguishable entities internal to our super-object, we can say there is nothing internal to our super-object which qualifies as an object. So internally our super-object contains no objects, and therefore is empty of objects. It would also seem to be ignorant, for without objects how can it have knowledge of them?

    So is not the non-empty state containing objects actually dependent on "nothing" ?

    My argument from this logic is that without "nothing - that which makes up the region or boundary between any bounded objects", than the non-empty state becomes identical to the empty state, so to exist as something distinguishable from the "empty state", the non-empty state must have objects bounded by "nothing". Thus it must acknowledge that there is an "empty state" a prior to the non-empty state, and if the "empty state" is a priori, then the non-empty state must find its cause in the "empty state". My more concise argument looks like this: There is awareness of a state that is empty of objects For this state to be perceived there must be another state, outside the first state, that contains objects To recognize an empty state there needs to be a non-empty state for comparison . The empty state is not perceptible on its own However, for the non-empty state to have objects, those objects must be bounded These objects have empty regions or interfaces (boundaries) between them What defines the empty regions and interfaces is found in the empty state Without perceiving the empty state, the non-empty state can not bound its objects If the non-empty state can not bound its objects it looks like an empty state So the perception of the empty state gives cause to the non-empty state

    But "nothing" is not something, it is the lack of something. How does springing forth from nothing (or "nothingness") differ from not springing forth from anything? This still seems to me to be the White King's error on a more subtle level. You seem to be assuming that if a state can be described then there is something attaining that state. This is actually somewhat similar to the claims about relative existence I've been making in my remarks on this page. A universe which contains nothing is certainly consistent, but it is a universe without cause or effect, because cause and effect imply both the existence of causes and the existence of time, neither of which is compatible with the state of nothingness.

    In a nutshell, I'm saying that "nothing" is not "something" or a real entity, but "nothing" is that which bounds real entities.

    OK, I think I understand your basic argument. I have a few objections, though. Firstly, you define "nothing" twice, but I don't think you're being careful enough to distinguish the two definitions. If you define a word differently in different contexts you can't use results obtained in one context in the other. You seem to apply conclusions you've drawn about the gaps between objects nothingness to the empty space nothingness.

    Secondly, it might be possible to have a universe where objects do not have location, but are distinguished by some other property. Also, location is only one distinguishing property in our universe, and there are other gaps, say in electrical charge. There's also the possibility that in some universe space is entirely divided into objects, or at least substance (This objection is kind of nit-picking, though.)

    Thirdly, you say that "nothing" by your second definition should "not to be confused with "space" which should be considered an entity or object in its own right", but I don't see the distinction. Or rather, I don't see how that "nothing" isn't just a special case of space.

    I don't think "real" is as straightforward a concept as it's usually taken to be, but it seems that whatever meaning is assigned to it would include physical position.

    I have a problem with "real" as well, if we accept the existence of other hypothetical universes, how can you define "real" in an absolute sense? Particularly to an omnipotent being?

    You can't, really.


    From: Angelo Salvo
    Date: Mon, 23 Feb 2004 19:11:43 -0500 (EST)

    I have been an atheist for nine years of my life now, and it isn't because I'm stubborn and think that God shouldn't let bad things happen to good people because though valid it's slightly childish to me which is the point I'm trying to make. By joining organized religion you are treated like a Lamb of God as it says in the Bible, unfortunately lambs are lead to the slaughter. You should be a good person, without a doubt but you don't have to have faith in God to do so. Every religion on earth is based primarily on just one law, “Do unto others as you would have them do unto you” and you definitely don't have to study the Bible to learn that, it's taught since you are old enough to listen.

    Don't get me wrong I actually do think religion is one of the greatest things ever fabricated by man. It does help people to see the fault of their ways through stories devised by people who wanted to change the world to their liking. Our voice today is the internet and back then it was literature but let's face facts, you can't base your beliefs on ideals thought virtuous in times when people were barbaric. It isn't feasible to believe you can be a good person by following a set of rules laid down by a barbarian. Why not just take congeniality lessons from Genghis Khan.

    Of course, modern believers tend to just ignore the parts of their scripture which justify barbarity. However, once they start picking and choosing which parts of scripture they accept they're at a loss to explain why they need scripture at all, instead of just applying the criteria used to pick sections of scripture directly to their ideals and beliefs.

    I suppose the foundation of my argument is that God is no more than a father figure but there hasn't been a legitimate sighting of the Lord's work in approximately 2030 years, so people run away fearful of a father they've never seen or met. Would you be afraid of your father sending you to your room if you had never seen him? The only logical explanation behind the existence of God is that he doesn't exist, besides if he/she were all forgiving why would anyone be afraid to go out and commit murder every day? It doesn't make sense to anyone; even devout religious people have days when they doubt God's existence. I don't care if you're Charles Manson or the Pope everyone has a point in their life when they stop caring and don't believe.

    My last statement is that if you are afraid of going to hell for being an atheist, don't be. If there is a God there cannot be a hell because the story is that Lucifer was of course an angel who defied God, this started a war and in the end St. Michael banished him from heaven but because of Lucifer's awesome power he became the ruler of hell. My point is that, if this is the story that Christians want to stick with, who was there to witness the events that took place between God's angels and Lucifer's demons (fallen angels)?

    Actually, I believe that a lot of the commonly accepted stories about Lucifer, St. Michael, etc. are just popular stories, and not based on scripture or dogma, even though I, for instance, was taught them in Catholic school when I was growing up.

    I'd like to say that I appreciate the reason for starting your website; I didn't know sites like this existed until I was told of my estranged grandfather's involvement toward these websites. Thank you, I think it was very insightful of you to create a site to explain not so much beliefs but more disbelief.

    You're welcome.


    From: Bomar Angelopolis
    Date: Fri, 16 Apr 2004 00:15:30 -0700 (PDT)

    The reasons that people have to believe in an essence would be, everything but logical. Ignorance is a predictable engine of self preservation. Early in life people tend to have a natural dependance on higher individuals (adults) and this dependance survives until adulthood. The shadow of a father capable of doing the things we are unable to is etched into human genes. Any belief has primitive roots.

    As an analogy, in an experiment of cognitive development with a groups [sic] of 3-year-olders an instructor asks toddlers the names of different items: a ball,an apple, etc. they will be able to name each. When kids are shown a cookie and asked "how many" most of them are able to say "one". However when the instructor splits the cookie in two halves and repeats the question: "How many?" kids usually say "2" (cookies). Reason: Given their Preoperational level of cognitive development "a half" is a rather abstract concept. Therefore, the most logical answer is "2" according to their experience.

    At a different level, just because the world is experienced by senses at a primitive level any culture would say that a deity has hands, eyes, ears and attach as many familiar elements as possible.

    At another level people would attach words and meanings to a deity those words were eventually written and used as social religious codes.

    At another level philosophers would attach meanings to meanings and use them to define more abstract concepts.

    If a God existed would that entity need to have hands?

    If not, then would the use [sic] words, verbs or thoughts be necessary? Why would it need to exist? Would it need to exist just because we commonly use that verb? If it did why would it need to?

    Wouldn't all of this be a re-enacted exponential version of human characteristics but nothing else?

    The concept of a God is the need of a father figure as adults. The use of words and meanings does not solve anything. Why would you attach them to something never existed when even the verb "exist" is necessary? Just because we understand the verb itself an we attach a meaning to it. Thus this does not imply that it is universally valid.

    An Atheist Apology by Denis [sic] Paul Himes resembles the experiment of cookies and toddlers with individuals at different levels of cognition.

    I find interesting that given a religious oppressive background, Denis [sic] Paul Himes feels the need to make people aware of the level of truth he has found after reasoning.

    Move on, because they will not understand, stop breaking cookies into halves and asking them questions. A continuous denial suggests an affirmation or vice versa.

    Denis [sic], it is not necessary to show people the truth.

    (Even if the last statement is an oxymoron)

    (Or the mail itself)

    Necessity is not my criterion for action. I do many things which are not necessary, but which are nonetheless useful. Most believers who read my essay are not going to be convinced, but a few may be, if not by the essay alone then by the essay in conjuction with similar arguments by others. I know of one reader who abandoned Agnosticism for Atheism after reading my essay. In any event, even if it isn't persuasive it helps people understand me, as I explained in the essay's introduction.


    From: Kristen Gardner
    Date: Mon, 17 May 2004 19:16:07 +0200

    I wish I had time to reply to everything on your web page, but I cannot. I have to say that I admire you. It is evident that you are a true intellectual and that you have reasoned and thought through everything very thoroughly. I, however, also have to say that I completely disagree with you, and I just wanted to ask you a question or two.

    If the Big Bang happened at time 0, as you claim, what caused this Big Bang to happen at time 0? And why did time begin at that time? Something had to have existed before the Big Bang for it to happen.

    No, it didn't. The Big Bang appears to have been the beginning of time, and if it was the beginning of time then there was no "before the Big Bang" and its causation not only isn't necessary but doesn't even make sense.

    Or something had to have caused the Big Bang to happen, if it happened. Nothing egenders nothing.

    Even if that's true, it's irrelevant to this case, as there was never nothing. The universe existed for all time; it's just that "all time" is finite.

    You have a very big trust in Science and in Scientists. It makes me wonder... are they always so perfect? I think that in 50 or 100 years we are in for a big shock. We do not know as much scientifically as we think we know. At the end of the 1800's, they also thought they knew a lot, and yet, they were very far from completely explaining the universe.

    But they were far closer than religious leaders were after millennia of studying scripture.

    You say: "The Argument from Design is the claim that complexity implies a conscious designer and that God is the designer implied by the complexity of living organisms and/or the universe as a whole. This argument has become moot now that it has been shown, most notably by Charles Darwin and his disciples, how complexity can arise without conscious design."

    Did Darwin prove that? I rather thought he said that if his theory were true that the fossil record would show so. He himself recognized that if the missing "link" were not found, that his theory would have no foundation. As far as I know, it has not been found. Not to mention many other arguments. I wouldn't believe everything you are told science proves until you investigate it a bit further and question whether or not it truly is proven.

    I don't know what you mean by "the" missing link. Darwin's theory predicted that subsequent fossil discoveries would include intermediate forms, which they have.

    There's a reason that that part of the Apology is so short. Creationists lost their argument before I was born. Their only remaining adherents are those who are aggressively ignorant.

    If you are standing, looking at the chips inside of a computer, and someone comes up and tells you that that computer was made by chance, what would you do? Laugh in their face. "No," they say, "Over time, really.. believe me. They just kind of fell into place, and look what they made! Amazing, isn't it?" Would you believe them?

    Of course not.

    If you study DNA, which is thousands times more complicated than a computer and contains much more data, how could you claim that it was "chance?" Or a human eye! Or thousands and millions of other amazing things in this world! How could they happen by chance?

    There are billions of people in the world, and I suppose it's possible that one of them believes that living creatures were formed purely by chance. If so, I haven't met him. The great majority of us know that it was a result of differential survival rates of inherited variations.

    Well, I would love to respond further to all your arguments, but I don't have the time! However, I hope that one day you may find the Truth because the Truth will set you free.

    So long as it isn't blocked by its mortal enemy, Religious Faith.


    From: Florian Blonigen
    Date: Thu, 28 Dec 2006 16:56:50 -0800 (PST)

    I would like to respond to your atheist apology.

    Thanks! I'ts been a while since I had some commentary.

    I am a Catholic and convinced that the theistic arguments prove God's existence. What perplexes me is why they don't convince everyone else. Therefore, I am always eager to figure out what is going on in the atheist's mind.

    You divide your apology into three parts: atheist, theist, and pseudo-theist arguments. I am not interested here in the pseudo-theist arguments and I presume you do not find them convincing anyway, so I won't comment on them.

    You note three theistic arguments: the cosmological argument, Anselm's ontological argument, and the argument from design. Again, I presume you don't find any of them convincing, otherwise you would not be an atheist. You note that the best arguments are cosmological, and I would agree. I am convinced that ontological arguments are all logically flawed and arguments from design are always problematic since they seem to deny widely accepted Darwinian principles. But what is wrong with the cosmological argument? (Perhaps I should email you my own version of this argument.) You say: "One difficulty with this [argument] is that God's decision is itself an effect which presumably [requires] a cause, and so the problem of a causeless effect is just pushed back one level instead of resolved." I will address this difficulty shortly. Anyway, right after that you say: "A greater difficulty, however, lies in the assumptions made in the original problem. A causeless effect is problematic..." Well, yes, it is problematic. To get around this problem we postulate that every effect requires a cause, so that there are no causeless effects. Ultimately, every effect requires God as a cause. I think you are playing right into the theist's hands, here.

    Who do you mean by "we" when you say, "To get around this problem we postulate that every effect requires a cause"? In the Apology I note that it's problematic because "when looking at it with time reversed it is equivalent to an effectless cause". I then go on to show how the Big Bang avoids that difficulty.

    There is a problem for Theists here, though. Either they do or do not believe that every effect requires a cause. If they do believe it then they believe that God requires a cause. If they do not believe it then their cosmological argument disappears.

    You also provide three atheistic arguments. On "the vanity of humanity", I will not comment. With the appeals to Occam's razor, I have no quarrels. I agree that we should not posit gods or any causes beyond necessity, but I believe the cosmological argument does require us to posit the existence of some sort of god as opposed to no god at all. Then there is your argument from the contradiction of omnipotent agency, which I thought you argued very well. I have never considered this idea that "theoretically possible" and "actual" are arbitrary labels when applied to the universe. However, I have heard of arguments from the contradiction of omnipotent agency before, and I believe yours as well as other such arguments make the same oversights, which I will now discuss.

    Arguments from the contradiction of omnipotent agency all boil down to the problem of how God could be both omnipotent and omniscient. If God is omnipotent, He has the power to do anything. If He can make decisions, then omnipotence means that He can choose to do whatever He wants. However, if God is omniscient, then He already knows what choices He will make, if He knows everything. This means He really has no choice at all. God is not free to do whatever, but must do what He has always known He would do from all eternity. God cannot go back and undo His decisions which He knows have been eternally decided, so to speak. But if God is not free to do or create what He wants, then He is not an omnipotent agent, and hence we have a contradiction.

    Now, let us stipulate the point that no agent can go back into the past and change his/her free decisions. This is as true for God as it is for any other agent. But this does not mean, of course, that those free decisions are not free; nor is there any contradiction in knowing what one's past decisions were. This is so even for an agent that is completely free (omnipotent) and has complete knowledge (omniscient).

    This is not quite the same as my argument. You are assuming that decisions are made and the question is whether or not those decisions are "free" by some meaning of the term. I claim, however, that there are no decisions possible, free or not, from an omniscient being's point of view.

    To avoid contradictions, though, when we consider God, we have to remember that God is traditionally thought to be outside of time. Being outside of time in itself seems to deny the possibility of agency, for as you say yourself, "being an agent implies a privileged temporal reference, a present dividing past and future," meaning one is in time, not outside of it. But let's consider the possibility of non-temporal agency for a moment. We can certainly imagine temporal agency, even if the agent exists for only a short time, in which case it must make decisions very fast in the short amount of time it has. For powerful agents like gods, though, short time spans are not a problem. With sufficient power, such agents would have the ability to make many decisions, even decisions that turn out to be very rational and good, in a very short amount of time and with little deliberation. The next step is to simply take the limit as an agent's power goes to infinity, in which case the agent becomes the omnipotent God. An agent with such power should be able to make decisions in no time at all without any need of prior deliberation. It is this sense of requiring no time that God is non-temporal.

    The limiting case reduces the decision to a point in time. It does not remove it from time. There is still, in the limiting case, a distinction between "before the decision is made" and "after the decision is made".

    So I am not convinced that an agent implies a "privileged temporal reference." It may in a sense imply it but does not necessitate it, at least in the case of God. Besides, a privileged temporal reference is "incompatible with omniscience, which includes an equally perfect understanding of every point in time and therefore no distinction between past and future" as you say in the very next sentence. Well, then, I suggest we junk the privileged temporal reference in the case of God and just say that God is non-temporal, as most theists do. If God is non-temporal, then He has no past or future; He is just present.

    Further, if God has no past, then He has no development. So there was never any time in the past when God knew less than He does "now" (I put "now" in quotes because temporal terms don't have a lot of meaning when applied to God). God did not develop complete knowledge through time. That is incompatible with omniscience. There was no time when God was not omniscient.

    What does "knowledge" mean applied to a non-temporal "being". For that matter what can "being" mean applied to something non-temporal? It's easy enough to remove temporality from your definition of God when it becomes inconvenient, but then you shouldn't continue to assign activities to that God, such as "make decisions". Even an instantaneous decision implies a difference in mental states between the time before the decision and the time after, which means the decision maker exists in time.

    But what about omnipotence? Doesn't decision-making take time, even for an omnipotence? Why should it? A real omnipotence should be able to make decisions in no time at all, even if it is a mystery as to how He does it. In fact, an omnipotence should be mysterious. For many philosophers and theologians, mystery is the essence of God. What about a state that God is in "prior" to making decisions, a state of "not having yet made a decision", even if it is not temporally prior? I would agree that it is meaningful to discuss such a hypothetical "state", only I would not call it a state "of God". It is not a state that God could be in, because I think He would not be existing in such a state; and if God were not existing, nothing else would be existing because nothing would be caused. This hypothetical "state" would be complete nothingness. There is no such state because we know that there is something rather than nothing. In sum, there is no state of deliberation or whatever that God is in "prior" to making His decisions. Rather, there is simply the state, or rather, an act of God deciding to will what He wills and know what He knows, which is an alternative to not making any decision at all. Without divine decision-making, nothing, not even God, would exist. The alternative of nothingness is not a state that God can be in.

    In this paragraph you repeatedly assume your conclusions. You assume not only that the universe was created but that it must have been created by God. I not only dispute that the universe was created but claim that the creation of the universe is an incoherent concept, since potentiality of a universe is sufficient for its existence relative to its inhabitants.

    Also, you use of "will" is confusing at best. What could it possibly mean for a non-temporal "being" to "will" something? For that matter, what could "make a decision" possibly mean when there was no time the decision was not made?

    Note, also, the word "alternative". The alternative state of nothingness really could have been chosen by God, but He did not choose it, nor could He ever go back and change it (so there is a real limitation on His power. Also, alternative universes could have been chosen by God, so I believe it is meaningful to talk about "theoretically possible" universes, even though they are in a sense impossible if God never chooses them.)

    What do you mean by "choose" here? How do you know God chose this universe and not another? How would your life, knowledge, or experiences be the least bit different if you were a potential Florian Blonigen in a potential, but "unchosen", universe than if you were an actual Florian Blonigen in an actual, "chosen", universe?

    Nor did the alternative state exist somehow "prior" to deciding. A state of pure nothingness never existed and never will. The state is simply a real "alternative" to the act of God deciding, a state which was never realized, as opposed to a state realized "prior" to deciding. If it were true to say that it was somehow "prior" then, since "prior" is a temporal term, then it must be true to say that God does indeed undergo some temporal development. But the traditional conception of God has always been that He is non-temporal.

    Finally, since God is non-temporal, and God can make decisions in no time at all, all of His acts of deciding occur in one instant, and, indeed, they coalesce into one act. Further, the act of deciding can coalesce into the one act of existing of the one God. So in answer to the problem of "God's decision [being] itself an effect which presumably [requires] a cause, and so the problem of a causeless effect is just pushed back one level instead of resolved," I say that God's decision is not an effect which needs a cause if it is identical to God Himself, and God is not an effect that needs a cause. We could say otherwise, but that would imply that we can meaningfully distinguish between God's acts of deciding and a state in which God is not making any decisions and He is just existing (or deliberating). But we just explained why God was never in such a "state" and why He never could be.

    I think I really need to know more about what you mean by the word "decision". Your meaning is clearly far removed from the standard English definition, which involves a change in mental state.

    I also do not understand how "distinguishing between God's acts ..." means that God is not an effect that needs a cause when you're acting under the assumption that every effect requires a cause. Note that I don't make that assumption myself; I'm merely pointing out that someone either makes it or doesn't, and the cosmological argument fails either way.

    I also find it interesting that you identify yourself as a Catholic, and yet appear to deny the divinity of Jesus (who, if he existed at all, assuredly was a temporal being).


    From: Florian Blonigen
    Date: Sun, 14 Jan 2007 18:16:49 -0800 (PST)

    Okay, here is my reply to your questions in response to my commentary on your atheist apology. Sorry if it is lengthy.

    No need to be sorry. You'll forgive me though, if I just summarize my reactions at the end instead of interspersing them as I often do.

    In my commentary, I jumped to a bunch of conclusions that you couldn't accept, like the conclusion that God created the universe. I did not argue to this conclusion because it has been argued elsewhere. As I said before, I believe that the cosmological argument proves not only that God exists but that He caused the universe to exist. The specific version of the argument which I think is the best is the argument from contingency, which is the third of Thomas Aquinas' "five ways" to prove God, presented in his masterpiece Summa Theologica. I am sure you know this argument. It begins by noting that every contingent being requires a cause. If the cause is contingent in turn, then there must be something else to cause the cause. Ultimately, in order to surmount this problem of contingency, we must posit the existence of a necessary being. This necessary being is "what all men mean by" God, to quote Aquinas.

    The aim of my commentary was not to re-hash the cosmological argument. Rather, taking for granted that all theists accept some form of the cosmological argument as valid and therefore believe that God exists, my aim was to explain how they deal with the problem of the contradiction of omnipotent agency. If you want to challenge the contingency argument itself, we should do that in a separate discussion.

    I think the best way to continue explaining my position at this point is to work out some of the properties of the necessary being. Doing so will make it clearer why we should identify the necessary being with God in the first place. Instead of demonstrating these properties rigorously, though, I will basically concentrate on presenting the results.

    First, since the necessary being exists of necessity, it cannot cease to be, and is therefore imperishable. If it's imperishable, then it's eternal. Second, it is inconceivable; for if it were conceivable it would be contingent and not necessary. To be conceivable is to be able to exist in the mind somehow but not necessarily in reality. If it's inconceivable, then it's mysterious. One can also show that it cannot be limited in its essence, for then it would be conceivable. If it's not limited, then it's unlimited. If unlimited, then unrestricted; if unrestricted, then totally free. If God has any power, knowledge, or goodness, then these attributes must also be unlimited. Next, if it is unlimited, then there can be only one necessary being. If there were two, they would have to be somehow limited from being each other and from coalescing into one. But then they would no longer be unlimited. Further, the infinity of God not only requires that there be one God, but also only one divine act. All acts of God must really coalesce into one act. Finally, if there is only one necessary being, then God has no cause; for the cause would have to turn out to be a second necessary being. The cause could not be a contingent being since that's an effect of the necessary being rather than its cause. But doesn't everything require a cause? No, the argument from contingency says that every contingent being has a cause; it says nothing about God or any necessary being requiring a cause. So this is why theists believe every effect has a cause, but God does not: God is not an effect, He is simply a cause. Effects are temporal and therefore contingent (they don't necessarily exist), but God is eternal and necessary. Your comments seem to indicate that you don't know this, which surprised me. I thought I could just state that God does not have a cause without having to justify it.

    The property I wish to focus on here is the eternity of God. It is because of God's eternity that theists say that God is non-temporal. (Yes, Jesus, the Son, was/is temporal in some way. But why can't God the Father be non-temporal? At any rate, these issues deal with the doctrines of the incarnation and the Trinity, which have nothing to do with cosmological arguments. And further, even if the non-temporality of God were at odds with the doctrine of the incarnation, certainly it is true that the eternity of God is an orthodox Christian belief.)

    I think it is important to emphasize that eternity means "outside of time" as opposed to "lasting forever" or even lasting some indefinite time. God does not endure through time, but, as I said in the last commentary, God decides to exist and create in one act in an instant of time. This does not have to mean that God existed only at one moment in the history of the universe, e.g. at the big bang singularity, and not at any other time. Since time is a continuum, to exist "in time" means to exist during a certain time interval. (We have to recall our calculus here.) An interval must have some length, even if infinitesimally small. If infinitesimally small, then existence is indeed limited to a geometrical point in time. But there is another possibility for the case of God: the time interval is so small that it vanishes. With no time interval, God cannot exist in time, by my definition of "in time". If not inside time, then the proper thing to say is that God exists outside of time, if He exists at all.

    Paradoxically, then, if God's act becomes a perfect instant, he leaves time and becomes eternal in the non-temporal sense. "Eternal instantaneousness" sounds oxymoronic, but only if we think of His instant as a point of time in "our time" as opposed to "God's time". "Good golly, how many kinds of 'time' are there?" you ask. Well, as many as there are intelligent observers in the universe. We have to keep in mind that our notions of time were revised with Einstein's revolutionary theory of relativity. Time is relative, meaning relative to the observer. Each observer experiences time and can extrapolate from his/her experienced time to consider the time span of the universe, even if he/she exists during only part of the universe's history. Each point in the universe's history corresponds to a point on the time line of every observer. But it is still mathematically possible for the length of each observer's time line to be different. For observers moving close to light speed, lengths are contracted along the line of motion, making the observer's time slow down (relative to someone "not moving") or making the observer's time line shorten, in order for the speed of light to remain constant, as required by the fundamental postulate of special relativity. In the limit as one approaches the speed of light, the time line shortens to an instantaneous point.

    But "God is light, and in Him there is no darkness" (1 John 1:5). This, of course, means not physical light but some sort of metaphysical light. Perhaps there is a real analogy between the two kinds of light, though. If God is like light, then I would guess that, relative to God, the time line of the universe shrinks to a point of instantaneous duration. But the time line still spans the whole history of the universe. It's just that, relative to God, the entire history of universe seems to happen in an instant. Relative to us, it seems to take billions of years. On second thought, it doesn't even have to seem instantaneous to God. At least it doesn't have to feel like an instant as we humans experience instants. All I am doing here is appealing to relativity theory and reasoning that God's watch runs infinitely slower than ours, and so I am lead to regard God's view of the universe as instantaneous by comparison with our view. I am making guesses here, but I think things should be this way in some way. If God is omniscient, He knows the entire history of the universe. If omnipotent, He can watch it as it unfolds through time. But for God it unfolds in an instant, at least in the sense that He doesn't have to wait to know what happens. If He had to wait to know, then there was a time when He lacked knowledge and is therefore not omniscient.

    So, to conclude, I think God exists at all times in the universe's history, if we must speak in temporal terms about God at all. But relative to God, the time span of the universe collapses to one instant.

    There is still the question about whether there must be some state of deliberation for God prior to the state of having made a decision. You said in your responses that: "I think I really need to know more about what you mean by the word 'decision'. Your meaning is clearly far removed from the standard English definition, which involves a change in mental state." Yes, but we must also remember that God is so out of this world that any divine property, whether it be cause, knowledge, will, or love, will be very far removed from normal, every day definitions when predicated of God. That makes it difficult to talk about the divine properties, admittedly. For example, we may wonder what it means for God to love. How can a loving God allow so much suffering? Well, perhaps divine love is very different from human love. It is hard to define the divine properties. That's the first point. The second point is that we still should be able to discuss the divine properties to some extent. After all, if God really decides, then it means He decides, no matter how different the divine act differs from analogous human acts. So a word like decision should make some sort of meaningful sense when applied to God.

    So here is how I would treat the issue of divine decision making: an act of deciding is one act, so there is nothing problematic about it occurring in an instant. But you said something about there needing to be a distinction between "before" and "after" a decision is made even in the limiting case that the decision occurs at a point in time. I acknowledged the distinction, but I objected to the prepositions "before" and "after", because I don't think there are any temporal distinctions here. The important distinction is between "making" and "not making" a decision. Not only do we decide on options; we also deliberate; and we end deliberation by finally deciding to make a decision. This is really what God is doing: He is "deciding to decide." What He specifically decides to do is also an important question, but it is also somewhat secondary. If God is perfect, we can be sure he will make a great decision even without deliberation. It should follow instantaneously from the original decision to decide anyway. Besides, there are no other divine decisions and no other acts of deciding. There is only one God performing one eternal act. So if God is deciding to decide, if that's what the divine decision is, it should be clear that He could not be in a prior mental state of indecision, where He is deliberating on whether or not to decide. For to be in a state of indecision in that case is actually to make a decision: a decision not to decide. But if God does not decide, He does not act. If God does not act, God does not exist; and if God does not exist, nothing else exists for everything else is contingent on the existence of God, the necessary being. On the contrary, however, something does exist. So God is not, nor was He ever, in a state of indecision, in a state of having yet to decide.

    I don't know how to better make my case. Simply by using the principle of "divine coalescence of acts", we know that God's act of deciding is one with God's act of existing and any act of deliberation. I think what is confusing is that we think decision making is about passing from a state of indecision to a state of having made a decision. Without that state of indecision, then God is eternally in a state of having the decision already made. If decisions are already made for God, God has no freedom. I do sympathize with you or anyone who is confused about this; this is not easy to understand. Indeed, it is intrinsically mysterious.

    I would attempt to explain all of this as follows: let us admit that decision making is some passing from indecision to decision. Fine. But isn't this "passing" the very act of God deciding? Yes! But does this then have to mean that God passes from being in a "state" of indecision to being in a "state" of decision? No, He is in neither of those states. He is just in the "state" of passing between them in the act of deciding. If God were in a state of indecision, but still having freedom and power to make a decision, then God would have real potential to decide, if He has yet to decide. But potentiality is possibility, contingency, which is the anti-thesis of necessity. Catholic thinkers have long held, with Thomas Aquinas, that God has no potentiality; He is pure actuality, or pure act (no darkness, just light). This follows from the infinity of God, which secures the idea that there is only one God and even only one divine act. God cannot act in one way and still be in potential to act in other ways. He is either actual or not. He acts or He does not. There is no middle ground where he has mere potential to act.

    Think of the analogy of light, again. In relativity theory, light is always in motion, never at rest (at a point in space), always with the same speed relative to every observer, and thus is the one thing that is not relative. So maybe we shouldn't say light is ever at various points in space, but simply passes through points. In the same way, it is erroneous to think of God in states of being as opposed to merely passing through them or between them. He never stops long enough in one state to be really in it. Besides, to "stop" is to limit oneself, and God is not limited.

    So what I think is going on is that God is eternally making a decision, with no time when He was in a state of indecision and no time when He completes the decision process and ends up in a state of cessation of activity. Of course, if God were to cease to act, He would disappear from actuality anyway. God's reality cannot be a state, it is an act, as mysterious as that sounds. He is not static, but dynamic. In fact, all of reality/actuality is this way; it is real through acts of being or existing. You are quite right when you say that nothing about a universe changes whether it is potential or actual. It remains the same. But in one case, one and the same universe is performing an act of existing, while in the other case it is not and is therefore unreal. There is no change in the state, or the make-up, of the universe, but the acting makes all the difference.

    Now, I totally understand that one can get around these difficult issues simply by saying that God, or rather, the necessary being, does not make decisions. The problem is that we are then left without an answer to our question, "Why does contingent being or anything exist?" I appealed to a necessary being, but even that is not so necessary upon closer inspection. The cosmological argument proves that a necessary being exists; it does not insist that it exists by definition, though if it does exist it must exist necessarily. Philosophers tend to say it is only factually necessary (eternal and imperishable), not logically necessary. This does not have to mean, though, that God is ultimately just another contingent. Perhaps He is impossible, in which case he could be neither contingent, nor necessary, nor any actuality for that matter, for whatever is actual is also possible. (But if one proves that God exists, one can conclude that He is actual and therefore not impossible.) If we insist that the necessary being must exist simply because it is defined as necessary, then we are guilty of defining something into existence, which is the notorious flaw of ontological arguments. Nothing, whether necessary or contingent, is logically forced exist.

    Remember also that the necessary being really cannot be forced anyway; it is totally free. There are no logical laws, or programming rules, or higher causes than the one necessary being directing that being's activity. Thus, it seems to me that the only possible reason for why there is something rather than nothing is the result of a free act on the part of the necessary being. That is all that I mean by "decision". Beyond that, I don't know what it means for God to "decide". I don't think anyone really knows. It's just another mystery of His activity. At any rate, I think that best explains why I assume that the necessary being is some sort of a personal agent, one that is probably quite conscious and intelligent. An agent that is omnipotent and free, but unconscious and blind, would tend to produce sheer chaos. It would not produce this universe, or any other world with some sustained order, except through an unbelievably improbable accident.

    Much of your argument revolves around the premise that a necessary being created the universe. I have trouble with three parts of that premise, "necessary", "being", and "created".

    You seem to be using "necessary" (necessarius in Aquinas) to mean "existing in every potential universe", which is more or less how Aquinas uses it. You repeatedly contrast it with "contingent" (possibilis in Aquinas), which seems to mean "existing in some but not all potential universes". Aquinas based his theory on the unproved assumptions that every contingent thing has a cause, every necessary thing doesn't, it's possible for a necessary thing to cause a contingent one, and there are no infinite cause-effect chains, at least in the past direction. Of course, as I noted in the Apology, the Big Bang removes any need for a First Cause, and the creator god just becomes a solution in search of a problem, but Aquinas, at least, had no way of knowing that. You, however, need to explain why you think a necessary being exists. Much of your discussion seems to be of the form, "if there's a necessary being, then it's a necessary being."

    You seem to assume that a necessary being cannot have any contingent properties. Much of your property assignment to a necessary being is predicated on the assumption that to do otherwise would involve it with contingency. However, then you take a huge leap and assign human qualities to this being. You talk about it "loving", "acting", "deciding", "allowing", and so on without any justification that I can see that a necessary being would have these properties. In fact, all of these properties introduce contingencies, and should be excluded from its attributes, if your previous assumptions are correct. If a god acts, by any remotely coherent meaning of "act", then that god is different from a potential god who potentially acted differently, and therefore is contingent on that act, and not another, having been made. If the act is itself necessary, i.e. it could not possibly have happened any other way, then the god is not an agent.

    My main objection, though, is that in all of this you never explained what you mean by "creating" a universe. As I asked in my response to your first commentary, "How would your life, knowledge, or experiences be the least bit different if you were a potential Florian Blonigen in a potential, but 'unchosen', universe than if you were an actual Florian Blonigen in an actual, 'chosen', universe?". Until you can answer it, "creating a universe" remains an incoherent (or at best undefined) concept, and therefore not possibly (i.e. necessarily not) the act of an agent.


    From: Florian Blonigen
    Date: Mon, 22 Jan 2007 09:34:07 -0800 (PST)

    (Note: For this section, underlined italics is where he quotes my previous reply.)

    As I anticipated, you are now challenging the argument from contingency.

    Much of your argument revolves around the premise that a necessary being created the universe. I have trouble with three parts of that premise, "necessary", "being", and "created".

    Yes, that a necessary being exists and created the universe was a premise of my argument. I just assumed it and did not prove it.

    You, however, need to explain why you think a necessary being exists. Much of your discussion seems to be of the form, "if there's a necessary being, then it's a necessary being."

    Well, it is true that "if there is a necessary being, then it's a necessary being." But, if you wish, I will explain why I think that a necessary being exists.

    You seem to be using "necessary" (necessarius in Aquinas) to mean "existing in every potential universe", which is more or less how Aquinas uses it. You repeatedly contrast it with "contingent" (possibilis in Aquinas), which seems to mean "existing in some but not all potential universes". Aquinas based his theory on the unproved assumptions that every contingent thing has a cause, every necessary thing doesn't, it's possible for a necessary thing to cause a contingent one, and there are no infinite cause-effect chains, at least in the past direction.

    I disagree that Aquinas based his theory on unproven assumptions. They can be proven, as I will now demonstrate.

    As I have been claiming, the existence of a necessary being can be demonstrated using the argument from contingency. But, of course, you and others question the validity of the argument. Is there really a "problem" with contingency? How do we know that every contingent being requires a cause? What do we exactly mean by "contingent" anyway?

    Well, to answer the last question first, a contingent being, simply put, is something that is not necessary. By not necessary, I mean that it is possible, conceivable, coherent, or meaningful to say that a contingent being does not exist, regardless of whether it exists in fact. By contrast, it is not coherent to say that a necessary being does not exist. After all, it is necessary.

    So, while you don't say so explicitly, it seems you agree with my formulation that something is necessary if it exists relative to every potential universe, and contingent if it exists relative to some universes and not others. For if something did not exist relative to some potential universe then surely it's coherent to say that it doesn't exist. On the other hand if something exists relative to all possible universes, than I don't see how saying it doesn't exist can be coherent.

    An example of a contingent being is an effect, or a sensible effect, to be more precise. Effects that we sense exist in time and typically last a limited time. They certainly don't exist necessarily, for it is meaningful to say that there was a time when they did not exist.

    Next, there is the problem of contingency, or rather, the question of whether or not we really have a problem. Well, I think there is a problem. We do sense effects, so there are contingent beings. (They are actually there. Period. They don't just seem actual relative to me in what is possibly a "potential" universe.)

    I'm sorry, but saying "Period" is not a convincing argument.

    But since it is not necessary for contingents to exist, by definition, people are prompted to ask why they exist at all. There must be a correct answer to the question, "Why does a contingent being exist?"

    Exist relative to what? Existence is relative to a universe. If there were no contingent things then there would be only one possible universe. The fact that a universe containing a being is possible is sufficient to that being existing in that universe, and if there's another universe in which it doesn't exist, then it's contingent. I think you're overgeneralizing the laws of physics. Physics requires physical events to have a cause, because you can run the laws of physics backwards and arrive at that cause. That doesn't mean that the need for causes extends into the metaphysical realm.

    One may well respond by saying, "There is no answer to the question." But if that response is correct, then we have a correct response to our question, which I would argue means that we have a correct answer to our question. No matter how we think about it, there must be some correct response or answer to our question which forms the content of a correct theoretical account for our contingent being.

    So there is a correct theory of our contingent. But a correct theory amounts to knowledge, or at least knowledge we are seeking to acquire. And since knowledge is always knowledge of something, and since by hypothesis we already know about the reality of the contingent, this "something" must be something else besides the contingent. So we are always required to posit the existence of something else in addition to any existing contingent. Because this "something" functions as a sine qua non with respect to our contingent, it is appropriate to call it a "cause" of our contingent.

    I guess one could criticize this by saying we have jumped too quickly to the conclusion that there must be a cause for a contingent. Sure there is a correct theory, but perhaps the correct theory is that the contingent being has no cause. Maybe. But we don't know that it is the correct theory. That theory contradicts my conclusion that there must be a cause, so I am led to dismiss that theory as being incorrect.

    A further criticism is that to posit a cause is to posit something else existing besides the contingent. Though it may be granted that the correct theory amounts to knowledge, there is no reason to assume that any given item of knowledge is knowledge of something existing. I would agree that this is quite true. To know that 1+1 = 2 is not to posit something as existing. But I would say that any item of knowledge is true, if it is really knowledge. Truth, moreover, must be objective if it is really truth. So our knowledge puts us in contact with something objective, meaning something actual or real. And something real or actual is something that exists.

    Are we then saying that the equation 1+1 = 2 exists? No, I wouldn't say that. Let's take a closer look at what this equation means. It means if you have one thing, on the one hand, and another thing, on the other, then you have two things. The equation doesn't assert that anything exits; it only suggests the possibility. What is existing corresponding to our knowledge may not be the equation but the possibility of two things existing. So, at the most, all that this knowledge may be asserting is a truth in the realm of mere possibility, in contrast to the realm of actuality, which is the realm of existing things.

    But let's go back to our problem of contingents. Are we just assuming the possibility of contingents and all the imaginable theories that explain them? No, we are hypothesizing that contingent beings actually exist. Since we are not in the realm of mere possibility, I think that the correct theory of the contingent ought to claim that something actual, a cause of our contingent, actually exists. There is probably a better way to argue this, but I think what I am saying is correct. As I mentioned, many philosophers have discussed elsewhere the argument from contingency.

    Accepting the conclusion that the contingent has a cause, we are led ask what the cause is. If it is something contingent, then it must have a cause in turn. If this next cause is contingent, then we are going to have to posit yet another cause. Ultimately, we are going to have to posit a necessary being in order to surmount the problem of contingency. Otherwise, we are going to posit contingent beings forever. You mentioned the assumption of there being no infinite cause-effect chains. This also is not just another assumption; we can prove it. Every time we posit a contingent being, we ask why it's there. Since we know that there must be an answer, we end up positing a cause for that contingent. No matter how many contingent beings or causes we posit, even if the set of such things is infinite, we must always posit something more. So saying that we have an infinite set of contingent causes doesn't get around the necessity of positing the existence of a necessary being.

    Besides, the idea of an infinite regress of causes is absurd, if you think about it. We would have a runaway solution that would not stop until it posits the existence of everything we can possibly think of as a cause for any effect. This would include existence of God, it would seem. But if we agree that God exists, our debate is over. Not only that, the infinite regress solution would also end up positing the existence of all sorts of causal deities: Zeus, Athena, Shiva, Ishtar, etc. Admitting the existence of all of these gods would surely be an embarrassment to any atheist.

    Whoa! Where did this come from? Up to now your arguments have been reasonable, if wrong, but the idea that an infinite regress "posits the existence of everything we can possibly think of" is just plain bizarre. I'm at a complete loss how you got from one to the other.

    Also, in the last email, I showed that the necessary being has no cause, so once we posit it our cause-effect chain comes to an end. Thus, the chain of causes is not infinite, at least in the sense of being endless.

    You're leaving out the possibility that the chain has gone on for all time, but not for an infinite amount of time.

    There we have it: the proof that a necessary being exists which is the ultimate cause of everything. "Ultimate cause" of everything does not have to mean "creator" of everything. For me, "create" has the connotation of "active cause". The argument from contingency does not show that the necessary being is anything more than a "passive cause" whose existence is merely the necessary condition for anything else to exist. It is the actuality of the possibility for anything else, so I would identify the necessary being with the realm of mere possibility which I spoke of earlier. However, I think if one argues that the necessary being is a personal agent, as I tried to do in the last email, then God is clearly an active cause of everything. In that case, it would make sense to call God a creator.

    Of course, as I noted in the Apology, the Big Bang removes any need for a First Cause, and the creator god just becomes a solution in search of a problem, but Aquinas, at least, had no way of knowing that.

    On the contrary, I just proved that the existence of one effect or anything contingent is enough information from which to deduce that a necessary being exists. We can know this before we even start doing science. Neither the Big Bang nor any other scientific theory can revise the conclusion of the cosmological argument, it seems to me. When I speak of the correct theory for an effect, I envision all possible scientific theories (and for that matter, all non-scientific theories) that could explain it, whether past, present, or future. So I am not worried about any present or even a future scientific theory nullifying the cosmological argument. The argument already takes these possibilities into account.

    Remember that science must presume that there is factual data to verify their correct theories. But the very fact that such data or anything exists is not explained by science. The fact of existence is dealt with (though not fully explained) by the cosmological argument.

    But the Big Bang is a counterexample to your claim that the cause-effect chain must have had a beginning.

    You seem to assume that a necessary being cannot have any contingent properties. Much of your property assignment to a necessary being is predicated on the assumption that to do otherwise would involve it with contingency.

    I did not have to assume it. One can deduce it from the infinity of God.

    However, then you take a huge leap and assign human qualities to this being. You talk about it "loving", "acting", "deciding", "allowing", and so on without any justification that I can see that a necessary being would have these properties. In fact, all of these properties introduce contingencies, and should be excluded from its attributes, if your previous assumptions are correct.

    In the first place, these so-called human qualities are really divine qualities of which the human analogues are only dim reflections. Secondly, since God performs only one act, God's loving and deciding and what-not must simply be different ways of speaking of one and the same act. Since that act is necessary, the loving and the deciding, etc. are necessary. You say such properties introduce contingencies. If that is true, then, yes, it must be wrong for me to say that God "loves" and "decides". I tried to explain to you anyway why I do say that God "decides", but I apparently failed.

    If a god acts, by any remotely coherent meaning of "act", then that god is different from a potential god who potentially acted differently, and therefore is contingent on that act, and not another, having been made. If the act is itself necessary, i.e. it could not possibly have happened any other way, then the god is not an agent.

    No, there are no potential gods, at least none that are necessary. Potentiality would make God contingent rather than necessary. I have already pointed that out. Correct me if I am wrong, but I think that you are still conceiving divine action as something that a god is "doing".

    Forgive me, but at this point I have no idea what you mean by divine "action". It is obviously not an action. It is obviously not something that God decided to do. I'm not sure, though, what it is.

    I am saying that it is God's very being. We spontaneously think of something as being in existence and then acting or doing something. But God cannot be that, for then He would be performing at least two acts: an act of existing plus an act of doing or some other acting. But I have said many times already that God is only one act. Now, one might complain that if God is not first in existence before acting, then He would have no potential to act; for without something existing there is nothing which can act. Well, that's right, actually. God has no "potential" to act. He has no potentiality; He just acts. It may be action in a very remote sense of the word, but I would argue that the sense is still coherent.

    Also, I previously pointed out how, even if the divine act is necessary, it does not mean it is logically forced to exist. If it is not forced to exist, but freely "decides" to exist, then things could "possibly have happened another way." Now, it's not that God could have acted another way. His only other option was not to act or exist at all. So I don't think it makes sense to talk about potential gods who act in another way from the actual God.

    I will admit, though, that there is a sense in which the divine act is contingent. It is contingent on God acting. But that acting is the divine act itself, so the divine act, which is God, is contingent on itself, strangely enough. But by "contingent" we really mean something contingent on something else. So, in the end, God is not a contingent; He is necessary.

    I suppose I could continue explaining myself some more, but if your doctrine is that a necessary act cannot possibly be freely performed by an agent, then we just have a fundamental disagreement.

    My main objection, though, is that in all of this you never explained what you mean by "creating" a universe. As I asked in my response to your first commentary, "How would your life, knowledge, or experiences be the least bit different if you were a potential Florian Blonigen in a potential, but 'unchosen', universe than if you were an actual Florian Blonigen in an actual, 'chosen', universe?". Until you can answer it, "creating a universe" remains an incoherent (or at best undefined) concept, and therefore not possibly (i.e. necessarily not) the act of an agent.

    I thought I did answer it. I conceded your point that nothing about a universe changes if it goes from potentiality to actuality. Nothing about any x changes if it goes from potentiality to actuality. It's the same x, but once it becomes actual, it is an x that exists. I am not sure what the confusion is. Are you really saying that "actual" and "potential" are just arbitrary labels? Is there no difference between an actual polka dotted elephant and a potential one?

    A polka dotted elephant is not a universe. A polka dotted elephant is either actual or potential relative to a given universe. Relative to the universe we both find ourselves in it's potential (almost certainly). Assuming for the sake of illustration that there is nothing inherently contradictory about a polka dotted elephant, then there are universes relative to which one is actual.

    I just take it for granted that there must be some difference between these terms "actual" and "potential", otherwise we have these absurdities. Since I already admitted the actuality of contingents in this universe in the cosmological argument, I must admit the actuality of the universe. There may be other universes, and they are either actual or potential or impossible.

    Relative to what? You keep using "actual" and "potential" as if they have some metauniversal meaning, but you have completely failed to indicate what that meaning might be. It is certainly true that relative to this universe we are actual and a polka dotted elephant is not, but relative to the p.d.e.'s universe it is actual and we are not.

    I don't see what the conceptual problem is with making a distinction between a potential and actual universe. You yourself seem to admit that you might not understand what I mean by "being" or actuality, which may be the problem here.

    Perhaps I should also give an answer to your specific question:

    "How would your life, knowledge, or experiences be the least bit different if you were a potential Florian Blonigen in a potential, but 'unchosen', universe than if you were an actual Florian Blonigen in an actual, 'chosen', universe?".

    My life, knowledge, and experiences would not be different. It just would not be actual.

    Surely you can see that saying "a potential universe is different from an actual one because it's not actual" has no information content.

    It can't be actual, anyway, as there is only one Florian Blonigen. I don't believe I am existing in more than one universe, so there is no other Florian Blonigen in another actual universe. If there were, he wouldn't be Florian Blonigen. If he really were me, his experiences would also be my experiences (for Florian Blonigen is me).

    Nobody's talking about a Florian Blonigen in any other universe. We're talking about you in this universe. How do you know that you are actual (in some absolute metauniversal sense) and not potential.

    Alas, I cannot verify that I have experiences in a parallel universe, at least not yet. Maybe I will in the future. I hope for a resurrected life in a parallel universe, or "the world to come" called heaven.

    What about another Florian Blonigen in a potential universe as opposed to an actual one? Again, my life, knowledge, and experieces would not be different, just not actual. In other words, the nature of my experieces would be the same, I just wouldn't have those experiences. If I did have them, then you are right, I wouldn't be able to tell the difference between potential and actual universes. But I can't experience unless I have a consciousness, and in a potential universe it would only make sense that I would only have a potential and thus inactive consciousness. Only with actual, active consciousness could I be conscious of my experience in this other, potential universe. Without that conscious experience I would never be led to ask whether my universe is potential or actual. Not only that, I would never ask that or any question if I only had consciousness but my intellect was inactive or only potential. But if we don't ask questions in potential universes, then your dilemma disappears. Since we do ask whether our universe is potential or actual, the very fact that we ask shows that it must be actual.

    I'm sorry, but you're not making sense. If you were a potential human you would have a potential mind and that potential mind would potentially operate exactly the same as your "actual" mind "actually" operates. It would not potentially be saying, "Darn it, why am I only potential?". A potential Florian Blonigen would potentially be writing a potential email to a potential Dennis Himes which would potentially contain the potential words, "Since we do ask whether our universe is potential or actual, the very fact that we ask shows that it must be actual."

    How is that any different from what an actual Florian Blonigen would do? More to the point, how could the potential Florian Blonigen potentially figure out that he's "potential" and not "actual"? He couldn't, is the answer. The potential thoughts he would potentially think would be exactly identical to the actual thoughts of an actual Florian Blonigen.

    So my question remains, how do you know you're an actual Florian Blonigen actually thinking and not a potential Florian Blonigen potentially thinking? Can you come up with a difference between actual and potential which is not either:
    1) a reduction of them to terms which are relative to a given universe, or
    2) a set of definitions which reduce to "the difference between
    actual and potential is that the former is actual and the latter is potential"?

    You might insist that it does make sense to have experiences or consciousness in a potential universe. I would simply disagree. I think that "experience" is really a transcendental attribute of actuality or being, which is why I restrict that term for actual universes. For example, we can think about an electron interacting with its environment in a universe. But we can't say that the electron is experiencing those interactions unless the universe is actual. We could re-define experiencing as mere interacting, but I think we should acknowledge a broader, transcendental sense of the word "experience". We can understand how an electron interacts, but we can't understand an electron's experience, or even if it has any "experience", because we cannot be the electron. Not only can we not do this because we are not electrons, but we also can't be the electron if it does not actually exist. So I don't think it means anything for an electron to experience if it is not actually existing in an actual universe.


    From: Spike McLarty
    Date: Mon, 17 Mar 2008 18:56:07 -0700

    Hi Dennis - Nice work! After puzzling about the nature of God and Reality for most of my life, I'd come along the same track, but not quite as far. I'd gotten as far as condensing my thoughts into a sort of informal theorem: "Any complete (closed?) mathematical structure which contains intelligent observers is experienced by those observers as 'real' i.e. as a physical universe." I know it's not quite right, but I hope you see I was seeing the same thing you see, but from a more constructivist viewpoint.

    Yes, I agree. I'm not sure if complete or closed is the best term; these have technical meanings in Mathematics which are not really what you want. I might say "maximal event graph", or something like that, but your meaning is clear and does seem to be the same thing I've been trying to describe.

    I came to it the back way, by reasoning about computer simulations of - of course very small - universes. It took me many years to realize that if I created a simulation of a universe with inhabitants, that actually running the simulation did not affect the experience of the inhabitants! Run it once, run it twice, run it backwards - the inhabitants have the exact same experience. So the experience of the inhabitants doesn't depend on the simulation actually being 'run', it depends on only the structure of the universe. Which showed me that every universe that could be mathematically described must exist for its inhabitants if any. And any universe which could be described as a set of events, could be mathematically described - as a simple enumeration if nothing else. And at least scientifically, any universe we can observe is indistinguishable from a set of events. So... our universe was in principal indistinguishable from a mathematical object. Note that I assume that subjective experience is a natural phenomenon, no different from a ripple on a pond - but lots of people refuse that assumption.

    I'm not sure why, surely the experience is part of reality (regardless of whether or not it is interpreted correctly).

    From that I could conclude that most questions of the form "why is our Universe this way?" were misguided. No more sensible than "Why does 22 follow 21?" And the questions "Where did the Universe come from / Who created this Universe?" are based on an incorrect assumption. Our universe does not need to be created any more than the integers need to be created, certainly not in the sense of 'built from materials or components' or 'conjured into existence'.

    Exactly! The truth of "In the structure we call our universe Dennis Himes updates his website on 29 March 2008" is no more dependent on a metauniversal creator of our universe than the truth of "1+1=2 follows from the axioms of Peano arithmetic" depends on Guiseppe Peano writing those axioms down.

    I had not seen the next step! That a priori existence of all universes makes an omnipotent agent impossible! Or makes that agent indistinguishable from the set of universes.

    My guess is that if you hadn't read it in my essay first you would have realized it on your own before long.

    The concepts in your Contradiction of Omnipotent Agency section are, I believe, extremely hard for people to grasp if they have not spent lots of time thinking carefully about the issue. When I try to talk to people about this stuff, they look at me like I'm daft. The concept of 'real' is, like truth, taken by most people to be extremely simple and obvious, when in fact they don't understand it at all, and can't answer even simple questions about it that are outside everyday experience. There is a truly deep-seated, presumably biological, need to distinguish between what is 'real' and what is 'in your head'. People consider mathematical (or logical/abstract) entities to be 'in your head' and therefore 'not real'. For a lot of people, the idea of a mathematical object which is real is an oxymoron. Well, you can see the survival value of it! I still look around and say "This is a mathematical object" - I'm convinced by my reasoning, but it's hard to really 'feel' it.

    Some of what I've read about metaphysics from ancient India and Buddhism makes me think they worked this out long ago, and realized that 'reality' and 'thoughts' are in fact made of the same stuff i.e. the Cartesian duality is a false dichotomy - not because mind is a side-effect of matter, but because everything is 'thought' i.e. information.

    Anyway, forgive me for going on - Thanks for writing and publishing your essay, and the fascinating commentary.

    Thanks for responding to it.


    From: Leonardo Castro
    Date: Thu, 17 Dec 2009 23:05:21 -0200

    As English is not my mother tongue, I expect you to put many sic's in this message if you post it.

    You underestimate yourself.

    I have read your intelligent atheist apology. The arguments about the "real" label are particularly sophisticated.

    But it's a little curious to cite the Big Bang singularity as a "solution", while some other scientists view it as a consequence of our lack of understanding of the Physics at that "time".

    This is where your command of English has perhaps failed you, as "consequence" doesn't really make sense here. In any event, while there is much unresolved about the theories of the Big Bang, its very possibility eliminates the apparent paradox of a temporally finite past contrasted with the otherwise infinite effect → cause chain, and this narrow point was the one I was making.

    By the way, it's ironic that the proposer of the Big Bang theory was a Catholic priest, working for the Catholic church.

    Indeed.


    From: Leonardo Castro
    Date: Fri, 15 Jan 2010 18:26:01 -0200

    (Note: For this section, underlined italics is where he quotes my previous reply.)

    This is where your command of English has perhaps failed you, as "consequence" doesn't really make sense here. In any event, while there is much unresolved about the theories of the Big Bang, its very possibility eliminates the apparent paradox of a temporally finite past contrasted with the otherwise infinite effect → cause chain, and this narrow point was the one I was making.

    Yes. Agreed. If time depends on space, there's no sense in asking what happened before the Big Bang. The objective of my comment was not to invalidate this argument. I didn't mean that the Big Bang is a consequence of our lack of understanding, but maybe our treatment of it as a singularity, i. e., a point in time with infinite density and temperature. Excuse me, but I'm going to cite Wikipedia, not because I think that it's a scientific authority but because I have not a book in hands and this is well expressed there:

    "Extrapolation of the expansion of the Universe backwards in time using general relativity yields an infinite density and temperature at a finite time in the past. This singularity signals the breakdown of general relativity. How closely we can extrapolate towards the singularity is debated - certainly not earlier than the Planck epoch."

    By the way, some people argue that both space and time may be cyclic: the high density was caused by a Big Crunch before the Big Bang. So, when the Big Crunch happens again, we'll turn back to the beggining of time.


    From: David Palmer
    Date: Fri, 26 Feb 2010 16:13:51 +1300

    I found a guilty pleasure in reading your arguments, Dennis, because while I enjoy the intellectual game of forming intricate logical arguments and jousting in defence of them, I can't escape a sense of extraneousness. In other words, your arguments are clever but beside the point.

    Atheism in its foundation is just as non-rational as theism. The truth is that no-one knows whether life began due to a God saying 'Fiat Vita!' or a series of wildly improbable events. Whichever one you opt for is a matter of belief. Occam's razer [sic] arguments depend on the universe being fully explained,

    What a weird thing to say. What on Earth would make you think that?

    but since it evidently isn't, no-one is in a position to say that God is an unnecessary hypothesis. People often forget that materialism is an assumption. Scientists have found it useful for describing phenomena they come across in the universe, which allows them to develop things of material benefit for society; but materialism explains nothing. It is an expedient hypothesis, which is fine, but the general public seems to think it is 'scientifically proven', which it isn't. I once had fun with a scientist - I made him lose his rational objectivity in fact - when I asked him what electricity was, then pointed out that his answer described the phenomenon but didn't actually explain it. It took him a while to apprehend the distinction, but when he did he was reduced to saying that it didn't matter what electricity actually was - which puts science in its place as the study of matter not the study of reality. Until science produces evidence that they are material phenomenae, [sic] life, love, consciousness and the universe are outside its province.

    "Material phenomena" is redundant. If it's not material it's not a phenomenon. Life, love, conciousness, and the universe definitely are, and they are all well within science's province.

    In other words, your philosophical arguments are based on a certain assumption/theory/belief (the distinction being determined by your emotional commitment). Were you to make another hypothesis, say that God exists, your well-honed intellect would doubtless produce arguments just as compelling. What determines the assumption you make? Psychology and personal history. Nothing rational about it at all. Rational structures are built on the sands of non-rational assumptions.

    No they're not. I know some Theists would like to believe that Atheism is just as irrational and arbitrary as Theism is, but that's just not true. If you're going to claim that my essay was based on non-rational assumptions, you shouldn't simply make that claim, you should try to identify those assumptions.

    I might also point out that I have a Samoan wife, and her people, being not fully committed yet to the Industrual [sic] Age with all its psychological indoctrinations, routinely have experiences that are so far off materialistic screens that no atheist would dare go there. An hour spent listening to her and some of her relatives makes materialism seem rather silly. (And yes, I know the arguments challenging the beliefs of pre-industrial peoples.) My own researches have proven beyond doubt that clairvoyence exists, for example, and that sinks materialism at one blow.

    Another very weird statement. How on Earth would proving that clairvoyence exists sink materialism?

    The problem with clairvoyence is that you can't read a few books and then rationalise whether you'll believe in it or not. Like all core spiritual concepts - and for that matter all core scientific concepts - it requires personal experience, of the order of five or ten years, maybe, of self training to reach a position where you can offer an intelligent opinion on it.

    None of this is meant to be attacking of your speculations, incidentally. The Latin root of speculate means 'watchtower' - i.e. to speculate gives you a higher view. Since the root of 'comprehend' means to grasp or contain, and since it's impossible for two things to contain each other, and God and/or the universe by definition contains/comprehends us, it is impossible for us to comprehend God and/or the universe.

    This is just the etymological fallacy.

    But I think all speculation that is open for discussion is part of an essential evolutionary process. It helps us to a 'higher view', and it doesn't matter too much whether you see this in philosophical or spiritual terms, since these disciplines are both trying to do the same thing, only using different techniques.

    Yes, but one technique is based on understanding and the other on pretending. I happen to think that makes a difference.

    Anyway, all the best, and thanks for your patience in reading this.


    From: David Palmer
    Date: Mon, 22 Mar 2010 00:05:17 +1300

    Thank you for your response, Dennis. Allow me to address the points you raise.

    First, let me say I enjoyed your use of the word 'weird'. So you believe my remarks related to 'fate, destiny' (I use the Shorter Oxford), or by extension to the causative rather than phenomenal aspect of things - well, yes, my point exactly. But you appeared to be using the word in a negative way, which reflects a materialistic assumption: this brings me to your most important objection.

    "If you're going to claim that my essay was based on non-rational assumptions, you shouldn't simply make that claim, you should try to identify those assumptions." You're quite right and I apologise for not doing so. First to clarify: your assumptions aren't necessarily non-rational in themselves - though some are certainly questionable - but your commitment to them may well be non-rational, judging by the tone of your response. I mean non-rational as in emotional more than reasoned. This incidentally, is why "some Theists would like to believe that Atheism is just as irrational . . . as Theism is . . ." - it's not that the tenets of atheism are irrational, it's rather that the allegiance atheists have to them is irrational - i.e. less logical than psychological. Richard Dawkins is a prime example of someone whose atheism is psychologically akin to religious belief, but in fact a lot of atheistic writing on the internet has much the same emotional tone as that of religious fundamentalists. The moment a theory becomes a belief reason goes out the window, no matter how often the word is invoked.

    In considering your assumptions, let's begin with another of your comments, regarding life, love, consciousness and the universe, about which I implied doubt as to their being 'material phenomena'. You commented that, "'Material phenomena' is redundant. If it's not material it's not a phenomenon. Life, love, conciousness, and the universe definitely are, and they are all well within science's province." I'll grant that 'material phenomena' is tautological (I tend to distinguish crude phenomena such as a kick on the shin from subtle phenomena such as a room's energy atmosphere changing, but of course all phenomena in a material world must have a material component). And obviously life, love etc have a phenomenal aspect. But your implication that their causative aspect must lie within the province of a materialistic science is an assumption, since no evidence has been produced for their having material origins. Now, I would call this a non-rational assumption based on my studies and practical experience, but I can understand that if you've never explored deeply into spiritual areas you might not see why.

    Well, I admit I'm not sure how to respond to this which won't result in just going in circles. You claim that assuming that material phenomena are material phenomena is just as valid as assuming that material phenomena are not simply material phenomena. The history of science and religion for the last several millennia is full of situations where this dispute arose over various phenomena, and in every single one of them it turns out that those who claimed that material phenomena were material were correct. Given that history, I feel justified in saying that the burden of proof is on those who say that the 10000th time we investigate something like this it will turn out differently than the other 9999 times.

    Let's take another of your assumptions, in the last paragraph of the 'Causeless Effects' section of your Apology. I was interested that you called this a recent discovery of science, as I can remember discussing this idea in 1986, but perhaps for you this is recent. Anyway, you actually make the very point I would make: that what occurred before the universe is outside the province of materialistic science. But you relegate it as being therefore irrelevant, which of course is an assumption, and again I would tend to question its rationality. What my mum and dad were getting up to nine months before my birth is independent of the life I live in, but this life can't be understood without reference to it.

    You misunderstood the point I was making. I never refered to anything occurring before the universe. That's because "occurring before the universe" is a contradiction in terms.

    Now let's take the 'Contradiction of Omnipotent Agency' section of your Apology. A clearly presented piece of logic, and within itself perhaps irrefutable; but the God it posits is as anthropomorphic as a portrait of a bearded old bloke sitting on a cloud.

    No it's not. There is nothing whatsoever in that section which assumes that. All it assumes is that God is an agent.

    Your argument comes across as an intellectual game; fun to do and perhaps instructive, but extraneous to real life. The god obsession of atheism is why it seems irrelevant to those who follow a spiritual path. Judaism, Christianity and Islam are religious institutions, and religious institutions have nothing to do with spirituality: they are material entities focused on material survival which employ anthropomorphic definitions of God as a way of encouraging the allegiance of a materialistically inclined public that is superstitious enough, or has a sufficient sense of divinity, to want the consolation a religious institution provides. But Buddhism and Taoism take little interest in God, seeing God as off our screens, while those who follow a spiritual path within Judaism, Christianity and Islam understand that anthropomorphic representations of God are only symbolic, an aid to mental concentration in approaching that which cannot be comprehended. Your assumption here is that God can only be seen in this comic-book materialistic way as a kind of super-intelligent human being.

    I specifically stated in the Apology that there are different conceptions of God, and that I was treating several of the more common ones. I also said that, "I am a strong atheist by most people's definition of god but can become a weak atheist when people hedge their definitions enough."

    Basically you follow what from my experience is the usual atheistic tactic of taking cheap shots at the lowest common denominator of religious allegiance ("most theists" is the term you use), which enables atheists to feel clever and superior, meanwhile steering clear of more reasoned and challenging spiritual views, dumping anything bearing the label 'spirituality' safely out of sight into a 'New Age feel-gooder or rich funny-looking guru' bin. Arguments directed at the ignorant masses will impress neither the masses nor those with knowledge.

    Spiritual is one of the vaguest words in the English language. However, if you're using it in a specific sense, please enlighten us as to what that is. I'm interested in hearing the "more reasoned and challenging spiritual views", but I haven't yet.

    The final example of what I suspect is a non-rational assumption is this: you respond to my comment on the equivalency of philosophical and spiritual disciplines by writing, "one technique is based on understanding and the other on pretending." So how many hundreds of hours have you spent studying the Vedas or an equivalent system of knowledge and practicing the associated disciplines? If I was to say that nuclear physics is a pretence and then acknowledge my experience of it was limited to reading a few sceptical books, would you consider this a rational assumption? How many years of endeavour do you think it takes to attain a knowledgeable and hence rational view of spirituality? Have you done them?

    No, and I feel no need to waste my time on mythology when it has failed to demonstrate any indication that its adherents know what they're talking about. Unlike Nuclear Physics, whose adherents have provided ample evidence.

    Now for your taking exception to my remark, 'Occam's razer [sic] arguments depend on the universe being fully explained.' Your Apology's analogy of the woman, the milk and the hypothetical burglar was deceptive since she is in a well-understood situation where safe assumptions can be made. The universe is a mystery; science has barely scratched the surface of it: no assumptions are safe. Yes, I know that some scientific materialists like to convey the idea they have the universe pretty much sussed but for a few loose ends like dark matter and the like. But scientists are prone to hubris and it's in their professional interest to sound sure of things, and any study of the history of science suggests it's wise to be cautious when scientists are at their cockiest. Look at the confidence of science in 1900 and then at what followed. If the universe is a mystery then no theory can be dismissed as auxiliary or unnecessary until proven to be so. What appears as auxiliary in a partial view may in the full view be essential.

    What else? Oh, yes, this was a fascinating objection. You were unclear how the existence of clairvoyence sinks materialism. Quite why is a mystery. Perhaps you didn't know what I meant by clairvoyence. Granted, I was using the word in its broadest sense, so let's take a couple of 'concrete' examples - from personal experience. First, when a clearly perceived idea or set of ideas is communicated among up to six or eight people simultaneously without a material intermediary and despite their being separated by impervious material barriers (given the time constraint) such as distances and walls. Second, when in guessing the astrological signs of 373 strangers in controlled circumstances a success rate almost double that anticipated by chance is achieved. I really do wait with bated breath for your materialistic explanation.

    You know, you can win a million dollars with that. I wait with bated breath the news of your newfound wealth.

    Finally, my 'etymological fallacy'. Sorry, but that kind of dismissive labelling is just sloppy thinking. It's a fact that you cannot contain that which contains you. The word 'comprehend' means to contain, grasp with the mind, embrace and include, with all its other meanings implying this containment sense with the addition of a consequent and dependent knowing. The universe contains us physically. God, if he exists, must contain us mentally. You can dismiss this with as many knee-jerk labels as you like, but the facts remain.

    The fact is that it is the etymological fallacy, which is about as sloppy as thinking can get.

    Your Apology attributes the personification of the universe to 'the vanity of humanity'. I quite agree. The scientific and atheistic dogmaticism regarding God is another example of the vanity of humanity.

    Thank you for the opportunity to discuss these ideas.

    You're welcome. I apologize for taking so long to post this.


    You can go to the essay this page is in reference to or to my homepage.